Session One, part 4: The Fundamental Paradox of Late Twentieth-Century Thought

The Fundamental Paradox of Late Twentieth-Century Thought

Before I leave the sphere of Language entirely for today, our first day in “History of Literary Theory,” however, I’m going to ask you to focus with me in a very simple way on something I’ve been touching on repeatedly. It’s the way that human beings, even as newborn babies, possess something that I’m going to call “a set toward systemicity.” Newborns orient themselves to the faces of their birth mothers in the first minutes after birth in extraordinarily detailed ways. This has been closely documented. As soon as babies can focus their eyes (two weeks), they try to follow the trajectories of objects passing through their visual range.

If you think about the explosion of sensory inputs the baby must be experiencing when it emerges from the womb into this external world of light and sound and color and touch…. yet in the midst of this assault of chaotic sensory impressions, the baby already has seems to have an orientation toward “concerted” or “constituted” phenomena, toward “stuff that moves in concert” as distinct from “background.” They also know a lot about language structure and distinguish familiar voices. And the baby is already attending to these things months before it has learned the boundaries of its own body and distinguished where they leave off and the rest of the world begins, a process of separation, by the way, that happens through language, because it is through language that they emerging psychologically as a human “self ” that possesses an “I” capable of “knowing.” [Boy oh boy, do I have something to say about the convergence of Douglas Hofstdler’s work and poststructuralism!]

So the human mind is not stocked from birth with Innate Ideas, nor is it a tabula rasa, a “blank slate.” Plato was closer than John Locke, though, because human consciousness does innately set itself toward certain systematicities and orients itself to relevant coherencies, as though this chaotic and changeable world of physical sensations were lit up for us by flashes of white lightning, telling us what to pay attention to. As we notice patternings and fluid or dynamical “moving in concert,” that concertedness is of course not something apparent or apprehendable at any one instant in time. Already we are selecting and comparing and combining sensory impressions across time – whatever time may be – so that “time” is woven in some fashion into all of human “knowing,” from the outset. Language is acquired by human beings only because of this innate genius for orienting our awareness to dynamic coherences and patterns that are both temporal and formal in their constitution.

Furthermore, of course, this means human consciousness has some kind of profound entanglement with time: it is a “time-consciousness.” Time is for human beings always in some sense psychological time (as Augustine knew) – and this statement has nothing to do with it being “subjective” as opposed to “objective” and “external.” (Dated categories, unless they should be redefined and renewed.) Einstein introduced the human observer into physics in a much deeper sense than that; he showed that what we know through physics is always-already what we can know according to our attempts to make measurements, and he realized that this cut the link between genuine human knowing and any claims to an all-inclusive or universal knowing. [N.B. The claims of this sentence to be free from bias have been contested! See comment thread! : ) ]

The discoveries of physics may very well describe “reality.” [Why is reality here in quotes? Not because I doubt that reality is manifesting itself in an ordered way in the experiements and data and theories of physics! But because all of those are still representations of the reality, and by their highly focused nature are representations of certain selected features of the whole of reality. These gaps and qualifications are of vital importance, because they mark an advance in sophistication of the theories.] But we cannot claim that we can know that they do [without noting the mediation of our own efforts to know]. After the twentieth century, every statement of what is known in every discipline must be prefaced with the qualification “for human perception” [or, according to what the observer can measure relative to something — thank heavens the speed of light is not additive …] This sort of thing has to be dealt with every single day by physicists in the interpretation of quantum mechanics, where profound philosophical, ontological, and epistemological issues are being dealt with from inside of the narrow framework of a very highly specialized way of knowing. The results are both utterly brilliant and at time quite naïve. [Eech! These were fighting words! Sorry. But isn’t this true of every discipline’s attempts to deal with the work of other disciplines? I’ve learned a lot since I first wrote the preceding sentence!] This will hold true to efforts within every discipline, including my own efforts, so it is imperative that we talk to one another.

This kind of characteristically human “set toward systemicity,” it seems, is not operating for the autistic child, who does not make eye contact like other infants and who does not assimilate language in the precocious way that most children do. Temple Grandin, that amazing woman who has revolutionized the way animals are handled in farms and slaughter yards all over the world, explains that having been autistic from birth, she finds that she can readily perceive the way animals are perceiving their environments and recommend changes that would not occur to someone else.

But she has emerged into language and the human world only with the greatest difficulty and the most courageous effort. Most little children, for example, have no trouble with “this is a puppy and it goes Bow-wow.” “This is a kitty-cat and it goes Meow.” But Temple Grandin remembers spending hours as a much older child, studying pictures of dogs and cats, trying to figure out how other people could tell them apart. She finally decided it must be the length of the nose in front of the stop of the forehead. If the nose was long, then it was a dog.

This haunting awareness of “the trees,” so to speak, without being able to perceive “the forest,” is typical of autism. What is perceived is the haphazard concrete, so to speak, and there is great difficulty dealing with identities or classes and kinds of things. The “haptic” art produced by autistic persons is extraordinarily compelling visually, precisely because it is without perspective or a sense of the identities of things as wholes. On the other hand, most toddlers “get” dogs and cats right away – or at least they develop pretty good “working theories” about them very quickly. And then they refine those theories without difficulty. This sort of dynamic formalizing capacity seems to be lacking, to various degrees, in the autistic person.

With autism in mind, then, let’s return one last time for today to the way human consciousness is conditioned by its language very early on, in order to be able to perceive and communicate about the world. By the time we are apprehending words and phonemes and syntax and so forth, we have been fully conditioned to perceive selections and combinations of what is going on in the flow of physical sound, but we are never, on any structural level of our language, interested in all of the sound. A great deal of the empirical and quantifiable flow of sound passes us by as mere “static” or “noise.” It is empirically there, but we don’t notice it. For our deeply conditioned perception, much of the sound is not “there” because it doesn’t signify anything, with respect to the signifying systems we now have in place for making our interpretations. In contrast, then, autistic persons do not readily distinguish between what is to be taken as noise and what is to be regarded as distinctive, in an onslaught of sensory impressions. But most of us can and do, most readily, pick up the signifying clues, however subtle, by developing what we have learned to notice in language into a system that interprets language.

So yes, the world around us as members of a human speech community is a physical or empirical reality, if you wish to call it that. (When I read the Copenhagen physicists I thought they weren’t so sure, but that seems to have changed, perhaps. I’m confused about this and wish someone would help me out.) But we human beings do not perceive the physical reality that is there. Instead we are conditioned by language – and in the same way by disciplinary training and by our culture (which is a fabric of languages or signifying systems) – to perceive certain selective elements of the physical totality, and combine them in certain ways, in order to arrive at communally negotiable meanings, while disregarding all the rest of the physical order. It is not always the most physically prominent elements that we select, either. And we are incredibly good at doing this, which is why our human ways of knowing are so productive and even awe-inspiring. Still, there is so much “reality” going on around us (and inside of us) that we must always be blind to a whole lot of it.

For today, then, I’ll close this consideration of language by simply relating a story told by a high-ranking member of JFK’s cabinet. It will serve as a little parable, an illustration given to provoke thought. The story’s about a dinner at a swanky
Washington DC restaurant in the early 1960s, where the cabinet sat with President Kennedy for hours, discussing the question: “What do black voters want?” It wasn’t until years later that the cabinet member who told this story realized there were as many African-American men in that room as cabinet members. But they were serving as waiters and filling wine glasses and carrying dishes in and out, while the white men one by one gave their opinions about what blacks really wanted. While the African-Americans in that room were physically present, they were as good as invisible, because they didn’t signify. They didn’t “make a difference,” even when they themselves were the topic on the agenda. Now that’s conditioned perception! And future generations will look back on us with the same sense of shock and incredulity over what we aren’t seeing right now.

So there is always more in the flow of sound and more in the world around us than what we are conditioned to perceive and “count” as being significant. This is how our language – like our disciplinary discourse and like our culture as a whole – conditions our perceptions of everything, even while enabling us to share and preserve and enhance our perceptions. Without such sets of limiting conditions, no human community could ever achieve any shared knowledge of their world, or have a communal “world” to share. Nonetheless, these are still limiting conditions.

It seems to me that this is the fundamental paradox of later twentieth-century thought. I am so drawn personally to Derrida and Kristeva because they are always trying to work out ways to negotiate this paradox faithfully and honestly. Here again, though, I must note that this state of affairs – one that psychoanalysis and literary theory have opened up for us as late-moderns – was familiar territory to earlier Westerners. But we’ll consider that more appropriately later, as when we get to Boethius, Augustine, Aquinas, and Dante, for example.

For now, however, we’re just about ready to move on to our next domain of human mystery, the sphere of Art. And if you think language turns out to be a strange and counter-intuitive phenomenon – as soon as we stop simply using it and start looking at its functional structure, or start pondering its origin – wait ‘til we get to art. Well, stop a minute. We can’t leave language, can we, without mentioning the emphasis placed on language and especially on its “naming” capacity, in the magnificent story of creation that opens Genesis, “the book of beginnings.” First, Elohim brings the universe into existence by speaking it. “And God said, Let there be light, and there was light….” This is why the instrumental maker of all things – Wisdom in the Old Testament and Jesus Christ in the New – is associated with the “word of God,” because everything that has being is brought into being through being named. Then the human creature, “made in the image of God,” likewise brings into being by naming, when the living creatures are brought to Adam to be named. After all, Adam is naming the kinds of animals. He is saying “camel” and “donkey” [or puppy and kitty]  not “Joe” and “Pablo.” And while the particular animals in the story are physically there in the garden, the “kinds” of animals come into existence, at least for the human perception, by being named as kinds.

But they are named as kinds of things, it appears, according as they have their being as kinds of things. Let’s not fail to notice just how insistent this ancient creation poem is about the way each thing is made by God “according to its kind.” And so it was. “The earth produced vegetation: plants bearing seed in their several kinds, and trees bearing fruit with their seed inside, in their several kinds….” The ancient mystery, then, of how to group the animals into kinds or how seeds grow up into the cedar or the grapevine or the acacia tree…registered as supremely worthy of mention in a mythic, symbolic, larger-than-life narration of the beginnings of things. The same mystery that we probe today as the “information” that is encoded and passed down in DNA molecules.

What we’re going to be seeing when we turn to the Greeks is that these same insights about being human – that humans are able to name things according to their kinds – will come to us also from them. The Western human project of coming to know originates in our ability and desire to name according to their kinds, where naming comes in the Greek tradition to mean finding the logos or “formula” or “account” for each kind of thing. So, from the Pentateuch so revered in the Hebraic tradition, and from classical Greek philosophy – from two very different and ancient and profound traditions of knowing – we find humans knowing by “naming” (which is the Greek word logos) and we find humans knowing by “identifying the kinds” (which is also the Greek word logos, as we shall see). Both traditions are saying: “Here is the place where humans truly dwell.” As late moderns, we are deeply conditioned by our culture to ask, “Who am I?” But we are not necessarily trained to ask the much deeper question, “Who are we?” And also, as the scientistic late moderns we are, we often miss significant revelations because we are trying to reduce everything to a certain kind of fact.

[Please continue to part 5, Our Peculiar Subject Matter, when Dr. Blumberg adds more pages]

149 thoughts on “Session One, part 4: The Fundamental Paradox of Late Twentieth-Century Thought

  1. breid

    Janet, I have to apologize for not getting in here and making some more articulate comment than “oh, wow!” before this. But the wow, let me assure you, is genuine. You have been talking about this for so long that I (like Caleb) thought, okay, show me the money. And you have! I printed out session one and spent part of my Friday afternoon sitting with a glass of wine and working through it. I have some questions, but for now I simply want to encourage you to keep going. This is just the course I always wanted to take, and didn’t find offered in my program. Even though you’re gearing it toward undergraduates, as we all know really good undergraduate courses and graduate courses don’t differ so much in content as in the level of engagement. And I am engaged. I have a New Yorker article to share with you about language development. I want to know more about the science vs. theism debate. But for now, I really want to “think with you” (I love that) back to the Greeks and see what happens. Bethany

  2. David

    “Einstein introduced the human observer into physics in a much deeper sense than that; he showed that what we know through physics is always-already what we can know according to our attempts to make measurements, and he realized that this cut the link between genuine human knowing and any claims to an all-inclusive or universal knowing.”

    I’ve read a a good deal of the early work of Einstein on the photoelectric effect, specific heats of solids, special relativity, genereal relaitivity, his exchanges with bohr, his famour Einstein, Rosen Podolski paper. There is absolutely no evidence in any of that work in him
    realizing “that this cut the link between genuine human knowing and any claims to an all-inclusive or universal knowing.”

    A problem with us scientists taking pomos seriously comes with these false assertions that keep bing repeated. Einstein didn’t realize such a thing, and if he did it might be in a not too well known writing (if you point me to it, I would take a look). But as far as what I’ve read of Einstein, which is not inconsiderable, what you assert about Einstein is not true.

    I might disagrre s

  3. David

    I might disagree with some of your thoughts and conclusions, and we might argue about them, but it is increasingly hard to hold a debate when explicitly false assertions and abjudications get thrown around.

  4. David

    Now

    “So yes, the world around us as members of a human speech community is a physical or empirical reality, if you wish to call it that. (When I read the Copenhagen physicists I thought they weren’t so sure, but that seems to have changed, perhaps. I’m confused about this and wish someone would help me out.) But we human beings do not perceive the physical reality that is there. Instead we are conditioned by language – and in the same way by disciplinary training and by our culture (which is a fabric of languages or signifying systems) – to perceive certain selective elements of the physical totality, and combine them in certain ways, in order to arrive at communally negotiable meanings, while disregarding all the rest of the physical order. It is not always the most physically prominent elements that we select, either. And we are incredibly good at doing this, which is why our human ways of knowing are so productive and even awe-inspiring. Still, there is so much “reality” going on around us (and inside of us) that we must always be blind to a whole lot of it.”

    with this I agree and think it is true. On the other hand, a lot of the progress in science (and perhaps in other fields, but this I do not know as I am a physicist) has come from people who have concentrated and pointed out ignored parts of reality (ignored because
    we were predisposed to not look at them sometimes, nut not always, as other wise they were inaccesible to our senses and more primitive apparatuses) and have demonstrated their important consquences to many different phenomena.

    I emphasize that concentrating and drawing importance to previously ignored or previously unknown parts of reality is not enough for success in science, also there needs to be relevance as far as utility or as far as relevance to a large range of nature.

  5. Janet

    David, I appreciate you reading section 4 beyond words!
    Let’s talk about Einstein. You are unhappy that I claimed Einstein realized “that the observer in SP cut the link between genuine human knowing and any claims to an all-inclusive or universal knowing.” What am I claiming when I say this? That Einstein knew his work preserved the fundamental laws of physics for all observers, and believed this was genuine human knowing. That’s what I’m saying.
    He (and the other physicists of his generation) were raised and trained in the Newtonian world of absolute time and space, where every event was laid out on one universal grid, as it were, and they knew that had ended. I’m a historian and also I have read and read what the West was thought before Einstein. And believe me, Relativity and QM made a huge advance in our understanding of the nature of science and human knowing in general, along with Godel’s proof. All of this is discussed in Roger Penrose’s The Road to Reality and my golly, does that guy know his maths and physics!
    I’ve now realized that scientists read any mention of enhanced epistemic sophictication in science as an attack on science! They leap to the conclusion that what we really mean is that science is socially constructed and it’s results are illusory. No! I don’t know any postmodern thinker who holds this position! The epistemic qualifications worked out in Relativity and QM makes science even more wonderful and more honest and more legitimate as a way of knowing!
    Jacob Bronowsky was another of those world-class physicists among Einstein’s peers and he explains the huge shift beautifully in his Ascent of Man episode called “Knowledge or Certainty?” I think the Copenhagen school especially recognized the shift in the relationship between human beings and the physical world. Method and measurement mediate in human knowing in powerful ways.Scientists are always thinking about the pay-off, the end results, and their validity. Yes! But semiotic theorists are always thinking about mediation and methodology AS METHODOLOGY. To think we are dissing on science is such a misunderstanding.
    I think it was George Ellis in a thread over at cosmic variance who gave us a link to the Bronowsky episode on the web and I’ve hunted for it and can’t find it. (I have Ascent of Man in book form.) Can anyone find it?

  6. David

    Thanks for the clarification. I agree completely with “Einstein knew his work preserved the fundamental laws of physics for all observers, and believed this was genuine human knowing.”

    You also mention the Copenhagen school. My main line of work is in quantum mechanics and quantum field theory. But not for high energy physics, mainly for the physics of materials.

    Because of my work, even though I am a theoretical physicist, I am confronted quite a bit with reading, interpreting and vetting experiments and their results. Because of this the Copenhagen school is close to my heart. I would like to add some incomplete and perhaps rambling thoughts on it.

    Their relationship I would not necessarily say human beings, but observation, from the physical world. Now, this is a Pandora’s box. Exactly how to draw the line between observation and phenomena is an unresolved problem in quantum mechanics and was not solved by Copenhagen. When the demarcation is clear Copenhagen works beautifully so far.

    The attitude we take is a lot of times to ignore the problem of the demarcation. This is naive, but it is not as bad as it seams in the sense that this naive attitude is taken when there is littel doubt that a measurement has taken place. In such a case, it is the most efficient attitude to move our science forward.

    Now for the cases in which there is no clear demarcation of where the boundary between observation and phenomena occur, we are in unknown waters and we really don’t know what to do. Usually different experiments are performed until one hits on a measurement that has a clear demarcation or if this doesn’t happen progress stalls.

    Because of this situation little progress has been made in solving the biggest problems at the foudnations of quantum mechanics. Hopefully in the future experiment, theory or both would point the way to the solution and to more complete and new knowledge.

  7. Janet

    David, I’m so glad to find out that you have the Copenhagen School near and dear to your heart, because I am so intrigued about them, and as brilliant as their strict science was, I especially admire them for their brave and humane struggles to step back and ponder what QM “means” about the nature of reality, if it is so hard to distinguish exactly where reality leaves off and our observational attempts to know it begin. (My scientists friends have assured me that “interpretations” of that sort are not science per se, but are viewed by real scientists as merely “cocktail party chat.” But it’s that kind of chat that interests philosophical types like me and they were so brave to attempt it. Who’s going to guide the rest of us on interpreting QM, if the QM folks don’t?)
    You are saying, if I understood you correctly, that, for the Copenhagen school, the big change or the big “shift” that occurred with QM was a separation not between “human beings and the natural world,” as I had put it, but between OBSERVATION and the natural world, right? (I think a word or two got dropped in your comment and want to be sure.)
    That change in terminology is great for me. It does open a Pandora’s Box, doesn’t it? I’d like to ask you two questions.
    When you say “observation,” are you physicists thinking that observations are always made or staged by observers? And observers seem to be equivalent with big-brained intentional agents (like human beings but not limited to them)? Or can an observation ever be “purely mechanical”? Where does the collapse occur? If a mechanical device takes a measurement, can we know if the reduction takes place with the mechanical intervention or with the interpretation by the scientist? Maybe this is a silly question? But I’ve never been sure.
    My other question is, can you clarify or give examples of when the demarcation between observation and phenomena occurs clearly and “Copenhagen works beautifully so far”? I mean in contrast to a case or two where the boundary hasn’t been clarified as yet? I’ve read Bell and all about EPR and so forth…so maybe I can follow you. An example of how a certain kind of measurement gives a clear demarcation when another one doesn’t?
    Also, on Copenhagen interpretations of QM can you particularly recommend things to read? I remember studying “eight interpretations of QM” some years ago and I thought it was in something by Feynmann… Have you seen Whittaker’s book on Einstein, Bohr, and the Quantum Dilemma, by any chance?

  8. David

    Janet, I will start by the end of your post and then comment on other parts. I have not seen Whittaker’s book. I will think of possible things to recommend, but this is a tough request for unusual reasons. Being a physicist my conception of the Copenhagen interpretation was formed not from reading books on it, but by reading the original literature of quantum mechanics, quantum field theory ,and the reading of quantum mechanics books for physicists. It was also shaped strongly by my work as a physicist. So my conception developed from grabbing bits and pieces from sources not dedicated per se to the Copenhagen interpretation, but dedicated to quantum mechanics, and by my work as a physicist. My conception is also the bare bones version necessary to work as a scientist. Basically, the rules of quantum mechanics, the probability interpretation and the collapse postulate upon measurement. If you wish I can expound as to what each means to me more thoroughly. I will comment on the rest of your comment and try to provide the examples you asked for later today or at the latest tomorrow

  9. David

    Let me now go back to the beginning of your comment where you say

    “if it is so hard to distinguish exactly where reality leaves off and our observational attempts to know it begin. (My scientists friends have assured me that “interpretations” of that sort are not science per se, but are viewed by real scientists as merely “cocktail party chat.” ”

    Well, I don’t know if this is science or not, but is certainly a worry for scientists working in quantum mechanics and this is why the early quantum mechanics worried about such things. It is true that one can do a certain amount of quantum mechanics ignoring some concerns, but they do pop up here and there and especially early on they needed to be understood however incompletely.

    Let me explain why the issues of observation required the attention of the pioneers. Before doing so let me state that the demarcation between observation and phenomena is purely artificial in a sense. They are both part of reality, part of the universe and subject to the laws of physics. On the other hand in quantum mechanics(QM), their role is very different. In that sense QM is very different from all theories that came before because even though both observations and the phenomena on which they are carried out, they seem to operate under different rules. They are both part of reality, but they are different. This came out of a long arduous process of making sense of the theory and working out how it explained and predicted experiment.

    So in order to see how this came about, let me give some background in quantum mechanics. In the Schrodinger representation, QM is a theory of waves. In this sense it is no different than a theory of sound waves or than Maxwell’s equations for electromagnetic waves. Like any theory of waves when there are restrictions, they can oscillate only at certain frequencies. A couple of decades before when Planck and Einstein became the rebels to fire the first shots in the quantum revolution, they realized that oscillation frequencies correspond to energies of a system. So the first success of the young quantum theory was to account quantitatively for the energies of simple atoms and accurately predict the energies of other atoms and molecules. They accounted for and predicted spectral lines.

    Now at this stage, they needed to know what the waves represented, what characteristic of the system under study they described. This was a puzzle, which was cracked by Max Born whose two Ph. D. students Heisenberg and Pauli had already so much contributed to the development of quantum mechanics. He proposed that the square of the waves represented probabilities. This1926-7 insight he based on Einsteins work on the photoelectric effect of 1905. Since Einstein had proposed correctly that the number of quantized particles of light, photons, were proportional to the square of their waves and since the average number is proportional to probability it was a keen insight on Born’s part. Now, it has to be tested against experiment. It passed with flying colors and it keeps passing with flying colors in the countless QM experiments that are run each day in our present world.

    Now that we knew the waves had to do with probabilities of events, their square are probabilities a whole lots of questions arose. First of all even though the evolution of the QM waves is fully deterministic their consequence for experiment is not. This is a radical break from all previous understanding of nature. but, here comes the disturbing part, Copenhagen, if the waves are probabilities the include many possibilities. When a measurement takes place what happens to the nonmeasured possibilities?

    QM does not say what happens. No one knows. Actually the Copenhagen school made up a rule based on consistency. since the nomeasured possibilities did no occur, they are not there. What to do? By hand we slice the from the QM waves. This is the collapse postulate. We take them away by hand. This is a rule imposed by fiat, outside regular dynamics but without which, there would be no consistency and furthermore we could not predict the probabilities after the measurement since if we did so from the wave without the excised possibilities we would get wrong results.

    Now, this is deeply disturbing. We have a theory of physics with a rule imposed by fiat without which the whole edifice crumbles. Does it work? Yes. Do we need it? So far, yes. Does it define what a measurement is? That is when do we apply it? No!

    I know I still owe you examples, but I live in Holland and I am falling asleep… I’ll stop here and go on after I hear your comments, objections and qualms

  10. Janet

    Yes, I would love for you to do that!
    Can you clarify whether the collapse upon measurement is thought to mean that the collapse “occurs” (whatever that means) with the intervention of the mechanical device or with the interpretation by the (human) observer? Or is there anyway to separate those, since we can’t “know” the machine measured (or what it measured or what happened) until we “know” it, which means we ourselves have gotten the information… have observed a measurement…. Is there a way to get at that experimentally? (More simply, to distinguish observation from human beings observing?)
    I’ve been reading Hawking and Penrose and some others disagreeing over superposition and the famous “cat” and also Jacob Bronowski insisting QM means that we must have seek “knowledge” and not “certitude.” Every instrument that shows us more also excludes other aspects we want to know about, in his moving Ascent of Man. I want to say that science and (semiotic) theory have come to a similarly chastened and humbled position: that human knowing has some built-in limitations based on the way we use what is available to get at things, and then we need other things to get at more of it, and that this makes everything more exciting than insisting dangerously on fundamentalist certitude and triumphalism (“this is the ONLY way; we have THE absolute truth”).
    Making overstated claims about what we know and how we know it is intellectually unsupportable and our biggest enemy in the political arena. Yet we fear that giving up even an iota of our truth-claims will make us incapable of distinguishing between different positions in the political arena, or between fundamentalism and thoughtfulness. I don’t think is as big a danger as our own fundamentalisms are.

  11. David

    Quick before I go to sleep. We can’t know if it is the machine or if it is a human observer at our present stage of knowledge. We cannot know because of the reason you point out. We only now when we go and read the machine, we only know when we get the information. With my scientific background I prefer to think it is the machine. Actually some arguments could be made that it is the machine or the measurement even if it is not done by a machine, but none of them are definitive, and none are proofs. Since for you and I or anyone else to know, a human being to finds out to convey the information it looks that there is no way to know. I say it looks because who knows what progress and knowledge tomorrow will wash ashore?

    On the famous cat and other things I’ll comment tomorrow…

  12. Janet

    Okay, that really helps me. And it’s coming from the horse’s mouth, someone who works with this stuff all the time! Thank you so much.
    By the way, I got to thinking some readers may be at sea here and that the witty and brilliant Sean Carroll over at cosmic variance had surely written on some of this, so I found this amazing post of his, where he replaced Schrodinger’s poor old cat (that gets shot with a gun, or not, so it’s in “superposition,” not alive and not dead or both alive and dead) with a Cute Puppy in a Box (we just don’t want to wake it up!). For anyone who’s wondering about the Pandora’s Box of QM (quantum mechanics) you’ll enjoy this post as great background.
    http://cosmicvariance.com/2006/02/27/quantum-interrogation/

  13. Janet

    Hi, that is an elegant link. Thanks. I’m recommending it tomorrow.
    David, Hi, and others, here’s another question, just to make you work! No, seriously, I am curious what sense you get as a particle physicist as to whether a quantum particle’s wave-function is “real.” If you find Copenhagen “works beautifully,” then I think that means you accept as a working formalization that “the wave-function collapses” whenever a measurement (observation) is made.
    The wave-function is a probabilty function, right, and so the “particle” is only potentially “there” at any position, and more probably it will be found in one place than another. So why can’t we actually think of it as a potentiality-structure of some kind that covers the whole space of possible locations? Sort of like (just analogy) energy suddenly converting into mass?
    At the quantum level, couldn’t things actually “be” continuous and be converted to discreteness by a macro-sized intervention? (Please don’t laugh.)

  14. David

    I won’t laugh promise! First a small correction, I’m a condensed matter physicist, but in here quantum mechanics and quantum field theory reigns supreme just like in particle physics.

    Yes, as far as Copenhagen a macro sized intervention, which as far as we know is the act of measuring, is where a discontinuous dynamical evolution. This seems to be true. The problem some physicists have with this is that it is an arbitrary rule. The rule is stop the quantum evolution, collapse the wavefunction to what the measurement gave you, and go on with the quantum evolution. So the collapse itself is outside quantum evolution. It is like a rule imposed from the outside. This works well, and we (myself included) use the rule all the time, otherwise we could not compare with experiment and from experiments predict the probabilities of new ones, But, it bother physicists because it looks like a rule outside quantum evolution and we this take it as a sign of lack of completeness. But it seems to work.

    Yes, you could think of it as a potentiality structure.

    The analogy with mass and energy I do not understand as they are not related by any potentiality structure in physics. Mass is energy and the other way around. In fact in Einsteins genera relativity things that don’t have mass and we think as pure energy like light produce gravitational fields according to the mass energy relation and seem to have inertia according to the same relation.

  15. Janet

    I am so glad you aren’t laughing. So I’m embravened to make a fool of myself even more!
    Given the enormous energy of nuclear fission, for instance, couldn’t you regard a piece of matter as potentially all that energy? Isn’t that a valid interpretation of E=Mc^2 for a physicist? Likewise, isn’t there a way to “collapse” energy into matter in a sense? How did the the energy get locked into the nucleus in the first place?
    I think a lot about the different kinds of “possibility” that different fields talk about.
    And here’s a question that’s been waiting for you. (And anyone esle out there is welcome to join in.) I just read about a version of the two-slit experiment where instead of a screen behind the slits to catch the particles being emitted with an electron gun or whatever, there is a physicist with either a test to observe particles or a test to observe waves. The physicist does not decide which test to use until after the electron has passed through the plane of the slits.
    So this way the test — ie. measurement or observation — occurs after the passage through the slits and yet seems to determine retroactively whether the passage was like that of a wave or a particle, by the test selected. The conclusion was that in this case the measurement determines the PAST! Any thoughts about this? (No one has actually done all these EPR experiments in outer space, right? But they have done equivalents of those in empirical tests, haven’t they? Is this latter experiment an empirical test or a thought experiment?

  16. David

    What you say is not wrong per se. but you seem to have a dichotomy between matter and energy. We understand them to be the same thing but in different forms. energy can take the form of matter, heat, light and it changes between one and the other. Now, what I am describing could be taken to be just as a semantic difference from what you say. And in that sense we might be saying the same thing.

    There is no direct test to measure waves or to measure particles. the screen or a physicist counts particles. Since all the measurements we perform are by large apparatuses be it screens, physicists or something else, they are sensitive directly to particle properties. They count particles. Now if when the particles or waves, or whatever they are went through the screen there is no way to determine which slit they went through, the wave properties will be revealed indirectly regardless of how or what does the measurement. The wave properties woul be revealed even if we count particles in the sense that the number of particles as a function of position will show an interference pattern.

    Similarly if there is way to distinguish which slit the particle went through there would be no interference pattern and we would see no wave properties, not even indirectly. Feynman I believe has good examples and discussions of this.

  17. HI

    Janet,

    Are you talking about “Experimental Realization of Wheeler’s Delayed-Choice Gedanken Experiment” by Jacques et al. (Science vol. 315, 966 (2007))? (You can read a description in the following blog post.
    http://scienceblogs.com/principles/2007/02/thought_experiments_made_real_1.php)

    I agree with David that when you detect a photon, you are detecting it as a particle and the wave nature is revealed as an interference pattern.

    The experiment by Jacques et al. is interesting, but if you think that the measurement determines the past, that is because you are intuitively assuming that the photon must have behaved either as a particle or as a wave.

    I came across an essay by Phil Anderson (Nature vol 437, 625 (2005)) which is relevant to the discussion (and is fitting since Anderson is a condensed matter physicist like David).

    Anderson wrote on Einstein-Bohr debates:
    “In reading about these debates I have the sensation of being a small boy who spots not one, but two undressed emperors. Niels Bohr’s ‘complementarity principle’ — that there are two incompatible but equally correct ways of looking at things — was merely a way of using his prestige to promulgate a dubious philosophical view that would keep physicists working with the wonderful apparatus of quantum theory. Albert Einstein comes off a little better because he at least saw that what Bohr had to say was philosophically nonsense. But Einstein’s greatest mistake was that he assumed that Bohr was right — that there is no alternative to complementarity and therefore that quantum mechanics must be wrong. This was a far greater mistake, as we now know, than the cosmological constant.”

    He adds:
    “Taking London’s point of view, one immediately begins to realize that the real problem of quantum measurement is not in understanding the simple electron that is being measured, but the large and complicated apparatus used to measure it. This apparatus has all kinds of properties that are not obvious consequences of quantum mechanics: rigid slits, for instance, and a photographic plate that darkens irreversibly where an electron hits it.”

    I agree that you need a better understanding of the apparatus in order to really understand quantum measurement. But we also have to remember that there were physicists who try to explain the measurement by thermodynamical irreversible processes. But they realized that they cannot explain negative measurement (in some cases, the detector not detecting the particle is a form of measurement) by such irreversible processes.

  18. Janet

    Do I correctly understand you to be saying that the negative measurement remains a problem for now, even while we need to look closer at the apparatus?

    Roger Penrose insists that Niels Bohr’s complementarity principle (that we can/must approach a phenomenon with two incompatible models such as wave/particle, which physicists are in fact doing) is due to something missing in the present state of the theory. He believes that there must an unknown fundamental principle lying deeper than the wave manifestation and the particle manifestation that will explain both of these currently incompatible results — something we haven’t found yet, but that he believes will be found in the future. But he is very respectful of other views, like Hawking’s that it really doesn’t matter, andpatiently goes through all the various aspects (Road to Reality).
    So how can Anderson announce that the complementarity principle “as we now know, was a much greater mistake” than Einstein’s rejection of his own cosmological constant (which just recently turned out to be so presciently correct after all)? Doesn’t this seem a little extreme, compared to Penrose’s magnificently calm and detailed and honest survey of the situation of the present knowledge in QM? Has Anderson discovered something monumental, all of a sudden, do you think? You seem to be suggesting he is a bit extreme….

  19. HI

    “Do I correctly understand you to be saying that the negative measurement remains a problem for now, even while we need to look closer at the apparatus?”

    I have to give an disclaimer that I’m not an expert and much of what I know comes from reading popular books.

    Measurement often involves irreversible processes to amplify the signal. It was an appealing idea that this is related to the “collapse” of the wave function, which also appears to be irreversible. But you can consider situations where not detecting a particle is as good as making a measurement. In such a case, the particle is not interacting with the apparatus and therefore the irreversible process is not happening. Therefore, such irreversible processes are not sufficient to explain the measurement.

    I still think that you need some sort of quantum mechanical description of the apparatus to fully understand the measurement. But it also has to explain the negative measurement.

    Phil Anderson is interesting. He is a true giant. If he is not widely known, that is because there is a reductionist tendency to worship particle physicists as working on more “fundamental” problems. Anderson, as Sean Carroll puts it, “takes a certain pleasure in tweaking the noses of his friends on the high-energy/astrophysics side of the department.” It seems he likes to be provocative. (Remarkably, he even made a great contribution to particle physics by proposing an idea that lead to what is known as Higgs mechanism. Now particle physicists are trying to find the Higgs particle.) Anderson did many great works studying complicated systems, so he has earned the right to emphasize the mystery and importance of complicated macroscopic systems.

    I don’t know enough about Bohr’s philosophy, but I don’t get the impression that he was the most rigorous thinker. In fact, I think that the Bohr model is a kind of idea that a more cautious scientist would never propose. It was important because a bold radically new idea was wanted in a period of transition. But it wasn’t a model that made too much sense and it had to be replaced by more complete theories of Heisenberg and Schroedinger. Bohr was great and influential, but that doesn’t mean that we have to agree with Bohr. I don’t necessarily think that the Copenhagen interpretation represents only single philosophical position. What it offers is a simple practical rule to apply quantum mechanics that has been verified by experiments.

    I don’t really understand Anderson’s objection to complementarity principle. I can only guess and I could be completely wrong. In experimental situations that we’ve been talking about, we learned that an electron or photon can behave as a particle or as a wave. But do we really see a situation when the electron or the photon behaves both like a particle and a wave at the same time? Even in the delayed-choice experiment, isn’t it most natural to consider the photon as a wave until it is detected?

  20. Gavin

    Hi Everyone,

    I just read this series of posts and wanted to weigh in on a couple of points. I’m a recovering string theorist who now teaches high school, and I devote a great deal of my time to making cutting edge science accessible to my students. I hope that some of that effort comes through in this post.

    The mass vs. energy issue is one that I understand very well, and can teach to my students completely in two lectures, but I won’t try to do it here unless someone really wants the details. The basic idea is that almost every time you think of mass you should really be thinking about energy. For example, we usually write the formula for momentum

    p = mv,

    but the correct formula is

    p = Ev,

    where E is the energy of the thing (including its “rest mass energy”). SWith this formula it is obvious that photons can have momentum even though they don’t have mass. You can also see with this formula that as you have a particle going near the speed of light, so you can’t increase its velocity, giving it more momentum (by applying force, which always changes momentum) will still increase its energy.

    (I’m using units c=1, which means distance and time are both measured in seconds and energy, momentum and mass are all measured in kilograms.)

    Similarly, mass is not what causes gravitational fields, energy does. In Einstein’s equations, the curvature is related to the stress-energy-momentum tensor, not to mass.

    It is good that energy is used in all of these areas because energy is a conserved quantity (warning: GR makes this a dangerous statement) so we can track energy much better than mass.

    The only thing mass is good for is relating energy to momentum for individual objects via

    m^2 = E^2 – p^2,

    which gives E=p for massless objects (like photons) and E = mc^2 + 1/2 mv^2 for objects going much slower than the speed of light. (Deriving that last equation requires expanding in powers of p.) Mass is not conserved in interactions. You can’t say anything very useful about the total mass of interesting systems. Its good for one thing and that’s it.

    Finally, Janet says:

    “He (and the other physicists of his generation) were raised and trained in the Newtonian world of absolute time and space, where every event was laid out on one universal grid, as it were, and they knew that had ended.”

    I disagree. Before special relativity everything was laid out on a three dimensional grid and assigned a time. The three dimensional grid was not absolute at all. It could be rotated in three dimensions, and it could be given a constant velocity. (Technical note: That is three rotational degrees of freedom and thee velocity degrees of freedom for a total of six way that space was not absolute.) Space was not absolute before special relativity, but time was.

    Special relativity replaced the three dimensional grid and absolute time with a four dimensional grid. This grid could be rotated in four dimensions (Technical note: In four dimensions there are six rotational degrees of freedom, exactly the same as before relativity.) These four dimensional rotations include the three dimensional rotations as well as boosts, which give a constant velocity to the coordinates but also mix time with the spacial coordinate in the direction of that velocity.

    Relativity removed the idea of absolute time, but it did not get rid of the idea of absolute space because there was no such idea and it didn’t add any new freedom for different observers to pick their own coordinate systems, because all of that freedom was already there.

    Quantum mechanics is much more difficult. I’ll work in it is a separate comment.

  21. Janet

    Okay, this is great! Thanks so very much, Gavin. Can you put those two lectures on a link for us, maybe?

    Your description of the Newtonian “grid” certainly makes it rotatable and non-“absolute.” This helps me understand why Newton himself and other Newtonians made a distinction between “absolute” space and regular space, which has always puzzled me. But Newton did say in Pincipia that “absolute time and absolute space flow eternally from the throne of God” (close paraphrase from memory).

    Every physics book points out that Gallileo knew about relativity in a simple sense. Your comments help me start to see why Newton thought it necessary to distinguish his absolute space from the regular space physicists actually worked with (in his day they were still “natural philosophers” and the word “scientist” wasn’t even used yet).

    But whatever content the notion of absolute space had in Newton’s mind (?) it was culturally this absolute time and space that went into the Modern West’s Newtonian worldview of the clockwork universe and how human minds could acheive a god’s eye view of the way things “really are” in the shape of finished, absolutized “facts” about an eternal steady-state unchanging universe. With an “absolute” determinism, as far as physical bodies go.

    Hence, the human mind becomes detached from nature and endowed with this absolute autonomy and godlike freedom of the will. (Here we go again. The Cartesian paradigm that we cultural theorists are so adamant about decrying. It would be okay if it were only historical, but it is alive and well in the many/most minds of North Americans today and guides their politics and voting…

    Does this make any sense? (Scientists look at the actual working physical theories while cultural theorists are tracing the cultural mindset that accompanies the theories, and the two influence one another but are definitely not the same thing formally speaking, or in terms of the details.)

    What do you think? (The speed of light being absolute with respect to every observer isn’t much of a change? I’m teasing, but this does mean something quite significant, doesn’t it? How to characterize THIS?)

  22. Janet

    “Absolute time and absolute space flow eternally AND CHANGELESSLY from the throne of God”! — Newton in Principia.

  23. Rick

    “Hence, the human mind becomes detached from nature and endowed with this absolute autonomy and godlike freedom of the will.”

    In what seems to be a growing stream of Catholic moral thought, following Servais Pinckaers and his “Sources of Christian Ethics”, they too decry what they see as a claim of “absolute autonomy and godlike freedom of the will” (Pinckaers terms it “freedom of indifference”) that they see in modern (or enlightment) thought. But they blame in on the nominalism of William of Occam, not Descartes.

  24. Janet

    Yes, yes, yes. Thanks for bringing nominalism back into my mind. And for the hint to see Pinckaers (whom I’ve never even heard of! good grief!). I’d forgotten the nominalists, and how they were often seen as the origins of bad (autonomous) humanism (as opposed to Christian humanism, which certainly was a humanism integrated with theism and one that has a long and marvelously rich history including Shakespeare and Milton).

    If you’ve seen my Kevin Hart post, you’ll know too — or you already know — that Hans Urs von Balthasar (may his name be blessed — what a sweet man!) traced the break (away from the traditionally Christian finite and situated and dialectical human mind, to the Cartesian “absolute mind”) in Duns Scotus! And Duns Scotus was indeed the basis for the theology of the Counter-Reformation theologian Suarez, whose work dominated the Jesuit school in La Fleche where Descartes attended!

    I’m fascinated by this. If you read medieval theology on God and mind and knowing and then you read Descartes, he is coming from a different planet! The degree of originality and differentness with Descartes is just astronomical. Especially this sudden hunger for “absolute certainty”….

    PS I want everyone to know that Gavin has a new comment over in Section # 4….

  25. Gavin

    Janet,

    I’d like to connect three comments that you make. I think this post is going to be somewhat blunt, possibly even rude. However, I think that there is a commitment to understanding here, so I’d rather be clear than polite. I’m not a gifted enough writer to do both, so I apologize in advance.

    David was critical of your original post where you said,

    “Einstein introduced the human observer into physics in a much deeper sense than that; he showed that what we know through physics is always-already what we can know according to our attempts to make measurements, and he realized that this cut the link between genuine human knowing and any claims to an all-inclusive or universal knowing.”

    His criticism was right on the money. Einstein did not introduce, show, or realize anything like that.

    You then say,

    “I’ve now realized that scientists read any mention of enhanced epistemic sophictication in science as an attack on science! They leap to the conclusion that what we really mean is that science is socially constructed and it’s results are illusory. No! I don’t know any postmodern thinker who holds this position! The epistemic qualifications worked out in Relativity and QM makes science even more wonderful and more honest and more legitimate as a way of knowing!”

    I cannot say what scientists in general read, but I can say how I respond to statements like the one you made about Einstein. I don’t see an attack on science, I see an attempt to hijack the credibility of science. I think your comment to my last post clarifies how this is done:

    “Scientists look at the actual working physical theories while cultural theorists are tracing the cultural mindset that accompanies the theories, and the two influence one another but are definitely not the same thing formally speaking, or in terms of the details.)”

    I agree with this, but it is often brushed aside, in part because few people appreciate the huge chasm between advances in science and the cultural responses. To a scientist, the cultural response appears to be totally independent of the actual advance, and almost totally dependent on the vocabulary and analogies chosen in articulating those advances to the public.

    A scientist might say, “Einstein showed that a head-on, inelastic collision between identical objects going .6c and .8c will result in something going .2c, not .1c as Newton thought,” when in fact he means that using the theories articulated by those people gives the claimed result. Of course we have no idea whether Einstein or Newton ever did that exact calculation, but their theories are totally clear about how to do the calculation, so to say “Einstein said this while Newton said that,” is pretty precise.

    However, when discussing the cultural ramifications, it is common to see comments like “Einstein put perspective squarely in the equation,” when in fact this means that after someone with a basic understanding of Einstein’s theory tried to put it in non-mathematical, metaphorical terms, that paraphrase was seized upon by someone who didn’t understand the theory at all, but was inspired to make a claim analogous to the metaphor in some unrelated field. The statment that Einstein demonstrated this claim is absurd.

    I can certainly understand the temptation. My wife might not be persuaded if I say “I’d like to get an iPhone,” but how could she argue if I say, “Einstein demonstrated that it is always-already known that I must get an iPhone”? If “since 9/11” is the automatic get-out-of-jail-free card for sloppy logic in political circles, “Einstein showed” and “quantum mechanics teaches us” play the same role is certain areas of academia, as well as in new age thought. They are hugely effective in this capacity because the theories carry unassailable credibility while being understood by almost no one. If you base your claim on Einstein or quantum mechanics, who can argue with you? Only scientists, who can be dismissed as arrogant.

    When people claim that science says things that it does not say, that is an attack on science because it undermines science’s credibility. That frustrates me.

    I’d like to write some more posts like the one I wrote about special relativity. My goal is to express exactly what these theories show, so we won’t be fooled when people say that relativity and quantum mechanics say things that they don’t. In the completed post I tried to show that special relativity changed the way physicists think about time and mass. That is a big deal in a physics sense, but it really is nothing philosophically. It doesn’t introduce relativism or perspective or cut any sort of epistemological link, even if people who heard rumors about this theory were inspired to do those things.

    To address your question, “The speed of light being absolute with respect to every observer isn’t much of a change?” No, it’s not much of a change. It is a rather mundane consequence of the four dimensional rotations. It doesn’t have any effect on perception, determinism or the “absoluteness” of how we view the world. It may have caused non-physicists to change their views on those things, but that is not a consequence of the actual physics.

    I’d like to address quantum mechanics next. There are three things in quantum mechanics that people use to justify all sorts of non-physics claims: the Heisenberg uncertainty principle, wave particle duality, and the issues of quantum measurement. The uncertainly principle can be seen in waves on a beach, it is not a fundamental new feature of quantum mechanics. Wave particle duality as an obvious consequence of the quantization hypothesis, which is not profound. Only the measurement issue raises big questions that need to be addressed. I think significant progress is being made in addressing those questions, largely by people studying quantum computing, where one cannot engage in sloppy thinking about quantum mechanics.

  26. Janet

    I really appreciate your honesty, Gavin. Without it, I don’t see how we can make any real headway in cross-disciplinary understanding. I don’t think you were rude at all. Honesty is the best gift we can give each other.

    I would like to know what some of the rest of you might say about what Gavin just said. Do you all agree with him?

    Also, Gavin’s clarification (I won’t call it a rebuke, though it is one! — and that’s okay) elicits these follow-up questions from me.

    1) Is there simply NO justification at all for seeing Relativity and QM as constituting a large paradigm shift in science, and one that might be tied in with other shifts in other fields and related to general cultural changes in any rigorous or exact manner? (If there is no strictly scientific justification, does that necessarily mean there cannot be any justification for such judgments coming from other fields?)

    2) What about the shift from a steady-state and highly deterministic universe to a probablistic and evolving universe, for example? Is such a generalization as the previous sentence in some sense extra-scientific? Outside of science? Illegitimate, scientifically speaking.

    3) Should we say that no changes or discoveries in science are ever PROPERLY culturally significant, but have only scientific significance? If they do seem to have a cultural effect, is that always only by being popularly misunderstood or hijacked by sloppy thinkers? (Think of the profoundly darkening effect of entropy and the heat death of the universe on the later nineteenth century, for example.)

    4) When we go to the scientists to tell us what interpretations we should NOT make, to avoid the trap of misinterpretations on the part of non-specialists, what do we do when the top physicists do not agree?

    In a way, Gavin brings us back to a basic fact. We thhink diffeently in different disciplines. My theoretical physicist colleague and friend Jim Crichton warned me, saying “Janet, always remember that for physicists, it is the math and the experiemental results that are the science. Any talk of philosophical interpretation or the larger nature of reality is for us just cocktail party chatter. It is mildly interesting to us, but nothing very serious, because it can’t be scientifically grounded the way the math and the experimental finding themselves can be.”

    (I have to add that no one kept up with philosophy more than he did and his passing from cancer deprived our campus of a liberal arts stalwart. He also wore a T-shirt that said: “A theoretical physicist is simply the universe’s attempt to know itself….”)

    In any case, I welcome the forthcoming explanations of QM that you offer, Gavin, and I will think a lot about the explanations you have already
    offered.

    P. S. I’m just not wholly convinced yet. We need a historian of science here, but my sense about Einstein, Bohrs, Heisenberg, Bronowski and everyone else during those Copenhagen decades was that they all had a significant sense of the ground shifting under science.

    Maybe I am not saying it right about Einstein, folks, but I think there is something there, and it can’t be just that we have a whole passel of historians, philosophers, cultural theorists, and New Agers maliciously hijacking science and ruining its credibility with sloppy talk. (I’m not denying that this hijacking and sloppiness does happen; I’m a poststructuralist and we are totally misunderstood and slandered all the time!)

    After all, remember, Gavin, it doesn’t affect science’s credibility as science, to us, when we want to talk about additional metatheoretical and cultural and historical issues with regard to science…. But to you this diminishes the true credibility of science, which you are committed to defend and explain. I get that. Good for you.

  27. Gavin

    I don’t plan to make rants a regular component of my comments, but before I get back to what I actually know, I’ll just say that certainly quantum mechanics represented a major shift in the foundations of science, due entirely to the measurement issue. Special relativity and general relativity were bold new theories, but didn’t represent new foundations. They are totally deterministic, clockwork universe type theories (except GR has singularities, which we aren’t quite sure what to do with).

    I don’t have a digital form of any of my lectures that I can link to. I’m working on that, but it will be several months before I have much useful.

    The Heisenberg Uncertainty principle is what I really want to talk about. Imagine you are lying by the sea listening to the waves break on the shore. When a boat goes by its wake makes waves which you can also hear breaking with a different rhythm. Can you accurately describe both the rhythm of the boats wake (its frequency) and the time that it broke on the shore. Yes, but there is a limit. If you want to measure the frequency accurately, you need to hear at least a couple of waves break. The more you hear, the more accurately you can describe the frequency. However, if there are a lot of waves breaking then the arrival of the wake is taking a long time. It becomes less clear what time you should use to describe the arrival of the wave. As the rhythm get longer you can describe the frequency more accurately and but the time less accurately. If the rhythm get short then you can describe the time accurately, but not the frequency. All of this can be put on rigorous mathematical footing and you get an uncertainty relation

    df dt >= 1/(2 pi),

    where df is the uncertainty in the frequency, dt is the uncertainty in the position and >= is a “greater than or equal to” sign. (Notice that the units work out nicely.)

    Would you be able to do any better if you went out and looked at the waves rather than listening to them? Now you can measure the wave length (or its reciprocal, the wave number, n) and the location of the wake. However, if you want a good value for the wave number, you need lots of waves which will give you an uncertain position, but if you have a tight position, then there aren’t many waves and the wave number is uncertain. So again you have an uncertainly relation

    dn dx >= 1/(2 pi),

    where dn is the uncertainly in the wave number and dx is the uncertainty in the position.

    The conclusion is that wave give uncertainty relations. Uncertainty is not a consequence of quantum mechanics. Does that mean that waves breaking on the shore demonstrate the in some way ultimate truth is not only unattainable but actually nonexistent? Opps, that’s not my field.

    To make the uncertainty relations above look like Heisenberg’s, we need the quantization hypothesis. This states that the energy in something with frequency f is quantized with energy levels spaced E=hf apart. Similarly, the momentum in something with wave number n is quantized in momenta of steps p=hn. Multiplying the relations above by h and substituting the quantization hypothesis formulas gives the usual relations,

    dE dt >= h/(2 pi),
    dp dx >= h/(2 pi).

    Now that we have these nice relations it is reasonable to ask why they apply to something like an electron, which doesn’t appear to be a wave. It applies because the electron is a wave. It is actually a quantum excitation of a wave. The wave obeys the first set of uncertainty relations above. The wave can only be excited in amounts given by the quantization hypothesis, so these excitations (which we also call particles) obey the second set of uncertainty relations.

    It may be a few days before I can post again. I’m leaving town tomorrow, but I may get something out in the morning.

  28. Janet

    Okay, great.

    One response though.

    You say: “Does that mean that waves breaking on the shore demonstrate the in some way ultimate truth is not only unattainable but actually nonexistent? Opps, that’s not my field.”

    Now see, this provokes in me the same sense of the credibility of poststructuralism being hijacked by sloppy thinking that you complained of in my statements about Einstein and Qm.

    I mean, it really makes me want to go on the warpath!! (Calm down, girl.)

    To my ears, when people tell me that my own field’s rigorous theory means that “absolute truth is unknowable or does not exist,” they are saying that the standard for truth is (of course) “absolute” truth and any other more qualified definition (or understanding) of truth is merely subjective, personal, arbitrary, or relativist twaddle…

    Actually, we scientists and theorists mirror each other more than I had realized! But I want to say that genuine, high-level postmodern thought does not amount to relativism or subjectivity or the loss of truth — and that when people imply this they are unknowingly thinking very sloppily and not really grasping the theoretical details at all. So I feel EXACTLY like you scientists generally do about the credibility of my discipline being “abused” (to use Sokal’s term about PoMo’s “abuse” of science).

    Sometimes it’s hard to read the tone, so please realize that the above remarks I just made are not meant to be snippy or acusatory. I’m being honest about how that comment of yours strikes me, even though you say it in passing, because everyone says it over and over again. From within my discipline it seems ludicrous that people interpret our work this way. So I guess I have to empathize with y’all more.

  29. Gavin

    Janet,

    Thank you for responding to my passing remark. I’m sure you hear remarks like mine all the time. They are condescending and ignorant. They show no understanding of what your field actually entails and simply treat the vocabulary as the ingredients of a word salad that can be thrown together in any roughly grammatical way to produce pseudo-profound mumbo-jumbo. I’m sure it is grating.

    People who study relativity and quantum mechanics (as well as other areas of science, like evolution) experience something very similar. The difference is that the tone is generally respectful, even admiring. However, the ignorance and sloppiness is the same. Many people think they can engage in a conversation about what some theory shows even though they don’t know what the theory is. The language of all of these theories is math. Anyone who doesn’t know at least the basic math required to formulate the theory is like the person who learned literary theory from cartoons in The New Yorker. I don’t know what “always-already known” means, but I can pretend to know and stick it into philosophical sounding sentences. Most people don’t know what the uncertainty principle actually is, but they can pretend and stick it into sciency sounding sentences.

    The good news is that the math of special relativity is quite basic, just algebra. There really isn’t anything preventing anybody from learning the details of the theory. I think everybody should learn special relativity; it is a dream of mine. General relativity is much harder, because it requires some differential geometry, but the committed non-scientist could get a pretty good understanding. Quantum mechanics can be pretty well understood with matrix multiplication, which is a lot easier than calculus. However, with quantum mechanics knowing the math doesn’t seem to be enough to understand what is going on. People who have mastered the theory are still prone to talking gibberish about what it means. When I figure out why, I’ll write a book.

  30. Gavin

    Alright then, wave-particle duality is next. After that maybe David and I can argue about the interpretation of measurement in quantum mechanics using actual equations. How about it, David, should we show them how it’s done? I’ll take decoherence if you want to do Copenhagen, but I could go either way. I won’t do sum over histories, though; I don’t get it.

    In my last physics focused post I mentioned the quantization hypothesis. This hypothesis preceded the complete quantum theory, which has quantization as one of its consequences. Although it would be best to describe the full theory, it is quite amazing how much can be done using just the quantization hypothesis. In particular, the essence of wave-particle duality can be understood from this hypothesis.

    The hypothesis states that anything that vibrates with frequency f has discreet energy levels with spacing E=hf, where h is Planck’s constant (which in usual units is very, very small.) Consider a tuning fork with a frequency of 440Hz. If you hit it hard it will have lots of energy in its vibration, and it will gradually lose that energy and get quieter. The quantization hypothesis tells us that the loss of energy happens in steps of size E=hf. The energy will step down and step down until the it reaches the lowest possible energy, the ground state. (This ground state only occurs if the tunning fork is very cold, but that issue is unimportant here.) If you try the same trick with a higher frequency tunning fork, then the steps will be bigger (E=hf gives bigger E for bigger f). If the frequency is lower, then the steps are smaller. For all tuning forks, however, the steps are too small to be noticed. It is only with very high frequency vibrations the steps become big enough to be noticeable, for example, in the internal vibrations of molecules. These little steps of energy are called quanta, and they give quantum theory its name.

    Now we can consider something more complicated than tuning forks. Let’s consider the electromagnetic field. This field can vibrate also, but the vibrations move, they are electromagnetic waves. These waves come in any frequency, different frequencies being associated with different wavelengths. ( lf=c, where l is wavelength and c is the speed of light.) Once again we can apply the quantization hypothesis, E=hf, and find that each frequency has steps in its energy, these steps are still called quanta, but they are also called particles. When you put one quanta of energy into a particular frequency oscillation of the electromagnetic field, that is a particle called the photon. Often people say that sometimes a photon behaves like a wave, and sometimes like a particle, actually a particle is a quantum excitation of a wave. It’s not either/or, it’s both. That is wave particle duality.

    All of the other particles come about the same way. Electrons are the quanta of energy in the various frequencies of the Dirac field. This field is strange because it will only allow one quanta in each frequency (like a tuning fork that could only hold one quanta of energy). Other particles are associated with other fields. This is what quantum field theory is all about.

    There is one especially cool trick with this that David will know about. In a material there are vibrations that we call sound. These vibrations have frequencies, so they obey the quantization hypothesis, giving rise to sound particles called phonons. Phonons are actually particles that have been extensively studied. They only exist in a material, because sound cannot travel in a vacuum. They play an important role in many superconductors and in heat capacity and conduction, but I like them just because I think its neat that sound is a particle too.

    Now, there are some complicated issues. In particular, there are localized jiggles in the electromagnetic field that don’t have a specific frequency, so they give rise to particles which don’t have a specific energy. These excitations are superpositions of excitations that do have specific energy, so when you produce one type you may measure the other type. But this is a measurement issue, which, as I’ve stated before, is where the weirdness is in quantum mechanics.

  31. “Often people say that sometimes a photon behaves like a wave, and sometimes like a particle, actually a particle is a quantum excitation of a wave. It’s not either/or, it’s both. That is wave particle duality.”

    …a particle is a quantum excitation of a wave…. This sounds nice, but I don’t know what it means. I get the quantum part, but “excitation of a wave”? And what happened to the collapse of the wave function? (Is this the measurement problem you refer to, or do you mean a different special case where the measuring device interferes with the wave?)

    When the photon (or electron or other particle) hits a screen, it “pings like a pebble hitting the side of a car,” as one article put it. In other words, it always lands like a “solid” particle (even though it is the excitation of a wave). But in the two-slit experiments, it’s where the particle lands that’s the issue? With one slit closed it hits probabilistically somewhere on a trajectory through the open slit, but if both slits are open it lands probabilistically somewhere consistent with the interference fringe pattern of a water wave parting to go around a rock and then coming back together to make an interference pattern.

    So if both slits are open, you observe an different probability and location distribution, from if only one slit is open. So are there two different wave functions, depending on one slit or two?

    And so far, is there no weirdness in your view? Or is that the weirdness, just above? Can you tell it like a story with a gun emitting a (what?) excitation of a wave at a certain quanta level and then….
    (If I get this clear, then we can discuss the beam splitter experiment with the retroactive effect.)

  32. “Phonons are actually particles that have been extensively studied. They only exist in a material, because sound cannot travel in a vacuum. They play an important role in many superconductors and in heat capacity and conduction, but I like them just because I think its neat that sound is a particle too.”

    Yes, I think this is cool too. (I take it that this kind of wave, moving through a material, doesn’t exhibit the paradoxical behavior of the wave/particles that move at the speed of light?)

  33. Gavin

    Janet,

    Good questions. I’ll answer your second question first. Phonons travel at the speed of sound, rather than at the speed of light, but aside from that they have all of the same sorts of features at photons. For example, they can be detected with a counter (like pebbles hitting the side of a car) and they can produce interference patterns. All of the crazy experiments that are discussed with light could be done with phonons (double slit, delayed choice, quantum erasure, etc.). Also, the electromagnetic force can be described in terms of virtual photon exchange. Virtual phonon exchange also describes a force. This force is attractive for electrons in certain materials. This attraction plays a crucial role in superconductors.

  34. Gavin

    Janet,

    The photon is a quantum excitation of an electromagnetic wave. This wave has nothing to do with the wave function. The electromagnetic wave is similar to waves that you are used to, but the wave function is a very strange beast. I haven’t talked about the wave function at all, and also haven’t talked about its collapse.

    When you try to measure the position of a photon, you are doing something that is rather contrary to the nature of the photon as I have described it. Waves don’t have well defined position, they are spread out all over the place. The photon, which is an excitation of this wave, is also spread out, so measuring its position leads to all of the measurement issues that make quantum mechanics weird.

    But don’t worry about that yet. First be sure you understand the uncertainty relation and the quantization hypothesis. When I get back from my trip (Thuesday or Wednesday) then I can attack measurement.

  35. David

    Janet,

    As Gavin said, phonons exhibit the same wave particle duality as any other quantum wave. Moving at the speed of light or at the speed of sound or any other speed has nothing to do with the apparently paradoxical wave particle quantum duality. In fact one could do almost all of the quantum mechanics and quantum filed theory of materials in a world where relativity wasn’t true and it would come out the same.

    This was recognized early on because the shortly after Einstein did his work on the quantization of photons and the photoelectric effect, he applied similar reasoning an methods to the quantization of phonons to explain why specific heat of solids (how the solid absorbs heat basically) goes to zero with temperature. All was done pre 1910. If phonons were not quantized the solid would absorb heat at a constant rate at all temperature. If they are quantized when the temperature became small enough that it didn’t have enough energy to excite a sound quantum, the solid couldn’t absorb heat and it’s specific heat would go to zero.

    The advice is to not mix things. Wave particle duality is about quantum physics and does not have to do with relativity or the speed of light.

    This does not mean that interesting things do not happen when we mix relativity with quantum mechanics. They do happen, but that is a separate thing.

    Gavin, I will be more than happy to run a discussion on measurement. I’ll wait for you to get back from your trip and fire the first shot.

  36. David

    Janet, just like you take phonons to be kind of waves moving through a material, you could take photons to be kind of waves moving through the universe. ;-)

    They are the same.

  37. HI

    I missed the dialogue between Gavin and Janet the last few days, but it’s great. Gavin, who has better understanding of physics than I do, expressed more precisely and eloquently the frustration that I also share.

    I started reading David Lindley’s “Uncertainty” as it was mentioned in Chad Orzel’s blog and the discussion here made me curious about the way Bohr and others thought.
    http://scienceblogs.com/principles/2007/06/uncertainty_by_david_lindley.php#more

    I haven’t even got to the QM part of the book yet, but I already encountered a sentence that express my feeling in a concise way.
    “Relativity, to be sure, allowed for differing perspectives, but the whole point of his theory was that it allowed apparent contradictory observations to be reconciled in a way that all observers accept.”

    Let me try to respond to Janet’s questions.

    Question (1) Is there simply NO justification at all for seeing Relativity and QM as constituting a large paradigm shift in science, and one that might be tied in with other shifts in other fields and related to general cultural changes in any rigorous or exact manner? (If there is no strictly scientific justification, does that necessarily mean there cannot be any justification for such judgments coming from other fields?)

    My response to Question (1):
    This is a difficult question to answer, because I’m not a historian of science or a philosopher of science. There were a lot of big paradigm shifts in 20th century science. I don’t know whether they are related enough to be part of a large paradigm shift.

    Take a look at the field of molecular biology as an example. Many of the scientists involved in founding this new field were physicists. Many were influenced by Schroedinger’s book “What is Life?” In the case of Max Delbrueck, Bohr’s speculation that complementarity argument might have applications in biology had a big impact on him.

    Delbrueck did end up doing great works in molecular biology, but equivalent of complementarity was not found in biology. On the other hand, it almost seems that the success of molecular biology owes a lot to the arrogant confidence of physicists who believed that they can understand life by applying their approach. (And being a molecular biologist, I am part of this.)

    Question (2) What about the shift from a steady-state and highly deterministic universe to a probablistic and evolving universe, for example? Is such a generalization as the previous sentence in some sense extra-scientific? Outside of science? Illegitimate, scientifically speaking.

    My response to Question (2):
    Being steady-state and deterministic are two different things. Many things are deterministic, but are dynamic and not steady-state.

    Also, even in a deterministic universe, you will be able to predict the future only if you have the complete information of the present. But in most situations, you have a lot of unknowns. Even before QM, people have dealt with the unknown by introducing probability. That dates back at least to Cardano in the 16th century. Boltzmann’s work on statistical mechanics was before QM. Also, you shouldn’t forget that one of the three great discoveries that Einstein made in 1905 was about Brownian motion.

    In many disciplines, scientists have always been interested in dynamic systems. (Think about evolution, for instance). And it is rare that you have enough information to have a deterministic picture. In that sense, classical physics might have been an exception.

    Question (3) Should we say that no changes or discoveries in science are ever PROPERLY culturally significant, but have only scientific significance? If they do seem to have a cultural effect, is that always only by being popularly misunderstood or hijacked by sloppy thinkers? (Think of the profoundly darkening effect of entropy and the heat death of the universe on the later nineteenth century, for example.)

    My response to Question (3):
    I think you need a distinction between when the cultural impact of science is direct or only metaphorical. For example, the knowledge that we have an ability to make weapons with enormous power of destruction, or the knowledge that our economic activities result in global warming should have direct impact on the way we think about our society and policy decisions. On the other hand, the influence of QM on the way we think about social phenomena are only metaphorical. To quote David Lindley from Uncertainty, “We already know that people act awkwardly in front of cameras, that they don’t tell their stories to a newspaper reporter the same way they would tell them to a friend.” We don’t need to refer to QM to explain such things.

    I don’t think it is necessarily bad that people in non-scientific disciplines get inspirations from scientific discoveries. But you should be always aware how strong or weak the connection is and be clear about it when you make a statement. I think the problem is when they make stronger and more general claims than warranted.

    Question 4) When we go to the scientists to tell us what interpretations we should NOT make, to avoid the trap of misinterpretations on the part of non-specialists, what do we do when the top physicists do not agree?

    My response to Question (4):
    Are you talking about interpretations of physical phenomena themselves, or their implications to philosophy and other non-scientific disciplines?

    If it is about physical phenomena themselves, it is best to leave it to the specialists.

    If it is about implications to non-scientific disciplines, you might have as much right to claim as scientists, but you should get the science part right and should be aware of the things I mentioned in my response to Question (3).

  38. Rick

    “On the other hand, it almost seems that the success of molecular biology owes a lot to the arrogant confidence of physicists who believed that they can understand life by applying their approach.”

    I’m pretty sure that almost every physicist knows, deep down in his heart of hearts, that 1) chemistry is a branch of physics, and 2) biology is a branch of chemistry, which, in turn, makes it a branch of physics. Physics is the foundation from which which all other physical sciences arise.

  39. Okay, I am lying prostrate in the dust. But I will rise again. There are anamalies that haven’t been addressed (at least as yet) in what y’all are saying, and we need the rest of what you have to say about QM.

    Give me some time to gather my thoughts. Meanwhile, I’m publishing the next installment of “Wily Socrates,” because it is ready to go.

    Oh — about the physics-chemistry-biology high-rise that Rock reaffirms and with which I have no quarrel (that was my understanding too), it is very important (from a theory viewpoint) that we still have to study chemical and biological organizations on their own level even when we “know” that they reduce to physics. This is the only way that non-scientific fileds such as the arts and sciences can be credited as rigorous.

    Many scientists (not y’all) seem to think that these fields deal with “mere experience” or “subjective” impressions as opposed to the “objective truth and universality” of science. (In semiotic theory, in human meaning-systems, things take their identity from what they are contrasted against!)

    Here’s David Lindley’s Uncertainty:

    “What fascinates, evidently, is the semblance of a connection, and underlying commonality, between scientific and other forms of knowledge. we return, in this roundabout way, to D. H. Lawrence’s jibe about relativity and quantum theory– that he liked them precisely because they apparently blunted the hard edge of scientific objectivity and truth. We don’t have to be as intellectually philistine as Lawrence to see the attraction here. Perhaps the scientific way of knowing, in the post-Heisenberg world, is not as forbidding as it once seemed.”

    To which I reply, “Ya think????”

  40. Could you go over and look at this summary of Bronowski’s episode “Knowledge or Certainty”?
    http://www.wsu.edu/~brians/hum_303/bronowski.html

    It’s better to read the whole chapter in the book, The Ascent of Man,” or watch the beautiful episode in the television series.

    Please pay especial attention to the point that Browoski believed in the objective knowledge and validity of physics. (He was a physicist who left physics for biology after the disillusionment of the atomic bomb’s use TWICE on Japan, instead of the demonstration of its use his own mentor and others recommended to the president. )

    He wants to call QM’s uncertainty principle the principle of tolerance (in the engineering sense) and he has a detailed understanding of QM. Can you see why he welcomes a certain shift in physics paradigms?

  41. David

    While I think that relativity to some extent and to a lot bigger extent quantum physics were paradigm shifts, the notion in the link Janet provided predares this shifts. For centuries scientists have understood that knowledge is not complete or absolute. If ti were, you stop doing science as knowledge is complete.

  42. HI

    Well, I’m sure Rick made the comment with tongue in cheek, but that’s something I as a biologist, and a condensed matter physicist like Phil Anderson take an issue with.

    Quoting from Phil Anderson’s essay titled “More Is Different”:
    “But this hierarchy does not imply that science X is “just applied Y.” At each stage entirely new laws, concepts, and generalizations are necessary, requiring inspiration and creativity to just as great a degree as in the previous one. Psychology is not applied biology, nor is biology applied chemistry.”

  43. Hurray for Phil Anderson! To my mind, he is saying exactly what I was trying to say above in my comment at 8:58 this a.m. Hurray, it’s happened, and Hi and I AGREE!!!

    David says: “For centuries scientists have understood that knowledge is not complete or absolute. If it were, you stop doing science as knowledge is complete.”

    David, and everyone else, look at this sentence of David’s because it is crucial. Does it mean scientists have understood that “knowledge is not complete or absolute” or that “it is not complete and absolute AS YET!?”

    In other words, what does “knowledge” mean? Does “knowledge” mean that which ends by being absolute and universal “fact,” so there are no loose ends and everything is explained and our representations of reality are PERFECT fits with reality? Because I contend that this is the concept of fact that arose in the 17th-19th centuries in the West and made us so blind and arrogant and led to the non-scientific disciplines being viewed as subjective and arbitrary because they were not able to arrive at scientific fact in this sense, even though science in fact wasn’t (yet) able to either, and still isn’t…

    I know that SOME scientists today believe that physics will arrive at “a complete description of fundamental physical reality” but many others doubt this very much. See the “fundamentalist” position as opposed to the “secular” and the “mystical” positions outlined in the essay by physicists on mind, math, and matter (which I’ll look up and give the link again in a second).

    But believe me folks, I could even live with that — a complete and final description FOR PHYSICAL (i.e. physics) REALITY if only it were not implied that this means ALL OF REALITY. But clearly, for many scientists and for the scientistic science bloggers on many sites, it does mean precisely this. It leaves no room for any “reality” on other levels of structure and other realms of human life, so Hi’s comments are very very important.

    Everything I say, you see, is in reaction against the inflated claims that “science knows everything and what it doesn’t know isn’t real.” Just read the science blogs for this! And you may say that scientists always knew their knowledge wasn’t complete, but did they not think it was “absolute”? Think of La Place. Think of Newton’s Principia. Think of the scientific rationalists who told me animals couldn’t think or feel and neither could babies because they are just machines. It’s the absolutism that enters in here (and that gets carried over into the common mind) that strikes many of us as so illiberal and dangerous.

    If we can be more precise about what science knows and how “absolute” its knowledge is or isn’t, we might be able to introduce the color “gray” into some of these black-and-white minds that are running around here. It is not merely some religious people who are fundamentalists in the sense of an absolutely closed worldview with no openness to genuinely Other ways of knowing. The critique of Modernity has been carefully qualifying and mapping out any limits it can find to militate against that absolutism.

    It is not the scientific “theories” themselves that scare me but the way many people HOLD those findings, as an absolute authority that should ride rough-shod over every field and over human beings’ deepest values and traditions and overrule any other kind of thought or discovery. This is why non-religious people are coming into the discussion of Dawkins’ The God Delusion, for instance, to point out Dawkins’ scientistic fundamentalism and to defend other ways of knowing (ways that leave Dawkins bewildered by his own admission — why would anyone want to know about that, he says). Dawkins wants to use a very narrow and very hard-edged and reductive and uninformed scientific rationalism to do away with some of the most precious intellectual and esthetic possessions of many people, including many very thoughtful and brilliant people.

    There are huge political and social pressures operating here, so that I know that in their guts many scientists want to sympathize and agree with Dawkins in branding religious belief as the root of all evil, but shouldn’t we nonetheless insist on careful discriminations here and not letting any field or any authority become an absolute authhority, including science? Again, to me it is not science or religion which is the problem, but holding either without knowledge and respect for 9at least the possibility of) other ways of knowing in their own proper spheres.

    This is why I think what Jacob Bronowski tries to say is so important. And the role of the observer in Relativity and the role of measurement in QM. Because prior to that, it was too easy for Westerners to think that the laws and formulas equalled reality, or were a perfect one-to-one fit. We forgot that our efforts to come to know were involved at all. The mediators of the theory drop out of sight too easily.

    It’s much more likely to me that all rigorous descriptions (in all fields) fail to get in all the reality even of their own little aspect of reality. But what I just said is a “belief.” And so is the “belief” that science will arrive at a complete description of physical reality, and that this would be a complete description of everything real and that is would not be modified by anything, even advances in unrelated fields. This are belief-structures. These philosophical commitments are not scientifically verified truths of any kind. Science student need to be drilled and drilled in what science is and at what point our informed belief-structures come into the picture! For every one comment pointing this out, there are 50-100 comments reflecting scientistic belief-structures on the science blogs.

    In the same way theists need to be standing up everywhere across the globe and saying “No!” to religious extremism of every kind. No, this is not our faith. This makes our faith into a closed and sealed-off and uncritical (and unloving and unspiritual) authoritariansim that is at odds with everything our fiath stands for. (Okay, I’ll stop preaching for now.)

  44. David

    I could mean as yet. But whether it will always be incomplete or not it’s impossible. Also if one were to think it is complete, there is no way to know. So whether one believes there could be a point where it can be complete, I do not find too relevant. It’s impossible to know.

    Even if something fits everything perfectly now, that does not imply we will not explore a regime where it does not fit ever. We should exptrapolate, and be dubious of experiments that don’t fit if everything fits so well. But if evidence accumulates then we move to new things.

    I just don’t think there is a way to know if we have complete knowledge, so supposing so
    it’s never a conclusion, it’s faith. There is nothing wrong with having faith as long as you
    make it clear that that’s the case and as long as you don’t let it come in the way of accepting evidence

  45. HI

    Laplace could have very well been an arrogant man, but, to be fair, what he said was a natural consequence of Newtonian physics. And Newtonian physics is just a theory trying to describe the nature. The theory itself is not arrogant.

    What I’ve been troubled is this value judgement. It seem to me that the critique of modernity is as absolutist as what it tries to critique. It necessarily take a black and white view, because it is not a matter of how accurate the theory is, but it is the matter of whether the theory is associated with a good or bad attitude.

    For instance, when you wrote that you could not stand Chomskyan because it’s too Cartesian, you don’t care whether it is a useful theory in linguistics or not. It had to be rejected because it is Cartesian.

  46. Hi, I do care that Chomskian linguistics is a useful theory! I don’t think any group of rigorous thinkers has ever gotten behind an effort in a discipline without it being useful and teaching us more about the larger state of affairs.

    In fact, I have taught it. I think it is particularly interesting on syntax. But I also believe that it assumes that language is to be regarded as like an axiom-driven system with transformation rules, like Russell’s PM or a computer program. This is a huge assumption, but they take it as the obvious way to approach it and their efforts coincided with the end in the U.S. for a long time of the very different approach that underlies Continental thought and cultural theory.

    And whenever we try to show how fruitful this work of ours is, we are told dogmatically that we aren’t using the correct basic approach and so our results aren’t worth considering.

    Once again, we have dogmatism on one side spurring a counter-dogmatism on the other side.

    At the heart of Continental theory lies a different linguistics that treats the language phenomenon as a meaning-system in which individual elements are most defined by what they are most contrasted with and therefore mutually constitute one another. And as I tried to explain, the upshot of it all is that we could not begin to know the world without our language and yet language also focuses and limits our attention and approaches.

    When I talk about “blindness and arrogance,” I don’t mean that individual persons were blind or arrogant, but that the culture held a very strong view of the infallibility of its scientific knowledge. To me, a country that upholds the arts and sciences education should have a balanced and qualified and thoughtful understanding of the dangers of taking any one system of thought too far. There have to be checks and balances and thoughtfulness and dialectical conversation, which it seems to me you folks do have as a value, personally. What I can’t get you to do is to step outside your systems of thought and think about how they would appear from the outside, that is, in fresh or different contexts. In a long view of history, for example, where the Newtonian era was very bold and liberating and also very oppressive and dogmatic.

    I need you to distinguish formally between a discipline and a dogmatic holding of that discipline. You say,
    “And Newtonian physics is just a theory trying to describe the nature. The theory itself is not arrogant.” If only that were so! If only we were ever disinterested seekers of truth whose finding were harmless! Maybe the problem is that we are just too optimistic in this country and we don’t want to be told that everything we do has ramifications far beyond what we may have intended. (This is the “Bad News” of poststructuralism. So we are asked to “own up” to those unforseen eventualities.)

    Hi, I appreciate you “calling me” on what you see as the the absolutist value judgments I make. When I’m sounding like that, I’m not helping myself or my endeavor. But I want to call you on this assumption that if science is pursued honestly and open-mindly, then it cannot close down a culture. It can and it has, when it is taken to be the only way to do things or the only field that deals with truth. I am not worried about you being absolutist. But what about the others out there?

  47. HI

    “What I can’t get you to do is to step outside your systems of thought and think about how they would appear from the outside, that is, in fresh or different contexts.”

    It all come down to this, doesn’t it? What I’ve been trying to write in several of my comments (and I guess Rob Knop did at his blog or Sokal did most famously) is how you on the postmodernists side appear from outside. And also in my case how the philosophycal quarrel within the West appears to someone from outside.

  48. Yes.

    And for you scientifically minded folks, in order for you to come into the inside of poststructuralist thought, for instance, and think with me, inside of my paradigms for thought, that would in effect require you to learn the way of thinking we use, and that in itself would carry you outside of science into a different (foreign) language.

    Hi has a very specially interesting viewpoint as a scientific Japanese person, who sees the language of science as a kind of universally honest and reasonable language, for Westerners and non-Westerners alike, if I have heard him correctly over many posts.

    For someone like me, on the other hand, who grew up as a literary-minded girl inside North American scientific rationalism, it was horribly oppressive and limiting and life-denying. Discovering art (especially Asian art with its very different esthetic values from Western representational art) was one way that I finally was able to escape into a place where I could be and think freely and honestly.

    Saussure’s linguistics and Continental philosophy were other places where my mind could take wing and bear witness to the truths that I had encountered in my life journey. But I love science and mathematics so much, too. In themselves, they are beautiful and clean, and I would wish for them to be taught and nurtured in young persons in college, without the accompanying exclusionary impulses that exiled me from the life of the mind and the spirit throughout my formative decades.

    (Also, the women’s movement and people like Luce Irigaray saying “to speak is NEVER neutral,” made it possible for me eventually to have the societal support I needed to struggle free of genuine bondage.)

  49. Rick

    May I offer an alternative reading?

    Science is done by people, and these people do not leave their societies’ preconceptions and their personal beliefs outside the lab, and become “rational scientists” while they work as scientists. They carry into their scientific work, as carry into their religious beliefs and their political understandgs, their own broader societal mind-sets and world-views derived from the societies in which they otherwise dwell, and the do their interpretations of their work within the contex of these societal and personal mindsets.

    When a person lives in a society that has some sort of “cartesian absolutism” as a part of its general societal worldview, then these people will carry into their activity of doing science this worldview. If the society has a worldview that treats knowledge as more contingent, then those from that society will bring this view with them to the science.

    Janet notes that “Hi has a very specially interesting viewpoint as a scientific Japanese person, who sees the language of science as a kind of universally honest and reasonable language, for Westerners and non-Westerners alike”. I take this view as correct; science does “abstract away” the cultural and language differences, and thus precisely does become a “universally honest and reasonable language”. However, in practice, this “abstracting away” occurs as a sometimes extended process that arises through the practice of the general methodology, through the broader “conversation” not only between the people doing science and that part of “nature” they are seeking to understand, but among the people themselves as they discuss, cooperate, argue, steal each other’s research, engage in petty feuds, and act in all the ways in which people treat each other, both good and bad.

    Now undoubtably the society in which those of us, like (I think) Janet and me, who are perhaps significantly closer to the end than the beginnings of our careers, grew up could be, as Janet said, “horribly oppressive and limiting and life-denying”; I grew up in the american south and am old enough to remember the pre-civil rights times an the segregation.

    However, to call this somehow “North American scientific rationalism”, and lay these societal sins at the feet of “science”, in particular “physics”, is simply wrong. There is, and was, nothing _in the science per se_ that caused, or led to, these attitudes and societal practices. The science is, and was, and forever will be, equally true, or equally false, for those slide-rule wielding boys like me and the literary-minded girls like Janet. There is, and was, nothing in the _science_ that excluded women or african americans, although there was plenty in the _society_ that excluded them. And, of course, those who did science tended to bring with them these societal attitudes, and thus sometimes acted “as if” it were the science, but it never was. Nature does not care the gender or race of the person doing the experiments; even if society does.

    It may well be that our american culture is undergoing a “paradigm shift” away from an ingrained and unconscious “cartesian rationalism” to some other “postmodern” way of thought. But the “cartesian rationalism” of pre-quantum physics was _imported into_ the science _from_ the broader culture, not _exported from_ it _to_ the broader culture. And to the degree the that there is a different “way of thinking” that qualifies as a “critique of modern rationalism”, this too is _imported into_, not _exported from_, the science.

    Our society had, and has, problems. But these problems are not caused by evil Newtonians, and will not be cured by enlightened quantum machanicists.

    This may be at the root of the communication problem. You, Janet, seem to be attributing as _essential_ to science something the we, for this side, see as at most accidental, and most likely, irrelevant, to science _as science_ and to the whole scientific enterprise. We do not deny that the individual people who do science bring to their doing of science their individual cultural, societal, and personal charactistics, but rather that these thing are, in the larger view, simply not relevant to the science _as science_. You, by emphizing the (to us) merely accidental, do not see the (to us) essential deep continuity.

  50. Paul

    Dear Prof. Janet —

    I admire the work you’ve done in trying to accomplish some communication with the “science types,” and speaking as one I can offer you a little advice on an approach that might prove fruitful for you.

    Imagine as a thought experiment that you meet a new person, somewhere, and in conversation they tell you the following:

    * “I have a new theory, a new way of looking at things, that yields beautiful insights into deep areas of human mystery.”

    * “My approach is rigorous and precise.”

    * “No, I can’t just `explain` it to you, as your own language is too limited. In order to understand anything about this beautiful yet rigorous subject you will have to learn an entirely _new_ language.”

    * The person claims to respect you and your day job, yet their sentences are somehow laced with hostile and accusatory language (“close down a culture”, “horribly oppressive”, “life-denying”) whenever the subject of your day job comes up.

    My first question for you, then, is: would it not be natural for you to conclude — at least provisionally — that you’re talking to a crazy person? More exactly, what would you do, what questions would you ask, to satisfy yourself that the person across from you is _not_ simply a lunatic with delusions of grandeur?

    In research physics we encounter people with unorthodox views, some of whom I refer to as “cranks”. Over the years I’ve noticed that cranks typically exhibit most or all of these behaviors:

    * Cranks spend a lot of their time/space staking grandiose claims, and comparatively little time/space getting to anything of substance.

    * Cranks express their results in a private notation, often in the form of long and elaborate derivations. A sure sign of a crank is that he will refuse to translate his derivation into any standard notation, while at the same time insisting that _you_ address _his_ claim purely within his framework.

    * Cranks typically exhibit a hostile and defensive attitude, and are quick to claim that the lack of respect they receive is due to their challenging posture rather than the fact that they don’t have anything of substance to say.

    Personally, unless I literally have to catch a plane I always try to give cranks the time of day and a respectful hearing; and I’ve always encouraged my students to do the same. A practical difficulty often arises, though, when you’ve been listening to a crank for a good amount of time and you just can’t see how what he’s claiming is either self-consistent, well-defined, or really distinct from existing theory. At times like this, I tell my students, a quick but completely fair shortcut is simply to ask “What experimental outcome does your theory predict differently from the standard theory?” At this point you can both then concentrate on what results have been observed, or with no disrespect send the crank back to the drawing board.

    With this part of my background experience in mind, I invite you to instruct me: How should I distinguish you from a crazy person? What questions should I ask? If an impostor were walking the halls of your department, who knew the sounds of your language but none of its substance, how could I most quickly distinguish between you and the poseur?

    I won’t limit you to “predicting experimental outcomes”, of course. Rather, the field is entirely open; you set the terms by which you want to be appreciated. If you can accomplsh this, in fairly straightforward standard English, then I predict you will able to gain substantial traction with the “science types” that you want to reach.

    Best of luck; regards,

    Paul

  51. Well thanks, Paul. I have a feeling that this is why Ferdinand de Saussure vowed never to publish his lecture notes (and never did). He said, for this theory of linguistics, every single word linguists use would need to be redefined. He got really depressed.
    His students published his lecture notes anyway. (For 85 years those very different terms have inspired powerful thought, though often rejected in the US.)
    I refuse to get depressed.
    And I would remind you that no one in my field is going to take time out to talk to the science types, because they assume they simply have “a naive epistemology.” I know that you do not. You have an incredible epistemology for the kind of thing you study. I want a conversation. And yes, I’ve been a little discouraged about it, too. Feel free to write me off, Paul. I’ve written my head off and I’ve been quite lucid. What more can I do? If I think of something, I’ll do it. I’m working on ideas/responses.
    I was looking for the Saussure diagram I would need and found that it is being used along with the Taiji diagram to begin a conversation between Chinese and Continental philosophy! (I don’t know enough about this yet to comment, but it’s an example of an extremely difficult effort to communicate between incomensurate philosophies.)
    Also, I would note that perhaps your bent shows why Foucault spent so much time on “madness” and Irigaray’s work came out of working with schizophrenia and language positions. Cultural discourses are built on selection and combination in a way that carries that culture’s quest forward, but the excluded bits and the possible other patterns are right there within the culture’s dominant discourse as its “other.” Derrida shows us that there is no way out of this bind — we cannot wave a magic wand and ordain an oppression-free culture. Any effective discourse and rationality is built upon the exclusion of its other.
    However, the rigidly binary and dualistic culture or system can PERHAPS be moved toward more flexibility and play in the holding of its dominant discourses, but it fights like crazy and to attack it makes it fight harder. I’m sorry that some of you perceive me as attacking when I see myself as trying to explain and be heard. Conversation and communion are, after all, sacred and miraculous things. They depend entirely on good will and good faith.

  52. Gavin

    Wow. It looks like I started a dog-pile-on-Janet thing here, which was not my intention. My goal was to get some discussion of the actual science going, since it seems like there is a belief that the actual science might have some bearing on the issues of knowledge and understanding and all. I don’t see a single equation since I left, so it doesn’t look like I got the ball rolling in the right direction. Since it appears there are some scientists here, let’s talk about science. Non-scientists can ask questions and learn what we actually do. Then we won’t have to keep telling them that they have no idea what we actually do.

    It’s way past my bed time, so I’ll answer some of the questions and take on David in the morning.

  53. David

    I’ve been waiting for you to get back Gavin. I would like to put up some thoughts about decoherence and the measurement process with some equations. I’ll let you start the ball rolling and then I’ll try add some thoughts and doubts of mine.

  54. Gavin, you have nothing to reproach yourself for!

    And Hi and Rick are addressing the very core of the difficulty in scientists and humanists conversing with one another. I need to mull over how to respond to their very thoughtful posts.

    I’m looking forward to more of the QM science from Gavin and David — in part (I must admit) because I’m eagerly waiting for you to get to the measurement problem! I have studied QM and worked with physicists and read and taught a lot of the history of science, after all.

    This matter of the paradigm shift in science with Einstein and QM… I don’t want to just throw out “proof-texts” from Einstein. I’d rather get to know where you see the shifts as working scientists. You have convinced me that my statements in the debated paragraphs 4 & 5 of my lit theory course (Part 4 above) are way too sloppily stated.

  55. David

    With Einstein, as far there was not a big shift as far as special relativity. I had very important consequences and predicted very novel and nonintuitive phenomena stemming from the lack of an absolute time, but it’s view of physics was not too different than classical physics. The postulate of relativity is the same used by Newton and that holds in Newtonian physics. The novelty comes in imposing the speed of light to be the same for all observers. This leads to the abandonment of Newtonian absolute time and to many novel interesting consequences.

    General relativity was a much bigger shift, not on classical deterministic thinking per se, but because it made spacetime itself a dynamical entity and that gravity is spacetime geometry. I consider this a very radical change from previous thinking.

    Quantum Mechanics with its intrinsic probabilistic interpretation, the measureemnt problem, and its superposition principle for matter (since matter is really waves) was an enormous shift from previous thinking I believe.

  56. Your descriptions and judgments about GR and QM, David, accord with the picture I thought I had received from many sources. (I want to get back to SR later sometime.) Especially, I think, for humanists, the three elements of QM you just mentioned have been very important. Let me expand on that a little.

    I’m assuming, but correct me if I’m wrong, that it is fair to say that with QM, science moved from the view that all matter obeyed strictly deterministic cause-and-effect laws, to recognizing that (apparently, at this stage of knowledge) matter on the quantum level exhibits behavior that can be described very precisely with probabilistic laws, but each individual particle appears to be able to behave almost randomly (WITHIN THOSE PROBABILITIES, of course). Almost as though each particle has “free will” is a description I’ve heard physicists use — just as a metaphor of course. (!)

    I hope this is right, because it is thrilling for humanists — for philosophers and theologians and poets — especially because it seems to be directly related to chaos and complexity theory, where new higher-order complexity can emerge spontaneously from certain conditions of turbulence. (What gorgeous news!)

    Any sense of spontaneity and freedom in the natural world is such a relief, because for several centuries we have had to try to reconcile ourselves to this grinding, inexorable, strictly deterministic natural law. And yet, it is very hard to experience the natural world or organic beings or especially animals and babies and our own bodies as being this inert, lifeless, mechanistic stuff that Descartes proposed it all was. (Res extensa, the stuff that extends in space and time.) I think, David, that your phrase “more dynamical” above may be more powerful than you intended (though you use it in a somewhat different context.)

    At the same time, if I am not mistaken, QM is also directly related to the new view that at every point in our universe’s development there was a certain degree of quantum indeterminacy. This would mean that any other universe, even with the same initial conditions and everything else the same, would have a unique history, not exactly the same as ours, within specific parameters, of course.

    Again, this is just the most marvelous breath of fresh air! It is exciting and imaginative and also rigorous at the same time. If we would teach this in schools, the kids would be blown away by the mysteries of our existence, instead of portraying science as the giant Grinch that stole our deepest intuitions and our imagination and our awe (which are deeply involved in many people’s personal religious feelings) from us. (Religious people will support science and embrace it happily when it isn’t such an endless kill-joy. We can’t let biblical literalists turn scientists and religious people against one another because they are united in the simple human desire to know — as John Haught puts it.)

    Any degree of freedom or spontaneity or “tolerance” is thrilling for poets and theologians and philosophers, who had had to try to resign themselves previously to this heavy linear determinism for everything in the universe, including organisms such as animals and babies and human bodies, all of which give strong impressions of more awareness than Cartesian mechanism had allowed. (I’m so glad neo-Darwinians are studying conscousness now on its own level, less reductively. Even though it all “reduces” to physics, nonetheless, the explanation on the level of physics isn’t enough to formalize the phenomena that can be studied on the level of consciousness.)

    On the other side, I have poet friends counseling me that we should be urging the “soul” or the “heart” — as a pushing back against scientific reductionism. But I’m too much in the Greek and medieval tradition, where the formal elements in the material world are its “mind” or “life.” Motion and design are plenty enough to call matter something other than inert and dead. (I am living sooo dangerously….)

    Anyway, does this help you understand why I doubt that simple science can ever be just simple science? (That we can neatly divide the doing of science from the people who do it?) Everything always has cultural ramifications, sometimes very different ones from what we would expect or want, just like religion or philosophy do. And we don’t always expect or like the ramifications, but it is always very exciting and interesting to deal with them integratively by talking to each other.

    By the way, here’s a quotation from old Albert Einstein just to play with. I’m tracking down the source (essay collection).

    “In the year nineteen hundred, in the course of purely theoretical (mathematical) investigation, Max Planck made a very remarkable discovery: the law of radiation of bodies as a function of temperature could not be derived solely from the Laws of Maxwellian electrodynamics. To arrive at results consistent with the relevant experiments, radiation of a given frequency f had to be treated as though it consisted of energy atoms (photons) of the individual energy hf, where h is Planck’s universal constant. This discovery became the basis of all twentieth-century research in physics and has almost entirely conditioned its development ever since. Without this discovery it would not have been possible to establish a workable theory of molecules and atoms and the energy processes that govern their transformations. Moreover, it has shattered the whole framework of classical mechanics and electrodynamics and set science a fresh task: that of finding a new conceptual basis for all physics. Despite remarkable partial gains, the problem is still far from a satisfactory solution. ” (Albert Einstein, 1954)

  57. Gavin

    Janet,

    David did a great job of articulating the magnitude of the various shifts in thinking, and it looks like you understand what he has said. However, your post makes many connections and jumps that are not scientifically sound.

    Quantum mechanics did take us away from a purely deterministic model at the quantum scale (processes where the action, computed with a precise formula, is not too much bigger than h). You then say

    I hope this is right, because it is thrilling for humanists — for philosophers and theologians and poets — especially because it seems to be directly related to chaos and complexity theory, where new higher-order complexity can emerge spontaneously from certain conditions of turbulence. (What gorgeous news!)

    I’m not sure how this is related to either chaos theory or complexity theory, except metaphorically. Chaos is a feature of classical systems, it doesn’t depend on quantum mechanics at all. Complexity theory is about the difficulty of computing solutions to certain problems using digital computers. I don’t see how it is related to anything else that you mention. The world is analogue, whether it is described classically or quantum mechanically, so complexity theory doesn’t have anything to say about most systems. Correct me if I’m wrong here. Maybe there is some branch of complexity theory or use of that term that I’m not aware of.

    Any degree of freedom or spontaneity or “tolerance” is thrilling for poets and theologians and philosophers, who had had to try to resign themselves previously to this heavy linear determinism for everything in the universe, including organisms such as animals and babies and human bodies, all of which give strong impressions of more awareness than Cartesian mechanism had allowed.

    Quantum indeterminacy appears to be no friend of free will, for two reasons. The first is somewhat philosophical, so it may be I just misuderstand. The randomness is totally random. I don’t want to exercise free will by doing things at random, I want to be making decisions. Randomness seems to be detrimental to my ability to control my destiny, not helpful. The second conflict is scientific and I feel much more comfortable with it. There is no evidence that the randomness at the quantum level has any impact at the scale of neurons, whose action is far, far larger than h. Certainly chaos theory has important things to say about brains, and complexity theory may as well (since neurons are digital, mostly, but also asynchronous, which makes the brain not a turing machine.) However, quantum mechanics is not relevant to understanding how brains think. I see no physics justification for why a brain in a classical world would be any less conscious or have less free will than brains in our quantum world. The quantum effects are far, far to small to play a role in thinking. At the scale of thinking, our universe is so close to deterministic that the quantum effects hardly even count as a whisper of a rumor of noise.

    Going back to an earlier comment, you say

    But believe me folks, I could even live with that — a complete and final description FOR PHYSICAL (i.e. physics) REALITY if only it were not implied that this means ALL OF REALITY. But clearly, for many scientists and for the scientistic science bloggers on many sites, it does mean precisely this. It leaves no room for any “reality” on other levels of structure and other realms of human life, so Hi’s comments are very very important.

    For the record, I am one of those scientists who thinks scientific reality is all of reality. I don’t want to enter that discussion yet, but I do want to confront this notion that somehow science contains within itself the revelation that this view is wrong. I see this in comments like
    This is why I think what Jacob Bronowski tries to say is so important. And the role of the observer in Relativity and the role of measurement in QM. Because prior to that, it was too easy for Westerners to think that the laws and formulas equalled reality, or were a perfect one-to-one fit. We forgot that our efforts to come to know were involved at all. The mediators of the theory drop out of sight too easily.
    The role of the observer in relativity does nothing to diminish our view of the world as knowable, objective and deterministic. Quantum mechanics does challenge those things, right up until the point when you actually do a calculation and discover that the quantum effects are dozens of orders of magnitude to small to effect anything in everyday life (like consciousness and free will).

  58. Gavin

    In the previous comments one of my quotes didn’t get block quoted. I should be

    . . . . I see comments like

    This is why I think what Jacob Bronowski tries to say is so important. And the role of the observer in Relativity and the role of measurement in QM. Because prior to that, it was too easy for Westerners to think that the laws and formulas equalled reality, or were a perfect one-to-one fit. We forgot that our efforts to come to know were involved at all. The mediators of the theory drop out of sight too easily.

    For the record, . . . .

  59. David

    I only want to reemphasize one of Gavin’s point because is a misunderstanding I see often.
    Chaos occurs in systems that are perfectly Newtonian, or perfectly deterministic. I have never understood where the misconception arose.

  60. Gavin

    I’ve been thinking about whether there is a concept in science that is being misidentified as relativity/QM/chaos/complexity, but actually does some of what Janet and others think it does. The one idea I had was collective phenomena. I’ll give an example.

    Liquids are pretty well understood. There are different sorts of liquids (water, mercury, honey, etc.) which behave somewhat differently. However, it only takes a few numbers to describe a liquid (density, compressibility, and viscosity, unless you want a couple thermal properties as well). These few numbers are all that is required to understand how the liquid will behave.

    However, at the atomic level there are huge differences between different liquids. Some are made of big molecules, others small, some liquids don’t involve atoms at all (like the liquid of conduction electrons in a metal). All the details of molecular structure are totally unimportant in understanding the behavior of the liquid. In fact, they are pretty much useless. There isn’t any way to start with the electron and proton interactions and predict the existence of liquid hydrogen. It is just too difficult to calculate how all of those hydrogen atoms are going to behave when you pile them together. Once you know that hydrogen forms a liquid, then you can use your knowledge to predict some of the properties, but actually predicting the existence of the liquid is tough.

    There is a certain sense in which ideas about liquids are independent of our ideas about atomic physics. When studying a particular liquid we can make connections, but the idea of a liquid is independent of atomic physics. This is an example of “liquid physics is not just applied atomic physics.” Liquid physics is something broader.

    This sort of hierarchy goes on at all levels. The standard model is independent of the particular grand unification scheme. The theory of computers is independent of the types of logic gates used (transistors and Tinker Toys obey the same rules). Evolution does not care which codons code for which amino acids. All of these are examples of collective phenomena where there is a field that studies the larger thing which doesn’t need, or even have use for, the theory of the smaller things that make it up.

    Looking at this, I think liquid physics is a profound and interesting subject. I would never claim that it is “just” applied atomic physics. Likewise, the theory of the mind is fascinating, it is not “just” applied neurobiology. The mind is a collective phenomena of many neurons, but I suspect that the details of neurons are of little use in understanding minds and that minds could be made equally well out of transistors.

    Is this the sort of idea you’re looking for, Janet?

  61. Rick

    I think that there is a basic logic being used when we start talking about this sort of thing, which runs, basically,

    (1)[deterministic |- predictable]
    thus,
    (2) [~predictable |- ~deterministic].

    This is often coupled with

    (3) [predictable |- ~ freewill] and thus
    (4) [freewill |- ~predictable].

    This is then taken to mean (by the logically erroneous steps of denying the antecendent/ affirming the consequence), to mean something like

    (#3) [~predictable |- freewill]
    and
    (#4) [~freewill |- ~predictable].

    Then, thrown into the mix, is something like

    (5) [random |- ~predicable].

    Quantum indeterminacy implies unpredicability which in turn implies, by (2) non-deterministic, which then, by (#3), is taken to implicate freewill.

    Chaos theory implies implies unpredicability which in turn implies, by (2) non-deterministic, which then, by (#3), is taken to implicate freewill.

    Complexity theory implies unpredicability which in turn implies, by (2) non-deterministic, which then, by (#3), is taken to implicate freewill.

    Of course, the problem is here that “unpredictable” means different things in each of these cases.

    In the case of quantum indeterminacy, there does seem to be a basic randomness built into the structure of the universe when we get down to the level of “really small”. But, as Gavin notes, “really small” means, in a lot of practical cases, “irrelevant”.

    Chaos theory deals with, as Gavin and David note, classical deterministic systems. The “unpredictabiliy” here arises because the equations, when applied to the real world (such as weather prediction), are exquisitely sensitive to the initial conditions, and thus very small differences in our starting data can yield very different results.

    Complexity theory, on the other hand, is generally “unpredictable” in only the most mundane sense, which is, as a practical matter, it takes too long. A classic introductory problem to complexity theory it the travelling salesman problem, for which writing an algorithm to solve is a fairly trivial task, if all you want is, for some fairly large number of cites, to leave your (great^n)-grand children the solution.

    As an aside, I would note that in general the “new higher-order complexity [which] can emerge spontaneously from certain conditions of turbulence” can arise through purely classical, deterministic, Newtonian type-processes. We do not need to invoke quantum indeterminacy or the like in our theries of, for example, hurricanes, which are a form of “higher-order complexity” which “emerge spontaneously from certain conditions of turbulence” in the atmosphere.

  62. David

    Well put Gavin. It is not only liquids, most macroscopic phenomena behave this way.
    All superconductors behave the same way independent of microscopics. All magnets behave the same way independent of microscopics. And so on. While their properties arise from micrsocopics, the macroscopics behavior is independent of such details in the sense that systems with completely different microscopics have the same macroscopic collective behavior. For specialists this is the phenomenon of universality understood for physical systems as fixed points of the renormalization group. Even when these systems exhibit quantum behavior it is independent of the microscopic details.

    A good reference for this type of thinking is an essay by Phil Anderson in Science in the early 1970’s. I believe it’s from 1972 but I am not sure of the exact year.

  63. David

    This was very insightful Gavin. It is something missunderstood by a lot of physicists I believe and something that took me a long time to grasp as a graduate student. It’s the only reason the standard model works like you say even if we do not know the ultimate theory (if there is such a thing). In the standard model it works exactly the same as in your water example through renormalizability. Once we fix a few renormalizable values at some scale everything is know at all lower energy scales independent of what happens at higher energy scales, that is at more microscopic levels. I would be more extreme I say that whenever a field theory works (whether classical as for water – hydrodynamics- or quantum – the standard model, a phase transition, a superconductor- ) is whenever we have such collective emergent behavior that has a life of it’s own independent of microscopic details.

    I apologize for some physics jargon.

  64. Gavin

    I think that nearly everything I have said so far is uncontroversial among scientists. Enough of that! Let’s talk about the quantum measurement problem. I’m not going to be terribly careful about defining everything. David and other physicists will know what I am talking about, so we can have our discussion in slightly technical language. However, if you are a non-scientist, please ask lots of questions.

    It is a long cherished myth that quantum mechanics introduces a an element of randomness into reality. This occurs when a quantum system is measured and the wave function collapses to one of the allowed outcomes of the measurement. The probability that a certain outcome will occur is given by the square of the amplitude of the wave function for that outcome. While the probabilities can be calculated, the actually outcome cannot be predicted.

    It is the view of many physicists that this interpretation is wrong. Quantum mechanics has a perfectly deterministic rule for describing how a system changes over time, the Schrodinger Equation. This equation tells what the system is doing between measurements, but this equation has not generally been used to describe the measurements themselves, which appear non-deterministic. However, we now have a pretty good understanding of how to use the Schrodinger equation to describe the measurements themselves. Since the Schrodinger equation is deterministic, this allows us to predict exactly what will happen when we do a measurement, every time. However, what we predict is very weird.

    Let’s say we have a quantum system that can be in two independent states, |a) and |b). Let’s also describe he measuring apparatus as having three independent states, |?), |A), and |B), where |?) is the state of the device before it makes the measurement. I can denote the state of the quantum system and the apparatus using two symbols. For example, if the quantum system is in the state a before the measurement, then the system is in the state |a)|?).

    To have the measurement work, the Schrodinger equation needs to cause the measurement device to give right answers.

    |a)|?) -> |a)|A),
    |b)|?) -> |b)|B)

    where the arrow represents the action of the Schrodinger equation. The interesting issue is what happens if the quantum system starts in the superposition |a)+|b). (I’ve left off the normalization factor because this comment space isn’t very good for working with equations. I want to keep it simple.) Note that the state is not |a+b) because the system only has two independent states |a) and |b), there is no additional independent state. It would be nice if the device could display that the system was in a superposition. However, it doesn’t. We can actually calculate the final result because the Schrodinger equation is linear, which means that if

    |x) -> |x’)

    and

    |y) -> |y’)

    then

    |x) + |y) -> |x’) + |y).

    Using this formula on our situation we find

    [|a) + |b)]|?) = |a)|?) + |b)|?) -> |a)|A) + |b)|B)

    Now we know the result of the measurement, but what does it mean. We now have a superposition of the quantum system AND the measurement device, with the measurement device giving the right answer without it being clear what that answer is.

    Note that this is not a superposition of the quantum system with a superposition of the measurement device. I.e.

    [|a) + |b)] [|A) + |B)] = |a)|A) + |a)|B) + |b)|A) + |b)|B)

    which is the wrong answer. The right answer is called an entangled state.

    In conclusion, the quantum mechanics gives a deterministic description of what happens, but the result is rather unsettling. Should we panic? I think not. You can show that if a person becomes entangled with the quantum system, the person will not be able to tell. He will be confident that he has the right answer (and he does) even though he is in a superposition of states with different answers.

    This sort of thing is going on all of the time, with us getting more and more entangled with our environment. This is often called the many worlds interpretation, or the Everett interpretation (the latter is the more sophisticated version summarized here).

  65. |x) -> |x’)

    and

    |y) -> |y’)

    then

    |x) + |y) -> |x’) + |y).

    If the state x goes to the state x’ and the state y goes to the state y’
    then (the superposition of) state x + state y goes to (the superposition of)
    state x’ plus y? (Or y’) Is this a typo? If not, why is it this way.

    Anything you can say to clarify how a mathematical “superposition” arises in the first place into your considerations, in terms a humanist can visualize or grasp? Why/how do superpostioned states arise — is that the description of the indeterminate state before the measurement? And remind us again of why it can’t just be state |a OR state |b before the measurement and we just don’t know which it is.. Was that what the double slit and the beam splitter experiments were showing us?

    P.S. I got up this morning and I saw tht you guys were off on a big conversation amongst yourselves, and I went to make myself my morning cup of tea. So then out in the kitchen I asked myself, “Okay Janet, what do you suppose they are saying?” I think back to my earlier experiences and I make for myself my humanistic, probabilistical projection, and I say to myself. “I don’t have the SLIGHTEST IDEA what they might be talking about yet, but I bet it won’t have anything to do with the main thing I thought I was trying to say.” And I laughed. “Viva le difference!”

    Sure enough, the thing I was trying to say and wanted you guys to consider — you don’t even MENTION. I think this is hysterically funny and wonderful!

    By the way, are you saying then that given the same initial conditions and the same scientific laws, our universe would have played itself out exactly as it has, every time? Doesn’t this have something to do with the planck constant time-intervals very early in the (big bang…) being subject to quantum indeterminacy.

    Your comments are great!!! I have much to digest. I really did think I was right about what I was saying about chaos theory (ok I shouldn’t have said complexity), so you guys do have a beef with us humanists. I mean I have studied this stuff with physicists etc. I’ll have lots more specific questions for you when I have time to formulate and research them. Keep going. David said he “has doubts” (earlier) about the measurement problem…..

    And the “many worlds” interpretation — that all these things happen in different possible worlds has NOTHING to do with the multiple universes argument against the anthropic principle to show it does not suggest that our universe was purposefully fine tuned for life — right? We don’t want any MORE confusion in our silly humanist heads….

  66. Gavin

    Janet,

    Thank you for catching the typo, and in an equation no less! You are really trying to have a dialog. It should have said

    |x) + |y) -> |x’) + |y’).

    I’ll think about how to describe superpositions. It will involve math, there’s no way around that, but I’ll make it as painless as possible.

    Mixing cosmology and quantum mechanics is still too speculative, so I am going to abstain from answering questions about Planck time and the anthropic principle.

  67. Gavin

    I’m going to see what I can do about superpositions, and to do this I’m going to introduce the density operator. If you are a physicist, I can hear you screaming. But I’m going to try none the less. I really want to do this right and not just give metaphors.

    Janet asks if |a)+|b) is just a way of writing “either |a) or |b), I don’t know which.” It is not. There is a way to write that with a density operator. Here it is

    1/2 |a)(a| + 1/2 |b)(b|

    This density operator represents something that is either in |a) or |b), with each having probability 1/2. If the object was in the pure state |a) its density operator is

    |a)(a|

    If the you don’t know if it is in |a) or |b), but there is a 3/4 chance it is in |a) but only 1/4 chance it is in |b) then the density operator is

    3/4 |a)(a| + 1/4 |b)(b|

    I think you get the idea. If there are more states than two, the list of things getting added together is longer. The sum of all the coefficients has to be 1, because the probability that the system is on some state is 1. All of the coefficients have to be real numbers between zero and 1 (inclusive).

    To describe the superposition from my previous comment I need to add terms called coherences. The result is this

    1/2 |a)(a| + 1/2 |b)(b| + 1/2 |a)(b| + 1/2 |b)(a|

    These terms that mix |a) and |b) tell us that |a) and |b) aren’t just possibilities, they are in a coherent superposition. There are some rules regarding what you are allowed to put in the coherences, which you may not need to understand, but here they are. The bigger the number the more coherent the states are, but you can only make states coherent if these is some probability of them being in that state. So if you want to have |a)(b| terms then you must have both |a)(a| and |b)(b| terms that are big enough (“big enough” is precisely defined: |z|^2 is less than or equal to xy, where z is the coefficient of the coherence and x and y are the coefficients of the homogeneous terms.) The coefficients on the coherences can be complex numbers, and they must be complex conjugates of the coefficient of the term with the states reversed. This allows phase information about the coherence.

    All of that has got to appear totally crazy, but the point is that there is a difference between “either |a) or |b)” and the superposition |a)+|b). Those two things have different density operators. Why do you care about density operators? Because they can tell you what the results of measurements will be. I’ll show that tomorrow.

  68. Gavin

    Janet,

    I forgot to answer your specific question about the double slit experiment. The double slit arrangement does produce a photon in a coherent superposition. Label the two slits a and b. Then a photon coming out of slit a is in the state |a) and a photon coming out of slit b is in the state |b). If photon could come out of either slit with equal probablility but is not in a coherent superposition, then the density operator for the photon is

    1/2 |a)(a| + 1\2 |b)(b|

    Which won’t produce any interference lines. This could be done by having a separate light behind each slit and turning one or the other on with a coin flip. This is also what you get if you use the wrong sort of light source (e.g. one that is too big).

    If you use the right sort of light source to illuminate both slits then you get the coherences in the density operator.

    1/2 |a)(a| + 1/2 |b)(b| + 1/2 |a)(b| + 1/2 |b)(a|

    This density operator will result in interference lines. Here the two parts of the superposition are in phase (a phase difference of 0). You can change the phase of the coherences by moving the light source slightly so it is a tiny bit closer to one of the slits than the other. If you move it so the difference in the two distances is half a wave length, then the density operator becomes

    1/2 |a)(a| + 1/2 |b)(b| + i/2 |a)(b| – i/2 |b)(a|,

    where i is the square root of -1. This is a phase difference of 90 degrees. (The last term has a minus sign because its coefficient has to be the complex conjugate of the coefficient of the other coherence. The complex conjugate changes the sign of the imaginary part of the complex number.) You can also get things in between, like

    1/2 |a)(a| + 1/2 |b)(b| + (3/10 + 4i/10) |a)(b| + (3/10 – 4i/10) |b)(a|,

    which represents a phase difference of about 57 degrees.

    You can also get things that are partial superpositions. For example, if your light source is good but not great, you might get

    1/2 |a)(a| + 1/2 |b)(b| + 1/3 |a)(b| + 1/3 |b)(a|.

    The coherences are not as strong as they could be, so the interference pattern is not as sharp.

  69. David

    I am quite busy today, but will try to weigh in tomorrow with some specific comments about coherence, the double slit experiment.

    And ohhh Gavin! Why did you have to bring many worlds? Why, why? Actually I dont’ think there is anything wrong with it, but I don’t think it solves the problem of the probabilistic interpretation. It’s consistent, but I would say in practice it does not lead to different outcomes than collapse.

    Today I am to busy, I will weigh in tomorrow

  70. Okay, I have to tell you that I actually kinda am following what Gavin is explaining by bringing in density operators. But it is ONLY because I have been reading and rereading Roger Penrose on QM in The Road to Reality. What other humanists reading this are thinking I can only guess.

    Again, it strikes me as wonderful and wild that when I ask for a clarification, Gavin brings in more mathematical concepts! And I understand this impulse, because that is what constitutes clarification to his highly trained scientific mind. Meanwhile, of course, I am trying to visualize all this, even though I know I’m not supposed to be doing that. I get more and more sympathize with the positivists among you scientists who just say “forget reality.” Do the math. If it predicts accurately, great.

    Remember when I asked: “quantum excitations of a wave?” No one answered me (that I could see, except there were some maths!). So I asked my son and he reminded me about if you have a person holding a rope fixed at the other end and waving it up and down you get undulations along the rope of certain amplitudes and frequencies. So a quantum excitation of a wave would be adding a planck’s constant amount of energy to waving the rope (or multiples of planck’s constant) and so the wave would get more energetic (in frequency or just in amplitude?). And now we are talking about “bundles” of excitations and their densities….

    But it appears that it would not occur to your marvelously scientific minds that I need THIS kind of guidance and help. In fact, it was a journalist who introduced “the way a wave divides to go around a rock and then reconverges in an interference pattern” as a helpful image for grasping the two slit experiment, without which I could not have followed your earlier math!

    It shows so clearly how differently we think about the world. And my main, main, main point is that we need to respect all of these ways, at least when they are thoughtful and open and come out of the rigorous formal work of a disciplinary community.

    But here’s the kicker, of course. In my discipline, the way we are trained to think “constructs” or shapes or enables the way we look at all of reality, insde and outside of our discipline. (Not reality, but the way we look at and interact with reality, which then of course feeds back into what reality seems to be for us. There is always a complex interaction between reality and our discovery procedures and poststructuralism problematizes the attempt to simply equate the results as being entirely composed of the external reality and not at all by our own attempts to know it. We don’t problematize for the sake of being skeptics but in the service of carefully defined and qualified truth-claims and insight into the human condition and our psychological and political psychic structures…)

    I see that happening with you scientists, that your training shapes your approach outside your discipline as well as inside of it, and Hi certainly sees it happening with me, in my outraged reaction to the Cartesian paradigm and to the overly dogmatic claims for science that formed the Enlightenment and the Modern West. (That I carry my disciplinary training outside my field into appraoching other fields.)

    Now I am NOT saying that reality is socially constructed!! (Do you know, I can’t pin down anyone who does say that. Is it just a complete strawman invented out of a misunderstanding of postmodern thought and language?)

    Instead, I am saying that there is so much reality that we have to select aspects of it and “abstract away from” the rest. Any language, viewed as a semiotic system, certainly does this. And so does the specialized languages of a disciplinary community.

    This doesn’t mean there is no reality. This doesn’t mean we cannot genuinely learn about and “know” about aspects of reality. It doesn’t even mean that science won’t eventually arrive at a complete picture of the fundamental physical reality — but I insist that this is a belief-structure (outside of science) and not a scientifically established finding of science. We’ve already documented this and discussed this and I think we at least agree on this.

    So I’m not quite sure what it means then, when a thoughtful scientist says that science does aim at a complete description of all of reality. I don’t think you mean this the way it sounds to me — that you think science is the only way to learn and know about music or humor or how much I laughed in the kitchen when I realized I had no idea what the science-guys might be saying on this site to one another, but it sure wasn’t going to be a response to the main point I thought I was making! Or about Plato’s Ion?

    Yes, Gavin, I really appreciated the liquid dynamics you came up with and David’s comments about your insight. More and more I understand how you folks go about relating scientific theory and evidence to efforts to talk about history and philosophy and cultural studies, within which science is a cultural institution and a highly dominant structural force in society in general. If you can find a legitimate area of science to support my point then it will help the conversation.

    But my fundamental claim was not about whether chaos theory is related to QM or is a classical process but that the strictly deterministic sense of the universe that hung over the 19th century and much of the 20th (remember behaviorism!?) has lifted. I really don’t care that randomness is truly random and not “free,” which of course I did know, but that the iron hand of scientific law has become a more flexible and life-affirming thing of late for us outside of science. The universe has a much more interesting story now than when it was absolutely steady-state, eternal, created in its present state and functioning deterministically to the end of time.

    Now don’t correct my science here!! Just this once, okay. (I’m laughing.) This is my field, the study of cultural meaning and transformation through time. Science as culturally perceived has changed. And this is important. For example, I believe that the huge cultural backlash against science came about when science was so deterministic that it didn’t correspond with the world we knew as non-scientists. And of course, because too many scientists would not separate the science they taught from a crusade for a strictly “scientistic” worldview.

    I think the biblical literalists are fighting their war against science because of, for example, the tenor of high school biology classrooms for decades — do you have any idea how deeply hurt sheltered religious kids were by the evangelicalism of aetheistic biology teachers? (Because I worked with these kids for decades, even though to me Genesis is self-evidently mythic and larger-than-life and poetic and not a science textbook.)

    On the other hand, all it would have taken to defuse a great deal of this would have been the teacher’s willingness to say that the connection between evolution as a science and the question of metaphysical belief in God was a very complex subject and not one strictly addressed by science itself.

    I know you folks are saving the big questions about reality and so forth for later, but I really would like to know whether or not you can see anything in the connection postmoderns make between totalizing worldviews (whether scientistic or religiously literalist) and our deep social polarization, or not? Or that holding a discipline as a monolithic explanation of everything sends a message that is dangerously opposed to the kind of liberally educated mind we are trying to foster in our students. A mind that can work thoughtfully with gray instead of in one kind of black-and-white, and resists authoritarianisms and totalizations of all kind. And bring many perspectives to complex problems.

  71. David

    Chaos occurs in some deterministic systems of differential equations including some systme in classical mechanics and classical hydrodynamics.

  72. David

    It’s good to many perspectives to complex problems.

    I only have problems when people try to bring many perspectives that contradict pretty certain evidence (no evidence is absolutely definitive, but there are instances where son much evidence has accrued that there is little room for reasonable denial)

  73. David, if you are referring to the scientific evidence that the universe and the emergence of life doesn’t need the intervention of a Great Spirit-Being, to me science is conclusive on this. But that does not mean biology teachers should feel free to denigrate religious faith!

    Even for bibilical literalists, a literalistic reading of Genesis is often not fundamental to their religious experience. God’s intervention at certain points in the evolution of our universe is not fundamental to religious experience, either. As Rick has pointed out, as persons we are very complex — and we don’t necessarily even integrate all the things that deeply matter to us.

    It’s interesting that it is cultural theorists and not just religious people who are very discouraged by the Dawkins crusade, because again we are the ones who are very aware of so many factors that go in to making a something like the most precious of all artforms — to an individual or to a culture — and that faith is not necessarily socially pernicious anymore than science is.

    We weave so many things into the very fabric of our being (and much of it without choice) that we need to be respectful of the fabrics of human being. Where they result in crime or bigotry or infringement on a valued discipline or institution, then we have to address those movements and actions, of course. But to just advise people to rip a whole part out of themselves….

    David and I are maybe getting off a bit the subject! I still want to know what I asked above.

  74. Paul

    Taking a slight break from the quantum mechanics discussion (which I think is of high quality!), I’d like to post some testimony on the general topic of this page’s thread. Apparently my earlier post was not so helpful, and since I’m apparently too boorish to be allowed in the house I’ll see if I can contribute from out here in the yard. Sticking to what I know best, which is the day-to-day life of experimental physics, I’d like to make four points (these essentially flesh out Janet’s earlier statement about the epistemology of working scientists).

    1. We are intimately familiar with the concept that our perception of reality is mediated. I look at a dial indicator and see that its needle is pointing to a 3. I’m _very sure_ I know what “needle”, “pointing to” and “3” all mean. I’m _pretty sure_ that the indicator is part of a voltmeter, and that the voltmeter is working. I’m _pretty sure_ that the voltmeter is connected to a Hall probe, intended to measure magnetic fields, since I plugged it in myself some time ago. I’m _fairly sure_ that the probe is still glued to the magnet pole piece and hasn’t slipped off. I’m _reasonably sure_ that the Hall probe is working as the manufacturer described and I so I know the conversion from readout potential in volts to magnetic field strength in gauss. So this chain permits me to believe that by seeing a “3” I have learned something about an entity I cannot sense directly, namely a magnetic field. But this belief is far from absolute, and I am very well aware that the farther I get from my own senses the more tentatively that knowledge has to be held.

    The way we make best progress in these situations is through cross-checks and consistency checks. Even if there’s no reason to suspect that the voltmeter is not working, I’ll swap it for another one to double-check. I’ll re-calibrate the Hall probe in a standard magnetic field. I’ll physically flip the probe over, and also reverse the current driving the electromagnet, and see if these changes produce the expected effects. And so on. Once a reasonable number (you have to stop somewhere) of cross-checks are all consistent, and not before, I can report the results. What doesn’t get talked about as much is the less glamorous reality, namely that _most_ of our day-to-day hours are spent addressing the situation when the consistency checks are _not_ all satisfactory and we have to figure out what’s wrong. Anyone can read a “3” and report a result; our real expert, trained ability is in carrying out the cross-checks, and when they don’t agree finding the most efficient way to design new, further cross-checks and isolate which part of the chain is not in accord with what we thought it was. Appreciating the irreducible uncertainty in our knowledge is, literally, our day-to-day work.

    2. We are familiar with the idea that our vocabulary limits what we can talk about, and even what we can think about. In the above example I might report the value of a magnetic field’s strength without complete reference to how it was measured. This is a useful shorthand, but I realize that it only makes sense within the Faraday-Maxwell vocabulary of magnetic field strength as a flux density in space.

    Rather than try to contrive another story I would rather point you to a wonderfully illustrative example on the theoretical side, found in Kip Thorne’s excellent 1994 general-readership book, _Black Holes and Time Warps_ (I cannot recommend the book highly enough). Two different communities of theorists used two different short, working names (“black hole” in the West versus “frozen star” in the East) for the identical theoretical object (the Schwarzschild solution in general relativity). These are both reasonable coinages; but Thorne describes how the mental pictures they conjure up were sufficiently different that one of the communities was greatly hindered — prevented, effectively — from reaching certain important insights. Buy it for the anecdote, then read the whole book; you won’t be sorry.

    3. We don’t deal in absolute knowledge, or claim to. Suppose I do an experiment by varying A and measuring B, and record the following results:

    A B
    — —
    1 1
    2 4
    3 9

    (This is a simplified example, of course, but not by as much as you might think: the end product of many experimental papers boils down to just such a plain table of numbers.)

    What we see here is that B appears to go as the square of A, which I will write A^2. Now when we go to publish this data we _won’t_ announce that we’ve discovered “the great Square Law between A and B”. We won’t even say that “we’ve observed that B = A^2”. What we will say is more like “over the range that we’ve measured them, and within our experimental uncertainties, the relationship B=A^2 describes our data.” (Even better form would be to say that “the relationship B=A^2 cannot be ruled out by our data”.) And that’s it. We go right up to the edge of the strongest statement that can be justified by our data, and not one step further. We will, literally, spend months re-wording a four-page paper, trimming and sanding down sentences to ensure that we don’t make even one claim that doesn’t follow directly from the data — and then our referees will spend more months double-checking us on that. The tangible record of our profession, experimental papers in refereed journals, are not claims of absolute knowledge but almost the reverse: exercises in controlled humility and restraint about what we can possibly know.

    4. We know just how much of a myth is the “disinterested observer”. I can testify, with great confidence, that experimental physicists do not practice controlled humility in reporting results because they tend to be humble people themselves. We all have personal ambitions, and we are vividly aware of a host of pressures from funding agencies and departments, as well as from various fads and fashions. Every one of us would like to become famous for making some great discovery. But part of becoming famous is having other people duplicate your results, and so we know that if we fake the data then the cross-check will _not_ verify them and our cause would be lost. Even in the much subtler cases, we know from experience that if you over-reach and claim what your data do not support there’s a very good chance that your over-claim will be revealed as false in the next round of measurements. So we are motivated to stick to the practice of careful restraint, claiming only what the data will support and no more, not (only) out of philosophical dedication but (also) just out of plain self-interest.

    In sum, appreciating the limits of knowledge is not just a nice philosophical principle to which we pay lip-service, like flossing every day; it’s the basic stuff of our day-to-day work. So after reading your explanation of Derrida. et.al. above — to the extent that I understand it — what’s remarkable is just how … unremarkable the whole thing seems.

    OK, I hope you find this (more) helpful.
    Regards,

    Paul

  75. Gavin

    Janet asks

    I really would like to know whether or not you can see anything in the connection postmoderns make between totalizing worldviews (whether scientistic or religiously literalist) and our deep social polarization, or not?

    I can’t answer because I don’t know what these words mean: totalizing, worldview, and scientistic. I do realize that when I teach that the universe is 13.7 billion years old while the local pastor teaches that the universe is 6,000 years old, and that anyone who teaches otherwise is an ignorant, untrustworthy, amoral, atheist fool who jeopardizes the salvation of our children and future generations, the result is social polarization. I still must teach that the universe is 13.7 billion years old.

    Or that holding a discipline as a monolithic explanation of everything sends a message that is dangerously opposed to the kind of liberally educated mind we are trying to foster in our students. A mind that can work thoughtfully with gray instead of in one kind of black-and-white, and resists authoritarianisms and totalizations of all kind. And bring many perspectives to complex problems.

    Let me clearly state my position on this. I think that science gets things right. However, there are many issues currently unaddressed by science. If they seem interested, I try to help people understand why certain ideas conflict with science, and should therefore be discarded. I generally avoid saying much about ideas that are not addressed by science, leaving it to the experts in other fields.

    Does this send “a message that is dangerously opposed to the sort of liberally educated mind we are trying to foster in our students”?

    Now I have a question. I have in my hand a copy of Shakespeare’s “Hamlet.” What do you think of the statement “this book is nothing but paper and ink”? The scientist thinks this statement is obviously true, while I suspect that the English professor finds it obviously false. But the reason that they disagree is not because they see the book differently, but because they see the statement differently. Certainly removing all of the paper and all of the ink from the book would leave nothing, so the book is nothing but paper and ink. However, knowing the properties of paper and ink will never reveal Hamlet’s tragic trajectory, so there is something more to the book than paper and ink.

    When I say “the only reality is scientific reality” I am speaking in the language of “this book is only paper and ink.” I recognize that the reality which we understand at a rather basic level leads to complicated structures and amazing events that are currently beyond scientific understanding. None the less, I know that those structures cannot defy and events cannot contradict the basic understanding. I think this is what most scientists mean.

  76. David

    Now back to quantum measurement

    Let’s go follow Gavin somewhat closely. He asserts that quantum mechanics itself describes the working of the apparatus and how it gets entangled with the system being measured. This is certainly true. What I disagree is with his statement that this makes everything deterministic. There is always a probabilistic nondeterministic aspect as far as the information we have.

    Let’s follow Gavin’s example we have system in the superposition

    |a ) + |b )

    so that the probability of finding the system in a is 1/2 and finding the system in b is 1/2.

    Like Gavin let us suppose the measuring apparatus can be in | ? ), |A ) or | B )

    The system plus apparatus is initially in the state

    ( |a ) + |b ) ) | ? )

    after fully quantum evolution which is completely deterministic a Gavin mentioned the system plus apparatus is in the state

    |a ) | A ) + |b ) | B )

    A measurement has been made. When we look at the apparatus we see either A or B
    with 50 % probability, we never see both. In here if you believe in the many worlds interpretation one says the system is in state

    |a ) | A ) + |b ) | B )

    we only feel |a ) | A ) or |b ) | B ), but not both we say we are in one of the branches. Nothing tells us which branch we are going to end up in. But if we do the measurement many times 1/2 of the time we end up in A and 1/2 in B. Even though the workings of the apparatus and how it gets entangled with the system is completely deterministic through quantum Hamiltonian evolution, there is always a nondeterministic aspect as one cannot predict whether one sees A or B, but we can predict the probability to end up in A or B.

    In this sense many worlds is not different than the collapse postulate. To say we end up in branch A, B is there but we cannot access it, is no different that say the wavefunction collapsed and is now |a ) | A ).

    The point, is many worlds interpretation or collapse postulate there is always a nondeterministic probabilistic aspect. For the philosophers as far as anyone knows this aspect has nothing to do with free will, self determination, consciousness or anything along those lines.

  77. David

    And Janet I do believe that science teachers nor anyone should should belittle religious belief.

    Although if religious belief gets in the way of reasonable evidence, which unfortunately lots of time it does, that should be attacked, because it will interfere with progress and advancement, not too mention how incredibly closed minded it is.

    Example, if a kid in class says that the earth is 6000 years old and it is that old even if there is considerable evidence to the contrary because the bible says so, if the teacher, respectfully, but strongly does not try to explain how nonsensical that is, how counterproductive that attitude is, the teacher is certainly not a good one. Of course people have a right to believe it, but is not true and the attitude of denial in the phase of evidence is unreasonable and should be explained and pointed out. People could take this as derision, but the alternative is worse.

  78. HI

    Janet,

    Scientists appreciate uncertainties more than you think as Paul illustrated above.

    I also want to emphasize that not all uncertainties are same. They come from different sources and come with different magnitudes. They are not black and white, but are different shades of gray. Scientists are accustomed to weigh them. A problem I have is when someone points at light gray and declare it is black (because it’s not white). It is a binary thinking and, I dare say, dishonest.

    As you like to judge science not just by its ability to describe nature, but also by its influence on society and culture, it is only fair to judge postmodernism by its influence outside the postmodernist circle. A disturbing thing is that the emphasis on uncertainty is hijacked by those who benefit by discrediting scientific claims, such as global warming.

    Read the following:
    http://scienceblogs.com/intersection/2006/04/the_science_wars_are_over_long.php

    The article above cites a reaction by the French sociologist of science Bruno Latour when his rhetoric was hijacked by a global warming denialist.
    http://criticalinquiry.uchicago.edu/issues/v30/30n2.Latour.html

    I have to remind you that at the Intelligent Ddesign trial in Dover, Pennsylvania, a postmodernist was on the witness stand defending Intelligent Design.

    It is also worth noting how postmodernism and postmodernists are viewed by non-postmodernists, but non-scientists. I recently finished reading “Reading Lolita in Tehran.” I will quote a passage.

    “She said, Of course, these books aren’t fashionable these days. Everyone has gone postmodern. They can’t even read the text in the original–they’re so dependent on some pseudo-philosopher to tell them what it says. I told her not to worry, that nobody taught James anymore either, that he too was unfashionable, which was a sign that we must be doing something right.”

    What postmodernism in Iran was like is by itself an interesting question. But I want you to note that, to these woman living under an oppressive regime, postmodernism was not a breeze of fresh air you described. Rather, it was another authority that marginalize them and they see it as only fashionable at the time.

    And your own student admits her frustration with deconstructionists. Why we scientists should be happy about it. The burden is on you to demonstrate it is as wonderful as you claim.

  79. HI

    Janet,

    Scientists appreciate uncertainties more than you think as Paul illustrated above.

    I also want to emphasize that not all uncertainties are same. They come from different sources and come with different magnitudes. They are not black and white, but are different shades of gray. Scientists are accustomed to weigh them. A problem I have is when someone points at light gray and declare it is black (because it’s not white). It is a binary thinking and, I dare say, dishonest.

    As you like to judge science not just by its ability to describe nature, but also by its influence on society and culture, it is only fair to judge postmodernism by its influence outside the postmodernist circle. A disturbing thing is that the emphasis on uncertainty is hijacked by those who benefit by discrediting scientific claims, such as global warming.

    Read the following:
    h ttp://scienceblogs.com/intersection/2006/04/the_science_wars_are_over_long.php

    The article above cites a reaction by the French sociologist of science Bruno Latour when his rhetoric was hijacked by a global warming denialist.
    h ttp://criticalinquiry.uchicago.edu/issues/v30/30n2.Latour.html
    (It seems I cannot post with proper URLs in a comment.)

    I have to remind you that at the Intelligent Design trial in Dover, Pennsylvania, a postmodernist was on the witness stand defending Intelligent Design.

    It is also worth noting how postmodernism and postmodernists are viewed by non-postmodernists, but non-scientists. I recently finished reading “Reading Lolita in Tehran.” I will quote a passage.

    “She said, Of course, these books aren’t fashionable these days. Everyone has gone postmodern. They can’t even read the text in the original–they’re so dependent on some pseudo-philosopher to tell them what it says. I told her not to worry, that nobody taught James anymore either, that he too was unfashionable, which was a sign that we must be doing something right.”

    What postmodernism in Iran was like is by itself an interesting question. But I want you to note that, to these woman living under an oppressive regime, postmodernism was not a breeze of fresh air you described. Rather, it was another authority that marginalize them and they see it as only fashionable at the time.

    And your own student admits her frustration with deconstructionists. Why we scientists should be happy about it. The burden is on you to demonstrate it is as wonderful as you claim.

  80. HI

    Janet,

    Scientists appreciate uncertainties more than you think as Paul illustrated above.

    I also want to emphasize that not all uncertainties are same. They come from different sources and come with different magnitudes. They are not black and white, but are different shades of gray. Scientists are accustomed to weigh them. A problem I have is when someone points at light gray and declare it is black (because it’s not white). It is a binary thinking and, I dare say, dishonest.

    As you like to judge science not just by its ability to describe nature, but also by its influence on society and culture, it is only fair to judge postmodernism by its influence outside the postmodernist circle. A disturbing thing is that the emphasis on uncertainty is hijacked by those who benefit by discrediting scientific claims, such as global warming.

    The following is a reaction by the French sociologist of science Bruno Latour when his rhetoric was hijacked by a global warming denialist:
    http://criticalinquiry.uchicago.edu/issues/v30/30n2.Latour.html

    I have to remind you that at the Intelligent Design trial in Dover, Pennsylvania, a postmodernist was on the witness stand defending Intelligent Design.

    It is also worth noting how postmodernism and postmodernists are viewed by non-postmodernists, but non-scientists. I recently finished reading “Reading Lolita in Tehran.” I will quote a passage.

    “She said, Of course, these books aren’t fashionable these days. Everyone has gone postmodern. They can’t even read the text in the original–they’re so dependent on some pseudo-philosopher to tell them what it says. I told her not to worry, that nobody taught James anymore either, that he too was unfashionable, which was a sign that we must be doing something right.”

    What postmodernism in Iran was like is by itself an interesting question. But I want you to note that, to these woman living under an oppressive regime, postmodernism was not a breeze of fresh air you described. Rather, it was another authority that marginalize them and they see it as only fashionable at the time.

    And your own student admits her frustration with deconstructionists. Why we scientists should be happy about it. The burden is on you to demonstrate it is as wonderful as you claim.

  81. HI

    Janet,

    Scientists appreciate uncertainties more than you think as Paul illustrated above.

    I also want to emphasize that not all uncertainties are same. They come from different sources and come with different magnitudes. They are not black and white, but are different shades of gray. Scientists are accustomed to weigh them. A problem I have is when someone points at light gray and declare it is black (because it’s not white). It is a binary thinking and, I dare say, dishonest.

    As you like to judge science not just by its ability to describe nature, but also by its influence on society and culture, it is only fair to judge postmodernism by its influence outside the postmodernist circle. A disturbing thing is that the emphasis on uncertainty is hijacked by those who benefit by discrediting scientific claims, such as global warming.

    The following is a reaction by the French sociologist of science Bruno Latour when his rhetoric was hijacked by a global warming denialist:
    h ttp://criticalinquiry.uchicago.edu/issues/v30/30n2.Latour.html

    I have to remind you that at the Intelligent Design trial in Dover, Pennsylvania, a postmodernist was on the witness stand defending Intelligent Design.

    It is also worth noting how postmodernism and postmodernists are viewed by non-postmodernists, but non-scientists. I recently finished reading “Reading Lolita in Tehran.” I will quote a passage.

    “She said, Of course, these books aren’t fashionable these days. Everyone has gone postmodern. They can’t even read the text in the original–they’re so dependent on some pseudo-philosopher to tell them what it says. I told her not to worry, that nobody taught James anymore either, that he too was unfashionable, which was a sign that we must be doing something right.”

    What postmodernism in Iran was like is by itself an interesting question. But I want you to note that, to these woman living under an oppressive regime, postmodernism was not a breeze of fresh air you described. Rather, it was another authority that marginalize them and they see it as only fashionable at the time.

    And your own student admits her frustration with deconstructionists. Why we scientists should be happy about it. The burden is on you to demonstrate it is as wonderful as you claim.

  82. Hi folks. I was away staying with a friend recovering from surgery, but now I’m back. I’m happy to hear about the book Paul recommends and I will get it right away and read it.

    Paul’s four comments are a welcome litany of acute perceptions about what scientists do and how he (at least) views what they do. Yes, given the things you say, Paul, Derrida is not very revolutionary, which is exactly how it should be! In my terms, according to what you write above, you are already a post-modern scientist, exactly the kind who would/does represent science humbly and responsibly within the liberal arts setting.

    But Paul, how typical, I wonder, is what you are saying? I know that very thoughtful scientists have self-selected themselves onto my weblog, for which I am very grateful and appreciative. but I’m not at all sure that even they would agree with you. (I would be fascinated to know.)

    I don’t know if you have read my earlier posts about my dismay over the dogmatic and “totalizing” claims I found on the science blogs — not from everyone, to be sure, but from many, many scientists.

    Once again, I find myself in an eerily similar position to what I hear from “the other side.” (As Hi always points out!) When I try to defend and explain poststructuralism and also that thoughtful religious faith does exist, I am told I am just an exception and that I should look around and see the religious fanaticism that characterizes Christianity in our society. And now, I find myself wanting to say to you, Paul, that you are “just an exception,” and you should look around you at the scientific fanaticism that characterizes scientists in our society!

    Well, it just shows that it’s a good thing we are talking about all this!

    The term “scientism” comes in very usefully alongside of “science.” It helps us sort things out a little bit, I think.

    “Science” is the pure doing and thinking of science, as it is directed to the subject matter of the discipline. This is what Gavin and David are always talking about. But “Scientism” is a metaphysical faith position that “believes” that science is the only way to find truth — that all truth is scientific truth — and that science is marching infallibly step by step toward a final and absolute account of everything that matters. In other words, that science is the new religion of the Modern West.

    If you will accept the findings of cultural historians, it is uncontroversial in our fields that scientism did become the dominant metaphysics or faith position of the West during the Enlightenment and the 19th century. If you read 17th, 18th, and 19th century texts, you will find real science and you will also find ardent “scientism,” side by side. This is what I mean when I say that science did get turned into a worldview and the rest of us lived inside of it.

    Now Rick, I take your word for it, and so I don’t think you do live inside of that worldview, when you do your science. You’ve shifted culturally while doing science and perhaps you are not fully aware of size of the shift from the historical past. But growing up as a little girl in the 50s in a university household, scientism was everywhere around me.

    I wanted to do math and history, but “girls” were actively discouraged from grad school in those fields. My mother and all the other mothers boiled glass battles and rubber nipples in steamy kitchens because breastfeeding was less hygenic and scientific than formula. (The Hershey co was telling mothers in Africa the same thing, but the water wasn’t safe for mixing with formula and this caused horrendous suffering.) “Father” always knew best because father thought objectively while women were emotional and subjective. I mean, I could go on and on….

    I’m glad the culture has “lightened up” and changed. This “deconstruction” of cultural “Modernity” (18th and 19th c’s and much of the 20th) is in my mind liberating overall. But no cultural movement is all good (or all bad!), and everything we do has unforseen problems and exclusions built into it, according to cultural or semiotic theory. (We postmoderns can be kill-joys, too, I must admit.)

    So Gavin, I think it is vitally important to deal with what scientism and worldview and “totalizing” mean. A totalizing claim is one that claims the truth of a certain position or way of knowing or authority IS TOTAL, it is all-encompassing and ultimately infallible and autonomous — it needs no input from any other source. You have to be a fundamentalist of some kind to think this way.

    The 19th century was the paradigm for totalizing claims! The scientific West was superior and deserved to colonize the world. “Progress” was inevitable and the West was its pinnacle. In science, the grand unified schema of scientific knowledge was almost complete: just a few small anomalies in electromagnetism and black body radiation remained to be cleared up….

    Instead, in the new century, we had revolutions in science, linguistics, anthropology, logic, mathematics, politics, social structures, family structures, sexual identities — this has been the most unexpected, disturbing, exciting, thrilling, instructive, depressing, and challenging century yet in human history — and for the human mind.

    Why don’t we teach THIS stuff to the young instead of re-running our old, old high-modern culturally outdated debates? Partly because we have changed SO very much, sovery fast, that we are caught in a cultural backlash, and religious and scientifically minded people are grasping onto the old absolutist, totalizing definitions of scientific and religious authority for a rock in the midst of all the turmoil….

    But the 20th century did shake everything up. For one thing, scientific Western man was very unhappy and neurotic, especially in Europe. Science hadn’t ushered in peace and prosperity but instead we had Nazi Germany, two world wars, genocides by highly advanced and scientific nations, the rise of totalitarians and so on.

    In every field of study we discovered that the binary oppositions associated with the older absolutist scientific outlook or worldview — mind vs nature, freedom vs determinism, objective vs subjective, masculine vs feminine and free man vs slave, rationalism vs irrationality, hard science vs humanities and arts, and on and on — all these were unworkably rigid dichotomies, and they were being broken down into much more adequate descriptions. We were finding much better ways to tackle these issues by moving into the middle ground methodologically….

    The Critique of Modernity that occurred during the last 100 years was intellectually brilliant. Michael Polanyi wrote his critique of objectivism in science — Personal Knowledge — at mid-century as a great physical chemist and a philosopher who understood and affirmed science but broke down the myths about science along the lines that Paul’s comments elucidate. (But when I read Weinberg, it’s as if Polanyi never wrote at all.)

    I had assumed that in our postmodern day the “armed fortress mentality” typical of high modernity had diminished, and instead it has seemed to me — to my alarm and chagrin — that it is worse than ever these days. (See my Jerry-Springer-show-on-science-blogs post, for instance.)

    Okay, here are some of Paul’s offerings. Do we agree with them? How do we assert the comprehensiveness and objectivity (or experimental verification) features of science without overstating these aspects?

    Paul says:

    1) “We are intimately familiar with the concept that our perception of reality is mediated. … our real expert, trained ability is in carrying out the cross-checks, and when they don’t agree finding the most efficient way to design new, further cross-checks and isolate which part of the chain is not in accord with what we thought it was. Appreciating the irreducible uncertainty in our knowledge is, literally, our day-to-day work.”

    2) “We are familiar with the idea that our vocabulary limits what we can talk about, and even what we can think about. In the above example I might report the value of a magnetic field’s strength without complete reference to how it was measured. This is a useful shorthand, but I realize that it only makes sense within the Faraday-Maxwell vocabulary of magnetic field strength as a flux density in space.”

    3) “We will, literally, spend months … trimming and sanding down sentences to ensure that we don’t make even one claim that doesn’t follow directly from the data — and then our referees will spend more months double-checking us on that. The tangible record of our profession, experimental papers in refereed journals, are not claims of absolute knowledge but almost the reverse: exercises in controlled humility and restraint about what we can possibly know.”

    4) “We know just how much of a myth is the “disinterested observer”. …
    In sum, appreciating the limits of knowledge is not just a nice philosophical principle to which we pay lip-service, like flossing every day; it’s the basic stuff of our day-to-day work.”
    ———————————

    Janet here again:

    Scientists, is this how you think and work?

    Humanists, does this describe the scientists you’ve worked with?

    P. S. Speaking for myself, YES, this describes the colleagues in science I taught with for 25 years and the ones I worship with in my Episcopal parish and in our sister congregation of Greek Orthodox (both filled with physicists, btw). But then I go onto science blogs and I read Richard Dawkins’s crusade against religion filled (it seems to me) with totalizing claims and scientism….

  83. Gavin!!

    “When I say “the only reality is scientific reality” I am speaking in the language of “this book is only paper and ink.” ”

    CAN YOU SEE THIS *MIGHT* SEND THE WRONG KIND OF IMPRESSION!!I

    Yours truly,

    “The Humanist Around the Corner”

  84. David

    Janet, you are making all sorts of connections and inferences that I do not know if they are true.

    Most of what Paul said is the way a lot of scientists thinks. Because of all the cross checks, etc. is why we are very sure of what we state, and why we state it strongly. This does not mean it is absoltely true. But it means it has a lot of evidence in its favor. When someone challenges it, there is nothing wrong wit hit, but the challenge better present errors in the evidence and inferences from the evidence or present at least as much evidence for their point of view, And that evidence should be able to be cross checked and reproduced.

  85. David

    And I am very much baffled. How exactly did we get into this rants about totalizing claims?
    We were trying to talk about quantum measurement and other such things. How come Dawkins keeps being mentioned? He has no bone in this fight. He has never said anything insightful about quantum measurement as far as I know

  86. David,
    I am very much baffled, too.

    First of all, this isn’t a fight, and I wasn’t “ranting.” I can’t see why you are not following me.

    Maybe someone else can help us out?

  87. It seems to me, to use Gavin’s analogy, that the QM explicators here keep saying, you don’t understand, Janet, the book is ink and paper and we are making solid progress with it. And so I engage with you on those terms, because I love science as science, and I find that the science of the ink and paper on the quantum level is beautiful and inspiring. And I find places where I misunderstood it and I say so.

    But when I say, okay now, this adventure into the structure of the ink and paper can also be read as text that tells its own unique story and has wider meanings, you say, No, it’s just ink and paper. There is NO CONNECTION between our intricate researches into the complexities of the sub-atomic structures of the ink and paper and any of the speculative questions studied by philosophy or history or culture or theory. (Remember the questions I asked, which Hi at least answered, thank goodness.)

    ??? How do we keep this a two-way conversation? At any rate, please, every time I try to explain the extra-scientific impacts of science, don’t tell me that I am ranting or attacking or misunderstanding science, okay? Give me the benefit of the doubt, that I am trying to get something of importance across, and I am not attempting to deny the continuity and credibility of science.

    It is fundamental to conversation that we don’t get to bracket out as irrelevant or unreasonable what the other conversation partner is saying, just because that isn’t how we have always seen things. The whole point is to see that perhaps things as we always see them can be interpreted validly in another dimension from the one we have always seen them in, up until now.

    I can assure you that when I wrote the comment David responded too above, I was thinking, Great, now at last they’ll understand what I’m getting at, the difference between the scientific humility that Paul articulates and the scientism that historically has afflicted Western culture…. Guess not! : )

    After all, I have freely admitted that PoMo often has been a trendy bandwagon in the US and that there has been shoddy thinking in it, while continuing to defend the rigor and importance of the best poststructuralist analysis. Why can’t you folks admit that something like scientism historically accompanied science in the West and that some scientists today make a militant religion out of their scientific rationalism? I ask this gentlely and calmly and without a shade of rancor. Mild as a veritable lamb.

    I cannot clarify poststructuralism unless you are willing to consider a structural impact of science upon society, evolving differently at various points in our history. (And to accept the noncontroversial findings of cultural history and history of science.)

    So perhaps we are, after all, arriving close to the heart of the problem of communication between “the two camps.” You folks are deeply involved in the brilliant pursuit of exploring and cross-checking the physical evidence with maths and numbers — and we are deeply engaged with interpretation of texts in terms of their deep patterns and structures of meaning.

    Our evidence is the texts of historical periods and we thrash out with one another the best theories to analyze the meanings and the most prominent cultural patterns that emerge. Our methods teach us to connect and synthesize the evidence from many different areas across various divides, into something like “force fields” of cultural meaning that don’t even have any physical existence if appraoched as physical reality, They are semiotic and perceptual realities.

    Maybe you should view what we do as a kind of artform! I don’t know how to help you see the value of it. “Hamlet” has to be experienced, I suppose. Well, sometimes you ask me to convey my discipline more clearly or simply. I have been enacting my discipline with you. Don’t mistake its passion for rancor or attack.

    I do not want to accept that we are inevitably locked into our separate camps, as though some of us are from Mars and some of us are from Venus, so to speak. I know we share the common desire to liberally educate young minds, and I think we also share the view that we need a pluralism of arts and sciences to do this. When I get time, I will type for you-all some things Roger Penrose has to say about the measurement problem that might be suggestive for discussion.

    But please read over the things I’ve been saying and see why of course I use Dawkins — not in his excellent science popularizations over the years but in his recent vendetta against religion — as an illustrative example of the problem I am concerned about the most. That in science and in religion we mustn’t get swept away into militant and fundamentalist attitudes. The reason this is related to QM and Relativity is that historically speaking they helped end the reign of Newtonian “scientism,” which yes, I know, I know, doesn’t really have anything to do with Newton at all!!

    Over and out.

  88. David

    When you say?

    this adventure into the structure of the ink and paper can also be read as text that tells its own unique story and has wider meanings, you say, No, it’s just ink and paper. There is NO CONNECTION between our intricate researches into the complexities of the sub-atomic structures of the ink and paper and any of the speculative questions studied by philosophy or history or culture or theory

    Yes, it gives me trouble. Because You can make those connections beyond the ink and paper. But those connections are analogies and can perhaps inform, shape, inspire, instill new ideas into speculative (and perhaps nonspeculative) questions of philosphy and culture. This they can do and it is fruitful I hope. But I think we have semantic problems because this is not a connection.

    To put it more simply, one can make analogies between quantum uncertainty and uncertainties in the real world. But they are very different things. Moreover I believe that since real world uncertainties are far more complex, richer, and of far more consequence than the uncertainties of quantum mechanics. Since the real world uncertainties really are very different than the ones in quantum mechanics, too close a connection will lead to a far too simple, possibly erroneous, but for sure incomplete and weak progress as far as attempting to grasp and deal with real world uncertainties.

    So beyond analogies and inspiration I believe there is little connection. Now, when gathering evidence, when cross checking evidence, and when determining the likelyhood that something occurs, I believe that not specific theories, but the general philosophy and framework of experiments can inform and will have a connection. After all experiment is a very sophisticated and effective detective work.

    So the problem is not that I feel you are attacking science. It’s that we get all this big connections that are nit only not there, but that if you make the connections and keep to them, they will not be liberating because they wil impose too much restrictions, that are not there, in the problems you care about. So I don’t think we are in two camps. And it is wonderful we talk to each other. It is great that we try to understand each other. But too direct a connection between some specific theories of science can only retard knowledge.

    Let me go back to the ink and paper analogy and Shakespeare. Let’s take Hamlet. Hamlet like all books is a collection of ink and paper. Ok, it is also a wonderful piece of literature with profound insights into human nature and interactions and also a very entertaining work. Now how ink gets absorbed on paper is a physical process. Understanding all of this physics is an intellectually challenging exercise, you need a good mind an dedication to understand how the theory works and to follow the experimental evidence to see if we buy that the theory is true then use it to predict new things. If I do all this and set out write not Hamlet, cuase that is too tall a task, but a not bad play, I ail completely. I need instead of studying how ink goes in to paper and the physics that goes into, I need to get immersed in literature, but also in life with a keen observation to human weakness, strength, interactions to even have a chance of writing a not completely crappy play. I might still fail if I do this, not everyone has talent for writing, but if to write a play I would have made sure I know the science of ink on paper I would for sure fail even if it is true that a play is ink on paper.

    Quantum Mechanics is like the ink on paper. The real world is Hamlet. Sure it is ink and paper, but it si much more and we will miss that much more if we try to understand it
    only through in kand paper. Making too direct and close a connection is not good because you are selling the wonderfully complex real world problems you care about short! This is what bother me and while I think the connections are only loose, indirect at best, and inspirational.

    You see where I am coming from?

    And I agree that fundamentalism and militantism are to be a avoided. Dawkins is very smart, but I think he is as religious in his irreligiosity as fundamentalists from many religions. On the other hand because he is smart, despite this he’s worth a careful read more than other fundamentalists even if I disagree with his fundamentalism.

  89. David

    Also, I have no problem admitting there has been scientism and that some scientist make rationalism into a militant religion. The reason I usually ignore comments to that effect is that they are not relevant to what science is and how science gets done which is my main passion are concern.

    These concerns of scientism and militancy are of course, extremely important as far as the way people perceive science and how people react to science. But, in this context connections with QM or other things are at the level of analogies and interpretation which can be useful, but cannot be too direct or explicit because the real world cultural problem is far more complex and far less restrictive.

    A separate problem is that nature is rational to a large extent. Via experimentation, inference, construction of theories, prediction, building of successful technology, we have understood an controlled a large part of nature. Of course we do no understand or control everything. But I have had instances of being accused of militant rationalism just for asserting this true facts and it had seemed to me that the only way to get the other person to think I wasn’t militant was to deny this true facts. This I didn’t do because
    I do not see what can be gained by being intellectually dishonest. And Janet, I know you
    are not one of those persons. But this type of thing does occur even among very good and smart people in your field, just like in my field some very good and smart people say and do all sorts of things that they shouldn’t to other people.

    And Janet, I shouldn’t have said rant. I wasn’t trying to pick a fight nor did I think this was a fight. I speak (type) strongly and carelessly.

    Now my bafflement was genuine because I keep things like QM measurement and the concerns you bring at a certain distance from each other for the reasons I’ve tried to explain.

    Now on a separate subject, you mentioned you might post some of Penrose comments
    on the QM measurement problem. I will start with a attack on some of his ideas. What can I say? I’m aggressive at times. Aggressive but not mean spirited I promise.

    Penrose has been thinking and advocating that wavefunction collapse has something to do with gravity. This I find highly unlikely and possibly not true. He makes good arguments for it, yes, but I don’t buy them. Now am I absolutely sure Penrose is full of it with this type of thinking? No, of course not because we do not fully understand quantum measurement. But wavefunction collapse, or if you don’t like collapse but like many worlds, the bifurcation into one branch occurs in many measurements where gravity plays absolutely no role, or if even if negligible one decides to include the gravitational potential in the quantum Hamiltonian, it acts on the system and measurement apparatus in ways unconnected to the measurement process. This is why despite Penrose’s very clever arguments as to why gravity has the solution for wavefunction collapse, I think in this respect he’s full of it.

    Now Janet, I really hope the comments you were going to post were on Penrose’s collapse of the wavefunction via gravity as I’ve jumped the gun and fired first.

  90. David says: “Sure it is ink and paper, but it is much more and we will miss that much more if we try to understand it
    only through ink and paper. Making too direct and close a connection is not good because you are selling the wonderfully complex real world problems you care about short! This is what bother me and while I think the connections are only loose, indirect at best, and inspirational.

    You see where I am coming from?”

    Yes, I do! This is very, very helpful to me. And I appreciate your remarks about fundamentalistic scientism very much as well. Gotta run for now. I think I was going to use Penrose more to explain some of what you guys said for humanists to understand better, and then ask about the connections (or not) of QM and the human world.

  91. HI

    Janet,

    I tried to post a comment yesterday morning, but somehow my comment never appeared. I will post it again if I can post this comment successfully.

    I think you were completely misreading Gavin’s comment on a book. David’s comments may have clarified it, but I would like to reiterate. The statement “This book is nothing but paper and ink.” is an appropriate description of the book, as far as what physically constitutes a book. It doesn’t have to be mutually exclusive with a statement like “This book is a great work by Shakespeare.” because these are description of the book at different levels. It is somewhat analogous to a difference between describing liquids as collections of molecules buzzing about and as macroscopic entities that have certain densities, viscosities, and so on. It depends on what kind of scale we are using. Do you think we, the unsophisticated scientists, cannot see things at different levels? (If anything, this is the reason we scientists get annoyed when QM is used to describe uncertainties in everyday events. This is like using paper and ink description to argue about the merit of the works of Shakespeare.)

    Regarding science and scientism, I think your arguments are not entirely consistent. In my very first post here, I objected to your characterization of science. You replied me describing the distinction between science and scientism and your only problem is only with the scientism. But when I objected again to what I perceived as your mischaracterization of science or your confusion of science and scientism, you replied me again saying something to the effect that science and sceintism cannot be separated. So, which is it?

  92. HI

    This was a comment I tried to post yesterday.

    Janet,

    Scientists appreciate uncertainties more than you think as Paul illustrated above. (So, to answer Janet’s subsequent question, this is how scientists work.)

    But I also want to emphasize that not all uncertainties are the same. They come from different sources and come with different magnitudes. They are not black and white, but are different shades of gray. Scientists are accustomed to weigh them. A problem I have is when someone points at light gray and declare it is black (because it’s not white). It is a binary thinking and, I dare say, dishonest.

    As you like to judge science not just by its ability to describe nature, but also by its influence on society and culture, it is only fair to judge postmodernism by its influence outside the postmodernist circle. A disturbing thing is that the emphasis on uncertainty is hijacked by those who benefit by discrediting scientific claims, such as global warming.

    Read the following post by the journalist Chris Mooney, the author of “The Republican War on Science”:
    http://scienceblogs.com/intersection/2006/04/the_science_wars_are_over_long.php

    Read also the essay cited by Mooney, which was written by the French sociologist of science Bruno Latour when his rhetoric was hijacked by a global warming denialist:
    http://criticalinquiry.uchicago.edu/issues/v30/30n2.Latour.html

    I have to remind you that at the Intelligent Design trial in Dover, Pennsylvania, a postmodernist was on the witness stand defending Intelligent Design/Creationism.

    It is also worth noting how postmodernism and postmodernists are viewed by non-postmodernists, but non-scientists. I recently finished reading “Reading Lolita in Tehran.” I will quote a passage.

    “She said, Of course, these books aren’t fashionable these days. Everyone has gone postmodern. They can’t even read the text in the original–they’re so dependent on some pseudo-philosopher to tell them what it says. I told her not to worry, that nobody taught James anymore either, that he too was unfashionable, which was a sign that we must be doing something right.”

    What postmodernism in Iran was like is by itself an interesting question. But I want you to note that, to these woman living under an oppressive regime, postmodernism was not a breeze of fresh air you described. Rather, it was another authority that marginalize them and they see it as only fashionable at the time.

  93. Hi, I am so sorry your comment did not come through yesterday. I never saw any sign of it. And I was hoping to get your perspective on where we are at now in the discussion.

    This second comment of yours above I especially think makes excellent points! When I have a bit more time I’ll say more. Reading Lolita in Tehran is one of the best books I have ever read and I’ve recommended it to everyone I know.

    As for the previous comment, it is hard to separate science and scientism! But it is utterly essential, it seems to me. That’s what I’m asking you-all for help with!

    I’ve spent a life-time separating out my faith from fundamentalism and it takes a lot of work. But there is a lot of great theology and churches out there to help us. Reading Lesslie Newbigin’s call to separate your faith from your cultural assumptions had a big impact on me, especially his marvelous little book, Foolishness to the Greeks: The Gospel and Western Culture, where he explains quite clearly how biblical literalism actually is very much like scientism even though in reaction to scientism. This made so much sense to me and I’ve taught the book for a decade so I keep wanting to raise this point about the structural homology or isomorphism of biblical and scientistic fundamentalisms.

    Hi, you certainly don’t let me get away with anything, do you!? I have to assume you are getting something out of this discussion and I hope I don’t strike you as dishonest in general. I am trying to be honest, even when it hurts.

    As for the passage you quoted, what can I say? I have admitted the trendiness of American PoMo, which your quote refers to. On the other hand, cultural studies and lit theory is probably the strongest kind of reading of literature to get at what she writes about. Now that’s she’s here in the US it will be interesting to see what she thinks of theoretical developments. Maybe she still won’t like them.

    What did you think of the book? (It frightened me badly to see how easily totalitarianism came and well-educated women were put under the veil — could that happen here, I wondered? We live in scary times, but that is when we especially can’t let ourselves rely on stereotypes. I hope I am not pointing to black and calling it “light gray”! What a great image, by the way.)

    Given what I’ve learned about Intelligent Design (and PZ at Galactic Interactions has campaigned to educate folks on this) I wouldn’t expect to think very highly of their witnesses. I suppose it is possible that some sincere person got roped in, but they seem to be cynically politically dishonet to me in general. One of the worst things that’s happened to our country. Anyway, more thoughts later.

  94. Ooops, it’s PZ at Pharyngula! Galactic Interactions is Rob Knop. I got mixed up for a second. I think PZ’s work on ID is very important. And I get a lot out of Rob’s weblog too.

  95. Gavin

    I was gone for a couple days again. I sure get behind fast.

    Janet says

    “When I say “the only reality is scientific reality” I am speaking in the language of “this book is only paper and ink.” ”

    CAN YOU SEE THIS *MIGHT* SEND THE WRONG KIND OF IMPRESSION!!I

    Yes, I can. Could you suggest language that would communicate what I mean more clearly?

  96. Gavin

    Here are two things that frustrate the people who study how ink and paper interact.

    1) People who say that Hamlet and the other characters in “Hamlet” have eternally existing souls that drive their behavior and that when a copy of “Hamlet” is destroyed these souls are released to perform eternally in a production of “Hamlet” that is perfect in every way.

    The people who study paper and ink can explain why this idea is absurd. Janet has never made a claim like this about “Hamlet” or anything else that I have seen. But many religious people make claims similar to this that are equally absurd. I see no reason why scientists shouldn’t point out why these claims are wrong.

    However, Janet, I’m trying to determine if you think we are being polarizing and arrogant when we do this. Can we go to someone in another field and say, “I don’t know much about your field, but I want to point out that this one specific claim you make violates well established physical laws, so it can’t possibly be right”?

    2) The paper and ink people publish a paper announcing that they have discovered that ink diffuses slightly causing the characters to appear slightly fuzzy rather than sharp under a microscope. Some people who don’t understand paper and ink then announce that the recent discovery of diffusion reveals something very important about the characters in “Hamlet,” they need to shave and they aren’t as smart as they appear.

    Now it is probably true that the charters in “Hamlet” were not silky smooth by modern standards, and certainly the characters are not prone to thinking things through with the wisdom and foresight that might be expected from individuals of their standing. However, this has nothing to do with diffusion, obviously.

    Janet, you have made claims that at least look like this to me. However, I think we’ve already covered this issue and everybody understands the problem, even if we haven’t quite determined the solution.

    Here is what I think the moral is. The study of paper and ink reveals almost nothing about the characters in “Hamlet.” For example, it can tell you nothing about the characters grooming or intelligence. However, the few things that the study of paper and Ink do reveal are indisputable. For example, they are not conscious and do not have eternal souls.

    In most cases, when considering the claims made in other fields, we scientists have nothing to offer and should keep our mouths shut. But in rare cases, we have great authority (usually over only a very restricted set of claims) and in that case I think we should enter the discussion quite forcefully.

    Let me be the first to admit that scientists are prone to overreaching. Roger Penrose’s ridiculous ideas about the connections between wave function collapse and consciousness are a perfect example. The claim of any connection between these two things is shockingly stupid. It is a testament to Penrose’s intellect that he has strung together elements of diverse fields beyond his expertise (Godel’s theorem, Turring machines, bio-chemistry, cellular anatomy, and neurophysiology, to name only a few) into a argument capable of fooling one of the prominent scientific minds of our time, his own. He’s hardly alone: John Haglin, Fred Allen Wolfe, Paul Davis, Frank Tippler, and many others say things that simply make no sense. Richard Dawkin’s speculations about why people believe may be in this category as well. His claim that religion is bad is almost certainly an example of him speaking on a subject he is poorly prepared to address. He’s still welcome to his opinion, but since he is not a historian or a sociologist, I see no reason to take his opinion seriously.

    I also see situations where people say “scientists can’t say anything about X, only X-ologists understand X.” Examples include claims that the existence of eternal souls is somehow beyond the reach of science, or that understanding the age of the universe is not real science because it can’t be recreated. These positions are wrong. Science can address these narrow issues quite effectively, and has. When scientists do this, it is polarizing and at times hurtful, but it is probably still right.

    How do we recognize these distinctions? How do non-scientist know when scientist are speaking with authority or with arrogance? How do scientists respect other fields while not compromising when those fields make claims that can be shown scientifically to be false?

    I think much of the conflict comes from thinking that scientists either have authority over everything, or must keep totally out of areas claimed by other fields. It really depends on the details of the specific issue.

  97. Gavin

    Back to quantum measurement, finally.

    David,
    I like Copenhagen, but there is a big problem as far as I am concerned. In the Copenhagen interpretation the there is the regular deterministic evolution that happens between measurements, and the probabilistic collapse that happens when there is a measurement. I find this two rule sort of theory unsettling, but that is not the problem. The problem is this: I don’t know what a measurement is. How do I know what counts as a measurement and what is just regular, non-measuring interactions?

    I also disagree that the many worlds necessarily has the probabilistic feature. Let’s look at the following statement.

    “I prepared a state |a)+|b) and then measured it to be in state |a)”

    This is the sort of thing we experience all the time when we do measurements. However, it is my claim that this sentence makes no sense. (This is like the statement that two well separated events happened “at the same time.” It fits common sense, but is actually nonsense.) If you prepared a state |a)+|b) and then measured it, I already know what happened. You ended up in an entangled state. The claim that you got a particular result is wrong, even though I know that is the way it seems.

    Sometimes it is hard to see where this mistaken view slips in. For example, you say:

    In here if you believe in the many worlds interpretation one says the system is in state

    |a ) | A ) + |b ) | B )

    we only feel |a ) | A ) or |b ) | B ), but not both we say we are in one of the branches.

    But I think we do feel both |a)|A) and |b)|B). We seem to only feel one because it is an entangled state, but I don’t think we end up on a particular branch. We are on all of the branches experiencing all of the results. I find this unsettling and a complete violation of my common sense about the world based on my experiences, but I don’t see any way around it given what I know about quantum mechanics. If somebody could tell me what a measurement is, I might be converted.

  98. David

    Finally back to measurement Gavin.

    The same thing you find unsettling about Copenhagen .

    Let me put my point more directly.

    I see little difference between we feel only |a)|A) or only |b)|B) because of collapse and
    we feel both |a)|A) and |b)|B), but we seem to feel only one when we are entangled.

    Now, having said this I do not think we can settle this debate in the sense that I do not see a practical measurable difference between the two approaches: 1) collapse or 2) when we are entangled we feel all the alternatives but because we are entangled we seem to feel only one. Also in many worlds for experimentalists to check quantum mechanics one of the things we provide is the probabilities that we seem to be in |a)|A) only or in |b)|B) only even though we really feel both |a)|A) and |b)|B) because we are
    entangled

    I think many worlds is logical an well constructed even though I have deep worries (as deep as my worries about Copenhagen) about the part of seeming to be in one branch even though we really are in all.

    It looks to me that it’s a matter of choice which of the alternatives you take because regardless of which interpretation we use, it seems to not matter as far as performing theory, explaining or predicting experimental outcomes. Do you agree? Do you think I’m confused? I’m all ears.

    Also if you can think of a doable experiment that can tell the difference between the two approaches I think it would be something illustrative and very important. It is something I have thought about a lot, but always failed to come up with.

    Perhaps we have philosophical differences as to our approach to physics because I see little difference between it seems and it is despite that it might lead to some disturbing things. The way I check if the theories I work with are correct is by comparing to what seems to be true from experiment. It is true tha logic, consistency, simplicity are all criteria I use to guide my theoretical thinking but in the end theory acquires its authority by successful confrontation with existing and predicted experiments and not the other way around.

    Do not get me wrong, the fact that there are disturbing things means there are things we either have not figured out or discovered yet or things we do not understand in the knowledge we do have.

    Also the problem of collapse or the seeming branching in many worlds is the toughest part of quantum measurement. I think how things decohere and look a lot more classical
    because of this is in a lot more sure footing and illustrative of many aspects of measurement. I would love to hear your thoughts and perhaps examples on decoherence before I offer some of mine.

  99. Gavin

    David,

    I do think that experiment bears on this question, but only with tremendous extrapolation. Let’s say that you wanted to measure the spin of an electron using one other electron. You set things up so that the electron spins become entangled. In that case any experiment done on the first electron would show that its coherence terms were gone, which means it would appear that the wave function had collapsed. But I also think that it would be fairly straight forward to design an experiment involving both electrons that would show that coherence terms still exist for the entangled state, and therefore the wave function had not collapsed.

    When you measure with something bigger than another electron, it will be more delicate to show that the electron and the measuring particle have entered a mixed state and not a collapsed state. You could extrapolate that collapse never really happens it just gets harder to see.

    The other experimental issue is that the best way I know to determine whether a particular sort of interaction constitutes a measurement is to solve the Schroedinger equation for the interaction and see if it removes the coherences from the density operator of the measured system, which is what collapse would do. So the way to determine if you can use the measurement rule is to use the between-measurement rule and see if it gives the measurement rule answer. Then you can use the measurement rule, but why would you, you already got the answer from Schroedinger’s equation.

    What I would like in a theory is something that has two properties: 1) Is should not include a huge amount of stuff that is unobservable even in principle. 2) It should have a set of consistent, well defined rules that apply all the time. It turns out that with quantum mechanics you can have either, but not both. I value 2 more, so I put up with breaking 1. In general, I think that people who value 1 more are willing to ignore 2, but I’m putting words in your mouth now so correct me if you think this is the wrong way to look at the compromise.

    Everyone,

    Let me emphasize something that David has said. We are arguing something very fundamental about the workings of the universe. I say it is deterministic and includes a staggering number of equally real but totally unobservable branching possibilities. David is arguing that the real world is what you see, but that at a fundamental level it is not deterministic but is governed by probabilities. Could our views be more different?

    Yet on one thing we totally agree, this debate is of no consequence at all beyond the fun of wondering at our incredible universe. This debate about how things work at a fundamental level is like an argument over the ink, which has no bearing on the study of the play. The fundamental rules lead to the rules that apply at the larger scale, but they are not the same rules. We know how the world works on the scale of cells and brains and societies and nothing that we learn in a debate about quantum measurement effects any of that. If you are interested in those things, there is no reason for you to ever mention quantum mechanics except as an analogy (and since analogies are supposed to make things clear, and almost nobody understands quantum mechanics, I think even this is a very suspect rationale).

  100. David

    Gavin,

    I very much agree with your statement

    “What I would like in a theory is something that has two properties: 1) Is should not include a huge amount of stuff that is unobservable even in principle. 2) It should have a set of consistent, well defined rules that apply all the time. It turns out that with quantum mechanics you can have either, but not both. I value 2 more, so I put up with breaking 1. In general, I think that people who value 1 more are willing to ignore 2, but I’m putting words in your mouth now so correct me if you think this is the wrong way to look at the compromise.”

    but I am not 100% sure that I agree completely. First , I myself vacillate between the two views. I am also not sure you choose 2 because many worlds has all those unobservable branches. I am also not sure I choose 1. By thinking collapse I don’t see the problem as unobservability. The disturbing part, and it does disturb me plenty, is that the collapse rule is outside quantum mechanics. It is consistent as far as I can tell, but outside QM. It also has the large ambiguity of exactly when a measurement was performed, but for this ambiguity I do use some of your thinking and take the measurement to occur when the system becomes entangled with the “measuring” system. You can fairly call me out on this.

    Also, you are right that we agree on the consequences for the larger scale. But it’s more than that, we also agree about most of the workings of ink. But there is one very profound and confusing aspect of ink that we and a lot of other physicists have some disagreement over. And it is hell of interesting to talk about it, to see if we get to better understanding even if we do not get to a resolution. But how a lot of the aspects of ink work is fairly certain and tested and we agree on it. We would calculate the same cross section to be measured in accelerator experiments, we would calculate the same response functions like conductivities to be measured in materials experiments, we would calculate the same spectral lines and decay rates for atoms and molecules. Moreover the experiments would verify as they have done those calculations. Now when it comes to quantum measurement, what it really is, how to think about it, etc. we tie ourselves in knots and have some debate. In this debate and lack of consensus we have very good company!

    Now I hear you on your arguments about coherences. But like you mentioned before a lot of physicists pull their hairs with density matrices. I am one of them. I am not a big fan of them. Now I think that arguments about how coherences disappear can be made from thinking about and manipulating wavefunctions. I am not sure I can make it work, but I will think about it and if I figure out a way to illustrate coherences and how they disappear when the system becomes entangled by using wavefunctions and will post it today or tomorrow.

  101. Thank you! I wrote a whole post and a whole long comment but not on this, directly!

    I’m mulling this from you and Gavin all over. By the way, I use Penrose to try to understand what you two are talking about and not for his own personal theory of quantum gravity. He is an excellent explainer for non-scientists, I find. He is very honest about separating his own views from his discussions of all the ins and outs of the QM debate. I’m fascinated by what you are saying above.

    But I am even more fascinated by how Gavin can say that none of this has any bearing whatever on anything else. We are talking about the fundamental properties of the material universe on its deepst level!!!

    This has nothing to do with any other activity, even philosophy, which tries to understand the biggest questions? May I say again that philsophy is necessary because science takes its own sweet time and demands precision and the rest of us only have four score and ten years if we’re lucky to figure out the meaning of life. Philosophy isn’t a worse discispline than science because it is far less precise. It HAS to be. This comes from what it has to deal with and the far greater ranges of indeterminacy. But it’s out there fighting away to do its job.

    See, this is where I think you guys are so focused by your discipline that you rule out and abstract away from anything else. You can”t say, imho, that QM science has nothing to do with anything else in general. I think you should say QM has no connection with other fields and quests IN TERMS OF THE SCIENTIFIC THEORIZING ITSELF.

    Then if philosophy or cultural studies wants to come in they can be legitimate in their own right, so long as they understand the theories. Except that science often impacts culture most through MISUNDERSTANDINGS of its theories — and that needs to be studied too.

    This all does have a bearing on how much scientists do not want religion to come in and try to talk about natural processes such as the origins of the universe (scientifically speaking) or evolution or cognitive studies. Non-natural entitites and interventions here are opposed to science. But many, many scientists are believers of various kinds, remember.

    We have to be so “delicate” here. As delicate as with wave/particle questions. Faith and God are much larger things than simply a means to explain creation or the emergence of humanity. Science can show we don’t need these things for scientific explanation, and this is only quite clear at this stage in history, by the way. But talking to kids about how a 6000 year old earth is not scientific or reasonable when they do not have the skills to distinguish that from the rest of their faith — this gets very tricky indeed.

    We never said these problems would be easy.

    I am working on some Penrose-guided questions for you. I love the above additions to the explanations. Well, okay, here’s one.

    Penrose explains how the two-slit experiments and the beam-splitter experiment show us that the wavefunction must be taken holistically. It is not like classical waves in this sense. It discharges a single particle in one place only through a non-local wholeness.

    He also talks about density matrices and explains them as bundles of quantum excitations. Am I getting confused between the description of the wavefunction of a single particle and the state-descriptions of many particles, maybe? (What’s an eigenstate…. ) Don’t throw up your hands in despair, okay? I am trying….

  102. David

    A wave function might have a holistic interpretation. This does not mean it is not like a classical wave. The equation we solve to find what wavefunctions are and how they evolve behaves in exactly the same way as classical waves. It exhibits the same properties as classical waves. One of the main properties of all waves is the superposition principle. This superposition is what leaves to a lot of quantum “weirdness”. Superposing things we think cannot exist at the same time, but they do. Penrose I am sure understands this, so he must have meant something else.

    There are certain oscillations that are natural and stable in quantum systems. When a system is in a state corresponding to those oscillations it oscillates stably. These states corresponds for example to the spectral lines of atoms. This natural stable states of quantum systems are called eigenstates. Mathematically they have a more general meaning but I hope this is goof enough for a nonexpert and quantum systems.

    If my explanation is confusing, please tell me. It is me who failed in communcating clealry. Tell me and I’ll try again.

  103. David

    In the lab in can be verified and has been verified that classical waves do the exact same thing as quantum waves in the double slit experiment as long as we don’t measure which slit the particle went through. So this is not open to question. It is an experimental fact and not a one off event. It is verifiable and has been verified independently many times. Quantum waves are just like classical waves.

    Having said this, in the interest of complete openness I must say that if we measure which slit the particle went through they apparently do different things. This is the measurement problem. It could be that even in this case the whole particle plus measurement apparatus behave like classical waves and a consistent logical argument for this can be made like Gavin has made above via many worlds. I do not know if many worlds is the answer to the measurement problem but is is logical and consistent. I would say collapse is logical and consistent too despite some ambiguities.

  104. David

    Well, the fundamental properties of water molecules have almost nothing to do with the properties of masses of water for example waves in the ocean. In fact, one needs to know nothing about the fundamental properties of water molecules to study the behavior of masses of water like oceans.

    Superconductors are macroscopic quantum systems. All of them behave in the same universal way even though each fo them is made of different atoms with different microscopics interactions. So once again the basic properties do not say much for the behavior at large scales

    We have understood a lot about particle physics and the standard model even though we do not know the fundamental theory behind it. If we find that theory, the standard model will not be different, once again the behavior is independent of the more fundamental properties.

    in fact, it is the exception and not the rule that in nature things divide in hierarchies with interesting complex emergent behavior at different scales that is independent of the behavior at more fundamental levels.

  105. David

    The point is that is not surprising nor strange that fundamental properties have little or nothing to do with the behavior at higher levels.

  106. David says: “in fact, it is the exception and not the rule that in nature things divide in hierarchies with interesting complex emergent behavior at different scales that is independent of the behavior at more fundamental levels.”

    And you guys argue with me that there hasn’t been such a huge paradigm shift in science such as I tried to point to in my part # 4!!!!!

    I rest my case. This could not be more different from the worldview suggested by Newtonian science and held in the West in the 18th and 19th centuries. I will state my case in part # 4 even more strngly than I did (before the scientists objected, though with considerably more technical correctness, hurray, hurrah!

    On the physics, let me digest what you’re saying….)

  107. The eigenstate definition is clear to me. Thanks!

    This, not so much:
    “The equation we solve to find what wavefunctions are and how they evolve behaves in exactly the same way as classical waves. It exhibits the same properties as classical waves. One of the main properties of all waves is the superposition principle. This superposition is what leaves to a lot of quantum “weirdness”. Superposing things we think cannot exist at the same time, but they do.”

    This equation you mention is the Schrodinger equation, right? A classical linear equation. It works fine until you get to the measurement. (Penrose has a great diagram of this, you humanists out there.) After the measurement is made, however, the Schrodinger equation starts to work again until the next measurement takes place. You and Gavin are discussing various ways to think about the measurement problem. Right? (But are you always talking about a single particle? We are never thinking about psi states of many particles in this discussion?)

    Okay. But this wavefunction we are talking about with the particle/wave is one that applies to where a single particle will show up on the screen. Obviously a classical ocean wave involves many, many molecules and energy passing through those molecules while the molecules themselves remain in roughly the same place. The wave shows up in many places if you hold up a screen up to the wave!

    Penrose says if the wavefunction were a classical one that dots might show up on the screen wherever there is a probability density and maybe even two dots might appear. This does not happen (and it would violate conservation law). This is why, I gather, the wave function is called holistic as the entire span of the wave refers to a single particle. (He also talks about the wave function being spread over the entire universe…. You guys are avoiding “nonlocality” a bit, aren’t you? Maybe not. Just teasing.)

    Now let me quote you again:

    “One of the main properties of all waves is the superposition principle. This superposition is what leads to a lot of quantum “weirdness”. ”

    You are saying no problem, just a classical wave with classical superposition and then you say this leads to quantum weirdness. So I’m not following you here. Maybe you need to do classical superposition again — or refer me to where you covered this above. (I will review the whole conversation as soon as I have time.)

    Then could you compare/contrast the quantum wavefunction superposition to the classical? (I think this is what Penrose is trying to talk about when he calls it holistic.)

    In other words, is it easier to think about superposition with classical waves than with the quantum wave function? Why?

    Surely the beam-splitter experiment seems counter-intuitive. The “neat little wave-packet,” as Penrose describes it, starts off from the source and then divides into something like two waves at the beam splitter. (Is this neat little wave-packet were Schrodinger gives us bundles of excitations or probability densities?) But the “particle” so to speak doesn’t split in two. It goes one way or the other, depending on what test is waiting at the end of the experiment, as if the particle “feels out the way” to the end beforehand. (Or feels out both ways to the end.) Hmmm.

    I hope you can discern better than me where I’m having my problems following and what will help. Some parts of this are certainly getting clearer to me! Thanks!

    By the way, did you guys learn this stuff without having the experience of doing experiments to help you enact the various strands of this problem and grapple with it spread over many courses of action? That helps so much. I try to learn baseball rules without having played baseball and it is very hard. To my son, who’s played since he was four, it is self-evident. Again, this interaction of theory and practice bears on our task of describing how different disciplines pursue coming to know their own elegant formalities.

    Penrose talks about how elegant the geometries and algebras of quantum descriptions are. The problem comes in figuring out what you do about the “reality” of the quantum phenomena itself, I gather. You know there is elegant formality there, because the math comes out with such elegant and exciting kinds of precision. You want to suppose the elegant formality you see in the descriptions/formulas is reflecting something that is there in the particle/waves themselves, so to speak.

    However, this can be so difficult to see that some of you like Niels Bohr say stop thinking of a reality being there. You become positivists and devote yourselves to continuing to develop the math as long as its predictions continue to be accurate — or better, lead to new ranges or dimensions of accurate predictions, pushing back the frontiers so to speak. Others like Gavin resort to crediting the reality of the superpositions, which leads to a kind of reality of many worlds (sort of a platonism)?

    But I bet you both will balk at this because “all we want is the facts, ma’am, nothing but the facts.” Does anyone else remember Jack Webb? A perfect hard-boiled/soft-headed confrontation there, enacted every week in the 50s?

    Sorry. I got carried away.

  108. David

    As far as we can tell the particle behaves as if it went both ways unless you perform a measurement to determine which way it went at the splitting point. It doesn’t matter if you looked at the result or not. If you do a measurement at the end you will not know which way it went and ti will behave just as a superposition of going both ways.

    I am confused about your comment about Bohr. If the predictions are true and we can explain what has already measured how is it that stopping to think reality is there. The only way we can know we are close to reality is first by making successful prediction and second by accounting about what we already know. Following the math with no predictions is not taking account of reality, but with predictions and checking if the ydo happen is talking account of reality!

    I’m about to go to bed because here in Hollabd is 12:40 and tomorrow I have to wake up early. I’ll say more tomorrow.

  109. Gavin

    Janet,

    You say:

    But I am even more fascinated by how Gavin can say that none of this has any bearing whatever on anything else. We are talking about the fundamental properties of the material universe on its deepest level!!!

    Cool, isn’t it. I think you are right to see this as a paradigm shift. I don’t think the importance of emergent behavior is appreciated by many people, including many physicists. However, it is a central concept in modern physics and is extremely powerful. It’s a good thing too, because we will probably never really know the fundamental rules of the universe. Particle accelerators, which are currently our best tool for studying the fundamental rules discover new particles and symmetries at higher and higher energy levels. At some point, we just wont be able to afford a bigger accelerator, and we won’t know what particles and symmetries exist at even higher energies.

    So the good news is you can study the mind or economics without having to worry about the mass of the Higgs Boson, but in return don’t go around saying that this or that discovery about the Higgs Boson supports your model of consciousness or plan for utopia. Also keep in mind that, although economics is independent of the details of the fundamental theory, the fundamental theory probably has a couple profound consequences (like conservation of energy) that must be respected by economics. (Perpetual motion machines are economically favorable, but physically impossible. In conflicts like these, the more fundamental theory wins: no perpetual motion machines.)

    David,

    You say:

    The equation we solve to find what wavefunctions are and how they evolve behaves in exactly the same way as classical waves. It exhibits the same properties as classical waves.

    This makes me a bit uncomfortable. I know what you are saying, but I think it might be misleading. First, because when you go to measure position eigenstates (with a piece of film, for example) they don’t behave as waves. More importantly, when you have more than one particle the wave function is not a wave in space, but in configuration space, which is quite different (many more dimensions). So the wave is local in configuration space, but this may be very non-local in regular space.

    Gavin

  110. David

    It was not my intention to be misleading. I think Gavin’s comment is clear. Yes this waves can be very nonlocal in regular space.

  111. HI

    Janet,

    Regarding “Reading Lolita in Tehran.” It is frightening that totalitarianism forces you a way of living that is painful and values that you don’t agree. But what is also frightening is that those in a totalitarian regime rely on certain values and rules so much that they stop thinking and lose scope. Wasn’t the trial of “Gatsby” on a charge of it being immoral funny? But that kind of thinking, or rather, non-thinking is used to justify horrible things. And it doesn’t only happen in Iran. You can find it in Japan or US and in movements of all kinds whether they are right-wing or left-wing, religious or non-religious. Labels like “immoral,” “unpatriotic,” “un-American,” “bourgeois,” etc. are thrown to silence the opponents. You can find it even in people with good intentions, perhaps even more so when they are idealistic and overzealous.

    I am enjoying this discussion, even if we don’t always agree. I hope I don’t sound like those radical Muslim students in the class of professor Nafisi.

  112. the poet

    HI, just wanted to chip in here, as when I read Reading Lolita in Tehran, I loved it because I completely identified with it. I teach Gatsby at a small community college in a working class town (it prides itself on being a “milltown”). I constantly encounter students who want to resist the novel on the grounds of its marital infidelities, and furthermore on the grounds of what they see as the basic immorality of a leisure class. Never mind that they (my students) are young and seem to spend a disproportionate amount of their time reconstituting a jazz age! So often, as I try to very gently teach, we are not “reading” but “judging.” We practice what Charles Baxter calls Algebrization (I think): to whit, we say, Oh, he’s _____, the way a tenth grader says X = Y x M, or Yes, she’s ______, and then STOP THINKING, when what is the study of literature but an invitation to keep thinking. What are the possibilities here? (And math allows the same kind of process as one comes to a deeper study of it.)

  113. Hurray, “a poet” is chiming in here! Hurray, hurrah! See how we can all enter into conversation around a book? (Or around a Platonic dialogue, I hope.)

    Come on now you humanists and theorists and you former students of mine — from 25 years ago! — out there (who tell me you are all following along), speak up now! We really need your voices in here and over at the Socrates discussions…..

    Hi, let me assure you that you NEVER sounded like some of those obstreperous and close-minded students in Prof Nafisi’s class! Sometimes you sounded a wee bit like me, in fact — perhaps just a little bit too “idealistic and overzealous”! I am so glad to hear you are enjoying the conversation — and not simply “grimly” forcing yourself to attend to it, in order to set the record staight and keep things honest — though you do tend to accomplish those things.

    By the way, the Renaisance Christian humanists (1450-1650) were teachers of rhetoric and literature, working out of Plato & Aristotle’s theory of the liberal arts, who pointed out that human beings don’t learn only didactically and expositorily, but even more dynamically from recognizing themselves being imitated (“possibly”)in the works of others and drawing parallels and homologies….

    Aristotle, after all, said in his great thought work on Poetics that poietike arises out of two fundamental human drives: to learn by imitating (mimesis) and to be compelled to delight and take enjoyment in every kind of elegant formal order (harmonia).

    So when Hi wondered, for instance, if he was like some of those students in _Reading Lolita in Tehran_, he was illustrating something we all do all the time, and that also happens in Plato’s Ion — that when we see others speaking and doing, we make all kinds of mimetic applications — both within the literary work (dialogue) and to our own situations outside of it. Plato is so consistently aware of this and using this that I don’t see how we can believe he was unaware of an “art of poetry” that worked by a mimetic way of knowing and thinking by the mind…

    A classic instance of this learning-by-mimesis is in the opening scetion to Sir Thomas More’s Utopia, where the “young Thomas More” refrains from voicing something at the close, because of something Hythloday had said about other folks earlier in the conversation! And the humanist More’s book as a whole, of course, is a great Mimesis of an Ideal Polis, just like Plato’s Republic, and full of the same sorts of endlessly intriguing puzzles over how we are to interpret and apply “what the poet meant.” (So you have to be reading the Wily Socrates posts, if this remark interests you!)

    By the Eighteenth Century, by the way, the humanists’ powerful Greek mimesis was deader than a doornail. The rise of Science obliterated it, by returning education to a new ascendency of linear logic and methodical step-by-step learning — in the name of scientific rationallism this time, instead of in the name of medieval didacticism! (Once again, literary language becomes simply a matter of “ornament” and not a way of knowing — see Francis Bacon.)

    By the 19th century, the Romantics resurrected something akin to this formally dynamic way of knowing that had guided earlier Western thought, this sense of apprehending patterned wholes and ascending to greater levels of insight through them — calling it the “Imagination”!

    We today use both words — rationality and imagination, oppositionally or together — to describe what in the Greeks was not separated out, to begin with. I call this dynamical formal and metaphorical mental activity “coming-to-know a formal kind-of-thing in terms of its “white lightning,” that elegant formality that “lights up” in particular objects whereby we “know” what formal kind of thing it is — in addition to its ipseity or uniqueness as an individual…. To apprehend a thing and NOT know its formal identity as a kind-fo-thing is terrifying and uncanny. This is the moment of origin for Greek knowing: there is order (white lightning) in the world!

    (P.S. I go off on these tangents with purpose aforethought! Every time I approach again some aspect of a counter-intuitive (to us!) idea or way of thinking, from outside our own cultural context and training, each time in a somewhat different way and from a different starting point, this increases the chances that something will “click” for you. If not, just enjoy the “thunder, thunder, over thunder road” that I can’t stop singing since Rick sent us over there….)

    P.P.S. You, gentle readers, are wonderfully tolerant of my attempts to build a new vocabulary we can use between us. Thank you. (Either that, or you are just ignoring my attempts!)

  114. “A poet” says that we tend to work an equation to get an concrete result, and then —

    “we STOP THINKING, when what is the study of literature but an invitation to keep thinking. What are the possibilities here? (And math allows the same kind of process as one comes to a deeper study of it.)”

    And math is such a great example — here’s a formal discipline that dreams up and pursuesits own possible elegant formal orders as far as they will go, and then suddenly discovers that one of these math-systems imitates or models some part of physical reality!

    You see how for Aristotle there is no large difference between the compulsions driving the mathematicians and those driving the literary artist? Both are trying to come to know, to develop their own precise way of thinking and knowing, because they are entranced with a formal possibility of a precise and elegant kind and the community is eager to work it through. “Because all human beings love to learn by mimesis and are drawn to the formal beauties of harmonia!” (Poetics, Bk IV).

    If we make this the formal foundation of all liberal arts, then we can add in the strengths of science (objectivity of results because of universal apllicability of experimental verification in this field, and comrehensiveness, etc.) while still assuming it is inherently limited — in that it is devoted to formalizing its own kind of thing — a huge kind of thing (physical reality) but not all of reality….

    “Reality” is a word causing us problems here…. For the Greeks it was the Formal Real as opposed to the Actual. For us. pur “reality” is the Actual, to which the real formalisms sometimes cannot seem to get us (as in QM, where the mathematical formalisms that trace or imitate for us the situation do not seem to get us all the way to an Actual situation, or to a situation we can conceive of as actual….)

  115. David says —

    “I am confused about your comment about Bohr. If the predictions are true and we can explain what has already measured how is it that stopping to think reality is there. The only way we can know we are close to reality is first by making successful prediction and second by accounting about what we already know. Following the math with no predictions is not taking account of reality, but with predictions and checking if the ydo happen is talking account of reality!”

    Bohr was so thrown by the QM situation that he asserted as the “Copenhagen interpretation” of QM that the mathematical formalizations had “nothing to do with reality” — a first in the history of physics, I believe. But he thus must have become the father of multitudes — the positivists like Stephen Hawking who say, all that matters is that the math makes accurate predictions and “he doesn’t know what reality is.”

    I like David’s description of “The only way we can know we are close to reality is first by making successful prediction….” This makes perfect sense to me. But it shows up, too, that “knowing we are close to reality” is the normative expectation these days, and KNOWING THE REALITY itself is only a devout hope, when once it was taken to be a scientific certainty.

    In a different but related vein, I often read that cosmology and QM are now so complex mathematically that the human mind cannot “conceive” of the realities to which the math apparently refers — and in that sense, what we used to mean by “reality” in science has become peculiarly irrelevant. What human mind can “conceive” of a reality corresponding to of all those dimensions in string theory, for example? Surely this is part of why positivism has become so popular, as a position held by scientists today (yet unheardof in the first 200 years of science).

    Does it seem strange to you as scientists that science sometimes must conceive of the physical subject matter in PURELY mathematical terms that can’t correspond to any kind of reality we have direct knowledge of? Or have we just absorbed this and take it for granted?

  116. I want to go back to a point that Gavin made earlier in this thread.. He quotes me first:

    “You [Janet] say:

    But I am even more fascinated by how Gavin can say that none of this has any bearing whatever on anything else. We are talking about the fundamental properties of the material universe on its deepest level!!!

    [Gavin continues) Cool, isn’t it. I think you are right to see this as a paradigm shift. I don’t think the importance of emergent behavior is appreciated by many people, including many physicists. However, it is a central concept in modern physics and is extremely powerful. It’s a good thing too, because we will probably never really know the fundamental rules of the universe. Particle accelerators, which are currently our best tool for studying the fundamental rules discover new particles and symmetries at higher and higher energy levels. At some point, we just wont be able to afford a bigger accelerator, and we won’t know what particles and symmetries exist at even higher energies.”

    First, I’m interested that Gavin is one of those physicists who doesn’t think that — probably — we ever will know all about the most fundamental physical properties of the universe.

    (Okay, now I go into a meditation on the burgeoning realms of what we do not know or even “cannot know.” Then I get back to Gavin.)

    In my own work, this is my intuition, too, when I look at the long picture of Western history. We started out entranced by formalizing the mystery of formal order in the world, with the Greeks, who saw it everywhere and who created geometry and formalized number theory as no other culture had ever done, as far as we know. (But look, right there is another thing we will never know all about but we have so much here yet to learn — previous cultures. At 3 Quarks Daily. I saw a piece on the Dead Sea scrolls that said most of the mss are still in storage boxes and haven’t been looked at yet!!)

    Then we saw the rise of science and the rise of a confidence that we could figure it all out and arrive at a “sum total of human knowledge” that would pretty much close the books. The universe was steady-state and deterministic and physics seemed to show that complexitiy could be reduced to very simple and elegant formulas. Words like “absolute” and “universal” (in its modern, not medieval sense) entered the Western vocabulary.

    Then, in the 20th century, we discovered in field after field that huge mysteries remained, and the more we have pursued them, the more intricasy and plenitude of structures seem to arise. It’s everywhere you look. The sub-atomic world, as Gavin points out. The world of viruses. The unknown species of the rain forests and the oceans. (I saw an Imax movie on the ocean once that said there were symbiotic organisms made up of trillions of little independent critters working together, and the entire structure was larger than 2 or 3 football fields. ????)

    What about the fact that we have mitochondria running our cells? Then you turn to the realm of the very large and every ten years our picture of the story and destiny of the universe is shifting (okay, I exaggerate, but wow!) and we have multiple universes and on and on and on. And “we cannot know” what happened at the singularity. We used to be sure that animals had no emotions and were just machines and we saw nature (with Darwin) as being entirely “red in tooth and claw,” locked into a brutal struggle for the survival of the fittest, and now we learn more and more about social and emotional development of animals, even altruism.

    It goes on and on. The more we look, the more mystery of formal structure there actually is. This is (or could be) a return to the Greco-European attitude, what I call “the stance of the question” or even “the stance of the question-prayer” — as opposed to “the stance of the (logical) declarative sentence” of the modern era — and it can make life very very meaningful to be on this quest if we could be comfortable with the existential mystery of it all and not keep saying, no mystery here at all. (Okay, okay, but we need logical and scientific too, or we see how global warming can be dismissed and ignored.)

    Okay, enough already. I wanted to ask Gavin (or anyone else) if this powerful principle of emergent behavior in physics has anything to so with the new trend in biology to treat cognition on its own level? It seems very novel to me that Dawkins (with his notorious memes) and Dennett and Hofstadler, while affirming as scientists that of course it all is in some sense reducible to biology and then to chemistry and then to physics, seem nonetheless comfortable now with treating cognitive phenomena on their own level, so to speak, rather than strictly “reductively.”

    And Hi spoke up for chemistry, as I recall, and affirmed that chemistry has its own level that needs study and is not simply physics, even through physics is the fundamental level. This notion of levels with their own emergent phenomena seems perhaps more accepted in science these days, andmore braodly than just in physics, or am I mixing apples and oranges again? I’m wondering if Gavin or others have a sense of this. Is physics perhaps once again leading the way to a re-evaluation or re-conceptualization on all levels and in all fields of science?

    Recent cognitive science represents some kind of big paradigm shift, doesn’t it?

    Previously, I got the feeling that looking at cognition on its own level might have seemed sort of taboo, because it might be tantamount to letting in “metaphysical entities” or the “soul” or our famous little “gnomes with shovels,” but now scientists are bravely tucking it up and wading into the cognitive waters and letting mental phenomena be mental phenomena. Am I way off base here?

  117. Gavin

    Janet,

    You have asked me about something outside my field, so I welcome the input of an expert. I know little about cognitive science in particular, but certainly computer science has done exactly what you describe, allowing the discussions of programing to be totally separated from and understanding of computer hardware. In fact, it has been mathematically proven that the hardware is irrelevant. Any computer meeting certain basic requirements is a “universal computer” and it can do the same things that any other universal computer can do, whether it is built of transistors or Tinker-Toys. Computer science is independent of the material science and electrical engineering on which computers are based.

    Brains are not universal computers because they are asynchronous. Non-the-less, I don’t think that many people believe that the only way to make a brain is out of nerve cells. It seems quite likely that we could make an asynchronous computer out of transistors. I’m not an expert, but it seems to me that the study of asynchronous computers is not going to care what those computers are made of. So I would say that yes, the study of the brain’s cognitive function is independent of the study of the brain’s biochemistry.

    However, even with my ignorance I see some danger here. Universal computers run programs. Our brains are hardwired to a large extent and can change the wiring to change the behavior, so the connection between the hardware and the performance is closer than in universal computers. Also changes in chemistry (the use of drugs, for example) affects brains in a way that has no analogy in universal computers. I don’t know what to make of that.

    Biology is messy like this, with the levels not separating cleanly. This is part of what makes biology so vexing, and wonderful.

  118. Gavin,

    Can you explain more what you mean when you say brains are asynchronous,and how that contrasts with synchronous computers?

    So is making an asynchronous computer the next step toward AI?

  119. Gavin

    Again, I’m outside my field, so if take this with a grain of salt. I’m also going to be speaking very generally.

    Computers have a clock that determines the cycles of the computer. On each cycle the processor takes a string of input, acts on it, produces an output, and gets ready for the next input. All of the transistors in the computer are acting in lock step, taking inputs and producing outputs in time with the clock. This is synchronous.

    Nerve cells do not operate on the cycles of a clock. When they get enough input from other nerve cells they just fire. They don’t wait for the output part of the cycle (because there is no cycle) they just do it. This means that brains are not universal computers, which obey a clock.

    I don’t know what the consequences of this are. It is possible that that there are no interesting consequences at all (but I doubt it). I don’t think that producing an asynchronous computer is going to be especially important for AI, but I don’t know. What I do know is that when Penrose observes that brains are not universal computers and then suggests that maybe this is because of some sort of gravitation induced wave function collapse issue he is way off base. The reason that brains are not universal computers is far more mundane: they are asynchronous. But, like I said, I don’t know how important this fact is. I doubt that it is either crucially important or totally irrelevant.

  120. As far as I can tell, he is trying to get at the strangeness or problem of the holistic nature of brain reaction with the holistic nature of quantum wave functions (I have no idea how gravity comes in, except that’s your problem with coming up with a GUT, right?) — that every part of a section of the brain changes and stores information and so maybe it involves a quantum state kind of thing. The other part that’s enticing him is entanglement — he just thinks more must be going on at a more fundamental level, BENEATH the wave behavior and then the collapse behavior, something that includes both in a more comprehensive picture (you have to like the goal, at least, right Gavin? comprehensiveness?)

    Anyway, give him credit for keeping his own theories separate from is explanations of QM for general readers!

  121. The synchronous/asynchronous distinction seemed very clear to me, Gavin. Thanks.

    I have a broader question for the physicists, sort of about the natural world as a whole, so to speak. I’ve always studied each part of physics by itself, piecemeal, so to speak. Now I find myself looking at everything with new eyes and actually wanting to apply more of this to what I see.

    Maybe it’s a broader disquisition on “waves” that I want.

    Pseudonym gave us the wonderful example of the butter gun for the inverse square law. Now that applies to all waves, and not just electro-magnetic waves? And all waves are quantum phenomena? (Shooting the butter isn’t a “real” wave — it’s just an illustration, right? I know this may sound silly, but there’s a serious question here if you can see where I’m at.)

    David said that we could think of quantum waves as traveling through the universe, in a sense, just as water waves are energy traveling through the medium of water.

    So what is the range or domain of the inverse square law? Which phenomena does it apply to, and which does it not apply to? (Wind, for instance, is a movement of air molecules from high pressure zones to lower pressure zones. It has nothing to do with waves, right? But then it encounters water and transfers some of its energy into waves there? Is this transfer happening on the atomic level, with the electrons and phontons in the atoms?)

    What about Newton’s “inverse square law” for gravitational fall? Is that a completely separate law that just happens to be an inverse square law?

    The sun gives off photons, obviously, in the form of light waves. An atom will give off photons and electrons too, as waves (or wave-packets). So can you describe what form the energy was in before these wave/particles were emitted, in each case?

    Basically, how many basic different things do we have going on here in the physical world around us, in terms of the dynamics we see going on everyday? I mean, I know the names of the basic physical forces and the lack of a GUT, but could you kind of just describe the world and where each of the forces is operating…. In ten words or less?

    One more example. The mitochondria in our cells produce energy, right? At what level of physics is that energy production going on? We are “powered” electrically, aren’t we? So where are the “waves” in these processes. We digest food, which has stored energy in it. Again, where are the waves going on in this? We are getting into chemistry and biology here, and I don’t expect an exposition of everything, but where do the quantum waves come in here? On the level of the atom?

  122. David

    Janet,

    You seem to have shot off a bunch of questions. I will try to tackle some.

    First, one of the principles of science is energy conservation. Energy is conserved. The evidence for this principle has accumulated over centuries in countless experiments
    in all fields. So even though I cannot assert definitively that it is an absolute truth, it is certainly truer and has a lot more weight of evidence that anything else we know. If one were to think it is an absolute truth, one would be a lot closer to reality than questioning it mildly.

    So even though people might speak of energy production there is no process in nature, no process in chemistry, no process in biology, no process in physics that violates energy conservation. So when we digest food we break food to get some of the energy stored in food. Similar processes occur in cells to break down chemicals. The only relevance of quantum mechanics to this processes is to explain and account for the energy stored in th chemicals. Well indirectly it most also play a role in the reaction of the chemical being broken down, but one need not understand this process in detail to explain predict account for the chemical processes one one knows the energies, the energy barriers, and the reaction rates.

    Before the energy was emitted in the process you described can be accounted for. For example if an atom is an excited state the electron in the atom has more kinetic and potential energy than the minimum possible. It emits a photon with the extra energy and goes to a lower energy state. In the sun the main process is nuclear fusion. The nuclei before being fuse have more kinetic and potential energy than when fused. They fuse and release the extra energy.

    I couldn’t follow about the inverse squared law because you jumped to talking about the wind and water. Huh?

    As far as the wind and water, all of their processes can be understood without invoking any atomic physics. Bot the air and water are fluids. Things like wind, a current of air, can happen in water although we do not call it wind. While both are made of atoms and the transfer of energy will involve the atoms, it can be understood, studied and predicted except in the most extreme conditions by studying the hydrodynamics of two fluids separated by an interface. This is an example of emergent laws which Gavin has alluded too. To study and understand this we need not allude to any quantum mechanics.

  123. David

    Gravity and electromagnetism operate at long ranges, weak and strong interaction at short
    distances. At super short distances maybe at the GUT scale maybe at the Planck scale, or even maybe at a lower scale the quantum physics that describes these forces cannot be extrapolated anymore. What happens there, no one knows. Maybe the notion of these forces survives but quantum mechanics breaks down, maybe there is a GUT, maybe these forces get supplanted by something else. No one knows.

  124. David

    Not all waves are quantum phenomena directly. OK water waves are made of things that are quantum mechanical. But one can understand and study water waves without any quantum mechanics because is emergent behavior.

  125. This is great! Thank you so much. Has anyone written a book just describing the processes going on in a landscape, say, based on physics, chemistry and biology? For humanists and poets?

    About the inverse square law, I was thinking back to the wave function equation (Schrodinger equation) used for an emitted electron or photon before it is measured. This is a classical linear function, you guys said, and applied to all waves, not just quantum mechanics…?

    And don’t all waves spread out according to an inverse square law illustrated by the butter gun? They dissipate with distance, like the butter being spread thinner over four pieces of bread at a greater distance than hitting one piece of toast closer up? (Pseudonym, where are you ? You gave us this example.) But if the wave happens to be the one described by an electron wave-function, then when it is measured it is suddenly a particle, but is found in a position that accords with the probabilities of the Schrodinger equation? The particle isn’t spread out, like a wave, but it is found in a position that is described by the probabilities of the wave function?

    If the Schrodinger equation gives you the correct probabilities by treating the electron as though it were a wave, that is a choice the physicist makes because it gives correct results. The electron phenomenon when measured always acts like a particle. The problem comes in that if only one slit is open the particle lands where it would if it were not a wave. (You use a different classical equation for this, correct?) But if both slits are open the measured particle lands where it would given the Schrodinger probabilitites?

    And I’m wondering where the inverse square law fits into this. Does it predict the range or spread of places the particle will be found?

  126. David

    Ahhhh this is somewhat clearer now. Yes the Schrodinger equation is not different in principle than other waves. This does not mean that all waves are quantum mechanical. It means that quantum mechanical waves have features that are universal to wave phenomena. When it comes to measurement as far as we know it either is not described by the wave equation (Copenhagen collapse) or we must concentrate on only one of the branches (many worlds interpretation that Gavin likes) that a system can take.

    Your inverse square law describes how, if you create a localized wave, it spreads out. This is common to all waves, For single particle quantum mechanics this would describe the spread of the probabilities.

  127. David says: “When it comes to measurement as far as we know it either is not described by the wave equation (Copenhagen collapse) or we must concentrate on only one of the branches (many worlds interpretation that Gavin likes) that a system can take.”

    This is the clearest statement for me of anything yet! (I think because I can visualize it.) And this is describing the case of a single particle, where the Schrodinger equation gives the spread of probabilities (or it gives the spread of the various “branches” or “paths” the system might allow the particle to take)?

    Now I want to know about the case where you are interested in describing the behavior of many particles and not just a single one.

    Let’s say we have light emanating from a source. Are all of these photons being emitting taken together also behaving as “waves”?

    Because it seems like, as the electromagnetic waves spead out traveling away from the source, there will be fewer and fewer particles for a given plane-area set perpendicular to the “wave” as it is spreading out? It seems like there will be an inverse square law applying to the particles considered together (and not individually). (?)

    So does this make a bunch of particles a wave?

    This is what I was trying ineffectually to ask some time ago. Are a bunch of quantum particles moving out from a light source a “wave,” and if so, do they then have their own description with a Schrodinger equation just as a water wave would? And if so, is the equation this time describing the spread of the particles, whereas with a single particle it is describing the spread of the probabilities or “branches”?

    This is where I am still a little confused on the subject of waves and quantum waves.

    Does QM always consider the single wave-particles, or do you also deal with describing waves (or some sort of system) of many wave-particles?

  128. I should have said “if so, is the equation this time describing the spread of the particles, whereas with a single particle it is describing the spread of the probabilities or “branches” FOR THAT SINGLE PARTICLE?

    And I should also have asked, is quantum weirdness seen with measuring the single particle, but not when dealing with the behavior of many particles, which is perhaps more classical, sort of like waves involving many units of energy spreading out?

  129. David

    It depends what you mean by quantum weirdness. The behavior of a single particle and of many particle system is both described by waves. The outcomes of measurements are probabilistic in similar fashion. The “measurement problem” is similar.

    The difference is that when we have more than one particle there are nonlocal effects like entanglement that are not accessible for a one particle system.

    The waves for a many particle system are not waves in the space we live in, but in something called configuration space which is made of the coordinates of all the particles. Gavin mentioned before when he reprimanded me for being misleading. When we work out the consequences for what happens in our space, we obtain nonlocality and other phenomena that do not happen for a single particle system.

  130. David

    QM describes both wavees for a single particle and also for many particle systems as implied by my last post, Janet. The second is harder to treat, but has more interesting effects I believe.

  131. Gavin

    Thanks, David, for jumping in here. I haven’t had much time.

    Janet,

    This is all very tricky, in part because there are two different ways to approach all of it, with quantum mechanics, or with quantum field theory. Quantum mechanics treats all of the particles as particles, not waves. The photon is not an electromagnetic wave in quantum mechanics, it is just a particle. However, that particle’s position is described by the wave function, which is governed by the Schroedinger equation. For one photon the wave function is a wave in 3 dimensional space. For two photons it is not two waves in 3 dimensional space, it is a single wave in six dimensional space. For 100 photons it would be a wave in 300 dimensional space. This is obviously a complicating factor and makes visualization pretty hopeless. This still gives rise to wave-like behavior in three dimensions, but it can give rise to more complicated, non-local appearing effects as well.

    Quantum mechanics does not work well with special relativity and has problems with massless particles like photons. The problems won’t affect any of the things you are interested in, but they are serious enough that quantum mechanics is not the correct way to describe our world.

    If you decide to go with quantum field theory then you treat photons as excitations of the electromagnetic field. The wave function then lives in an infinite dimensional space, giving probabilities for every possible shape of the electromagnetic field. This causes challenges not just for visualization, but also at a mathematical level. However, those challenges haven’t prevented the theory from doing a great job predicting experimental results and won’t interest us at all.

    There is an excellent approximation that can be made for photons in quantum field theory, where the photons are quantum excitations of the waves in the electromagnetic field. (This approximation works well for particles that are moving freely, not bound to anything else.) In that case we describe the state of the system (the wave function) by stating how many excitations (photons) there are and what each photons momentum is. We don’t actually attempt to write down the whole wave function in infinite dimensional space.

    Adding more particles allows new types of quantum weirdness that you do not see with a single particle. Entanglement is the most basic, Bose-Einstein condensation is one of the more complicated. However, in most cases adding more particles has the effect of decreasing the amount of quantum weirdness. Lots of new subtle effects are available in many particle systems, but they are generally not relevant to the issues of experimental interest, so the quantum nature of the system effectively goes away.

    Finally, the inverse square law doesn’t specifically apply to waves. The inverse square law applies to anything spreading out in all directions. For example, light from a spotlight does not fall off in the way that the inverse square law suggests because the light is all being directed in one direction. The inverse square law applies to gravity and electric fields when they are spreading out in all directions from a roundish object. The field of a long thin object or a big flat object will not follow the inverse square law because the field cannot spread out in as many directions. The field created by the weak and strong forces also don’t follow the inverse square law because the fields can’t reach very far, so they don’t continue to spread in all directions. The field is stuck close to the source.

  132. This was really, really helpful and clear. Thank you so much.

    Gavin says: “The inverse square law applies to gravity and electric fields when they are spreading out in all directions from a roundish object. The field of a long thin object or a big flat object will not follow the inverse square law because the field cannot spread out in as many directions.”

    So is reference to gravitational fields here, the reason that the formula for gravitational acceleration obeys the inverse square law (Newton’s law)?

    Also, Gavin says “Finally, the inverse square law doesn’t specifically apply to waves. The inverse square law applies to anything spreading out in all directions.”

    And so, does this mean that the butter gun example is simply a visual way for students to REMEMBER the proportionalities described by an inverse square law? But shooting butter from a gun with a rectangular opening wouldn’t really obey this law, not because butter isn’t a wave, but because it isn’t spreading out in all directions? (Sometimes humanists are way too literal?)

    But remind me, what spreads out in all directions that is not a wave?

  133. Gavin

    Janet,

    You ask some well thought out questions:

    “So is reference to gravitational fields here, the reason that the formula for gravitational acceleration obeys the inverse square law (Newton’s law)?”

    Yes. The field is spreading out in all directions and this give rise to the inverse square term in Newton’s Universal Law of Gravitation.

    “And so, does this mean that the butter gun example is simply a visual way for students to REMEMBER the proportionalities described by an inverse square law? But shooting butter from a gun with a rectangular opening wouldn’t really obey this law, not because butter isn’t a wave, but because it isn’t spreading out in all directions?”

    Yes. Real butter guns would probably not follow an inverse square law. Butter is not a wave, unless you want to treat the butter using quantum field theory, which is a very bad idea. (I couldn’t quite tell if that was part of the question.)

    “(Sometimes humanists are way too literal?)”

    (Moving on….)

    “But remind me, what spreads out in all directions that is not a wave?”

    The gravitational and electric fields are not waves. You can make waves in these fields, but the fields of a massive or charged object is a not wave. (The fields can be thought of as the result of virtual gravitons or photons, but this is really a math trick. Virtual particles aren’t really waves either.) Explosion fragments often travel out in all directions. The 4th of July is a good time to observe this, although some fireworks only spread in a plane to make circles an other flat patterns.

  134. HI

    This is a little old topic, but what mitochondria does is to produce ATP using energy from foods. ATP is a very convenient source of energy for cells to use. So, the role of mitochondria is like that of a power plant that burns coal and oil to generate electricity, which is convenient for us to use. Actually, it is more like a hydroelectric power plant, because it uses flow of protons to produce ATP. How scientists found out how this happens has a fascinating history.

    Going back to the topic of QM measurement, I feel that Copenhagen interpretation is not satisfactory for the same reason Gavin mentioned awhile back. On the other hand, many-worlds interpretation seemed too weird to me. But perhaps I don’t really understand decoherence. The July 5th issue of Nature celebrated the 50th anniversary of the many-worlds interpretation. It is interesting that the many-worlds interpretation is gaining popularity.

  135. Thanks!
    Both the mitochondria and many worlds are unthinkable revelations from a historical perspective. This open-ended exploration we’re now on is so much more fascinating than having everything pinned down and the books closed, which was what I was told as a child….

  136. David

    I do not think Copenhagen is satisfactory. But putting aside the weirdness of many worlds I just do no see much difference between the Copenhagen view, one the system becomes entangled with the measurement apparatus you collapse the wave function and stay with the piece that corresponds to your measurement and the many worlds view once the system becomes entangled with the apparatus we concentrate and on the branch that corresponds to the measurement. To me while in principle very different, in practice they are quite similar to me and more importantly, you cannot tell them apart via measurement as far as I can tell.

  137. So, I was going to ask this question but David suggests the answer. When you folks say “Copenhagen,” you mean basically the complementarity principle, right? That you USE the wavefunction model and math until you measure, then you work with the collapse of the wave function?

    So, if I understand your terminology correctly (a big “if”), whether you are talking about a single particle’s collapse of the wave function, or an entire system that becomes entangled and then through decoherence you get the (appearance of) a collapse of the wavefunction — either way, basically you go from wave functions to collapsed state in your maths and that’s what you mean by “Copenhagen”?

    So when you say “Copenhagen,” you might be thinking about decoherence (in the case of measuring a system) or simply about the collapse of the wavefunction with measurement of a single particle, but either way, this is what you call “Copenhagen” because you simply “accept” the collapse and then go on from there?

    In other words, with “Copenhagen,” you use two different, seemingly incompatible, but “complementary” (Neils Bohr’s term) models — wave and particle — by moving from wave model to particle model at measurement.

    Whereas many worlds uses a different interpretive “explanation,” but are the maths different?

  138. David

    Collapse of course causes decoherence, but one can have decoherence without collapse.

    When we collapse the wavefunction (which could be a single particle’s wave function or the wavefunction of a many particle system) by hand because there was a measurement we go from wavefunction to a wave function with the piece that does not correspond to the measurement chopped off.

    I am somewhat busy today. So if this is not clear cause I am writing in a hurry, someone else might jump in. If not, I’ll try to be clearer tomorrow.

    But at a measurement we go from wavefunction to wavefunction

    If the original wavefunction is | a ) + | b ) and we measure the system in a, we use the wavefunction | a ). This is collapse in simplistic terms.

    many worlds would say we are in branch |a ) and don’t feel | b ) but the wavefunction contains both parts. This is overtly simplistic as we need to include the apparatuses wavefunction in both versions to be correct and complete.

    The math of how the wavefunctions evolve are identical in both versions before and after collapse, basically the Schrodinge equation.

  139. David

    I do not, but they are physicist friends of mine. Good friends! They decided to start a blog. Well I might have something to do indirectly since I was talking to them about our discussion here and saw in their first post that some of my ideas were in there.

Leave a comment

This site uses Akismet to reduce spam. Learn how your comment data is processed.